You are on page 1of 220

AAOS

A M E R IC AN A CAD EM Y OF O RTH OP AE DIC S U R GE ON S


Your Source for Lifelong Orthopaedic Learning

Adult Reconstructive
Surgery of the

1
Hip
Answer Book and
Knee

AL-Madena Copy

2
2010 Adult Reconstructive S urgery of the Hip and Knee Examination Answer Book • 7

Question 1
During the course of a revision total knee arthroplasty via a medial parapatellar exposure, the surgeon does a
complete intra-articular release and synovectomy but exposure is still inadequate. A quadriceps snip is performed
and, at the end of the procedure, the knee is stable throughout a range of motion and the postoperative radiographs
show acceptable alignment of the components. The patient’s postoperative physical therapy regimen should
include which of the following?

1. No restriction in range of motion or weight bearing after surgery.


2. Limit flexion to 30 degrees postoperatively, progressing 10 degrees per week
3. Limit flexion to 90 degrees for the first 6 weeks postoperatively
4. Limit to active flexion only with no passive flexion or active extension for 6 weeks
5. Use of a hinged knee brace for 6 weeks postoperatively

PREFERRED RESPONSE: 1

DISCUSSION: A quadriceps snip is performed by extending a medial parapatellar approach superiorly and
laterally across the quadriceps tendon. It is then repaired primarily at the end of the procedure. The primary
advantage of this technique over other surgical maneuvers that improve exposure at the time of revision total knee
arthroplasty is that the postoperative regimen for physical therapy does not need to be altered.

REFERENCES: Younger AS, Duncan CP, Masri BA: Surgical exposures in revision total knee arthroplasty. J Am
Acad Orthop Surg 1998;6:55-64.
Della Valle CJ, Berger RA, Rosenberg AG: Surgical exposures in revision total knee arthroplasty. Clin Orthop
Relat Res 2006;446:59-68.
Barrack RL, Smith P, Munn B, et al: The Ranawat Award. Comparison of surgical approaches in total knee
arthroplasty. Clin Orthop Relat Res 1998;356:16-21.

Question 2
A healthy 72-year-old woman is seen 14 days after cemented total knee arthroplasty. She reports increasing pain
and swelling for the last 4 days accompanied by 4 days of wound drainage. Examination reveals that she is
afebrile, and has erythema and moderate serosanguinous drainage from the wound. The knee is moderately
swollen. Aspiration of the knee reveals no organisms on Gram stain. Culture results are expected back in 48 hours.
Optimal management should consist of

1. initiation of a first-generation cephalosporin while awaiting culture results.


2. initiation of broad-spectrum antibiotics while awaiting culture results.
3. ultrasound to evaluate for fluid collection around the knee.
4. surgical debridement of the knee before culture results are available.
5. inpatient observation and no antibiotics until culture results are available.

PREFERRED RESPONSE: 4

AL-Madena Copy

3
8 • American Academy of Orthopaedic S urgeons

DISCUSSION: Increased pain, swelling, erythema, and drainage 2 weeks removed from the primary arthroplasty
are all signs of a probable infection. Erythrocyte sedimentation rate and C-reactive protein may not be helpful as
they are elevated postoperatively even in the absence of infection. Even in the absence of infection, persistent
wound drainage is an indication for surgical debridement to prevent subsequent infection. When a postoperative
infection is easily recognized by clinical examination, there is no need to wait for a positive culture before
proceeding with debridement.

REFERENCES: Weiss AP, Krackow KA: Persistent wound drainage after primary total knee arthroplasty.
J Arthroplasty 1993;8:285-289.
Jaberi FM, Parvizi J, Haytmanek CT, et al: Procrastination of wound drainage and malnutrition affect the outcome
of joint arthroplasty. Clin Orthop Relat Res 2008;466:1368-1371.
Insall JN, Windsor RE, Scott, WN: Surgery of the Knee, ed 2. New York, NY, Churchill Livingstone, 1993, pp
959-964.

Figure 3a Figure 3b
Question 3
A 72-year-old woman underwent a primary total hip arthroplasty 14 months ago. She states that the hip has now
dislocated four times when rising from a low chair, requiring closed reduction. A radiograph is shown in Figure
3a and a CT scan of her pelvis is shown in Figure 3b. What is the most reliable method for rectifying her
instability?

1. Use of an abduction orthosis for 6 weeks


2. Modular exchange of the femoral head to a 36-mm head and a longer neck
3. Modular exchange of the polyethylene liner to a constrained acetabular insert
4. Revision and repositioning of the acetabular component and use of a 36-mm femoral head

5. A physical therapy program stressing abductor strengthening PREFERRED RESPONSE: 4


DISCUSSION: The radiograph shows well-fixed components without evidence of loosening. The CT scan shows
severe retroversion of the acetabular component. Revision of the component into the correct amount of anteversion
will most reliably rectify the instability in the face of severe component malposition.
AL-Madena Copy

4
2010 Adult Reconstructive S urgery of the Hip and Knee Examination Answer Book • 9

REFERENCES: Parvizi J, Picinic E, Sharkey PF: Revision total hip arthroplasty for instability: Surgical
techniques and principles. J Bone Joint Surg Am 2008;90:1134-1142.
DeWal H, Su E, DiCesare PE: Instability following total hip arthroplasty. Am J Orthop 2003;32:377-382. Barrack
RL, Booth RE Jr, Lonner JH, et al (eds): Orthopaedic Knowledge Update: Hip and Knee Reconstruction 3.
Rosemont, IL, American Academy of Orthopaedic Surgeons, 2006, pp 475-503.

Question 4
A patient with a history of rheumatoid arthritis reports a painful total hip arthroplasty 3 years after the index
procedure. Radiographs reveal loosening of the femoral component. Preoperative blood work shows an
erythrocyte sedimentation rate (ESR) of 38 mm/h (normal 0-29 mm/h) and a C-reactive protein (CRP) of 8.9 (0.2-
8.0). What is the most appropriate action at this time?

1. Technetium bone scan


2. Hip aspiration for culture
3. FDG-PET scan
4. Surgery with no further investigations
5. Revision surgery and obtain an intraoperative frozen section

PREFERRED RESPONSE: 2

DISCUSSION: The question centers on the appropriate work-up for a failed total hip arthroplasty prior to revision
surgery. The preoperative ESR is elevated and the CRP is at the upper end of normal. If either the ESR or CRP is
elevated, further investigations are required to exclude infection as a cause of loosening, particularly in a patient
only 3 years after the index procedure. A technetium scan alone is nonspecific and will show increased uptake
because of the loose femoral component. An intraoperative frozen section is a helpful confirmatory investigation,
but whenever possible the diagnosis should be made preoperatively to allow for appropriate surgical planning.
Recently, investigators have shown the value of FDG-PET scanning as a useful investigation for diagnosing
infection; however, it is no more accurate than the combined use of an ESR and CRP, and does not allow for
identification of an infecting organism. At this point, a hip aspiration for culture is the most appropriate
investigation.

REFERENCES: Bauer TW, Parvizi J, Kobayashi N, et al: Diagnosis of periprosthetic infection. J Bone Joint Surg
Am 2006;88:869-882.
Pill SG, Parvizi J, Tang PH, et al: Comparison of fiuorodeoxyglucose positron emission tomography and (111
)indium-white blood cell imaging in the diagnosis of periprosthetic infection of the hip. J Arthroplasty
2006;21:91-97.
Spangehl MJ, Masri BA, O’Connell JX, et al: Prospective analysis of preoperative and intraoperative
investigations for the diagnosis of infection at the sites of two hundred and two revision total hip arthroplasties. J
Bone Joint Surg Am 1999;81:672-683.

AL-Madena Copy

5
10 • American Academy of Orthopaedic S urgeons

Figure 5a Figure 5b

Question 5
A 7 5-year-old man who sustained an intertrochanteric hip fracture underwent open reduction and internal fixation
with a sliding hip screw. Six months after the procedure, the patient has shortening and external rotation of the
extremity and progressively severe groin pain with ambulation. Radiographs are shown in Figures 5a and 5b.
What is the most appropriate management?

1. Valgus/flexion osteotomy of the proximal femur with repeat open reduction and internal fixation
2. Conversion to bipolar hemiarthroplasty with a cementless femoral component
3. Conversion to total hip arthroplasty with a calcar replacement femoral component that bypasses the
hardware
4. External bone stimulator

5. Removal of hardware, followed by physical therapy and use of a shoe lift PREFERRED

RESPONSE: 3
DISCUSSION: The patient has an intertrochanteric fracture malunion with protrusion of the hardware and
penetration into the acetabulum. To restore leg length and relieve pain, total hip arthroplasty is necessary. Valgus
osteotomy is appropriate for fracture nonunion with an intact femoral head with no signs of osteonecrosis. Bipolar
hemiarthroplasty with acetabular erosion will most likely lead to pain as will removal of the hardware with or
without physical therapy.

REFERENCE: Said GZ, Farouk O, El-Sayed A, et al: Salvage of failed dynamic hip screw fixation of
intertrochanteric fractures. Injury 2006;37:194-202.

Figure 6

AL-Madena Copy

6
2010 Adult Reconstructive S urgery of the Hip and Knee Examination Answer Book • 11

Question 6
Figure 6 shows the radiograph of a 72-year-old woman who underwent a primary total hip arthroplasty
17 years ago. She now reports groin pain. Optimal surgical management should consist of which of the following?

1. Synovectomy and polyethylene liner exchange


2. Synovectomy, polyethylene liner exchange, and femoral stem revision
3. Synovectomy and complete acetabular revision
4. Synovectomy, bone grafting of lytic lesions, and retention of the components and
polyethylene liner
5. Revision of all components with synovectomy

PREFERRED RESPONSE: 1

DISCUSSION: Polyethylene wear is evident due to the superiorly eccentric position of the femoral head within the
acetabulum. Despite proximal femoral osteolysis, the component appears well fixed, as does the acetabulum. The
acetabular component appears to be well positioned. Therefore, an isolated synovectomy and polyethylene liner
exchange is indicated. If the hip is stable, there is no need for more extensive revision work.

REFERENCE: Barrack RL, Booth RE Jr, Lonner JH, et al (eds): Orthopaedic Knowledge Update: Hip and Knee
Reconstruction 3. Rosemont, IL, American Academy of Orthopaedic Surgeons, 2006, pp 521-528.
Question 7
Which of the following statements best describes how unicompartmental knee arthroplasty (UKA) differs from
total knee arthroplasty (TKA)?

1. TKA has a higher reoperation rate than UKA during the first 10 postoperative years.
2. TKA provides for more normal rotational kinematics than UKA.
3. UKA more closely replicates normal knee kinematics than TKA.
4. Mobile bearings have been successful in UKA but not TKA.
5. Mobile bearings have been successful in TKA but not UKA.

PREFERRED RESPONSE: 3

DISCUSSION: Because UKA does not require cruciate sacrifice, patellofemoral resurfacing, or rotational changes
to the femur or tibia, it reliably recreates normal knee kinematics. UKAs have generally demonstrated higher
reoperation rates than TKAs at intermediate and long-term follow-up, due in part to progression of arthritis in the
nonresurfaced compartments. Mobile bearings have been clinically successful in both UKA and TKA.

REFERENCES: Patil S, Colwell CW Jr, Ezzet KA, et al: Can normal knee kinematics be restored with
unicompartmental knee replacement? J Bone Joint Surg Am 2005;87:332-338.
Gioe TJ, Killeen KK, Hoeffel DP, et al: Analysis of unicompartmental knee arthroplasty in a community- based
implant registry. Clin Orthop Relat Res 2003;416:111-119.

Question 8
Which of the following is associated with the use of large femoral heads in total hip arthroplasty?

AL-Madena Copy

7
12 • American Academy of Orthopaedic S urgeons

1. Increased risk of total hip dislocation


2. Decreased range of motion
3. Fewer options for femoral head bearing material
4. Acceptable wear with modern bearing surfaces
5. Increased component impingement

PREFERRED RESPONSE: 4

DISCUSSION: Larger diameter femoral heads reduce the risk of hip dislocation by allowing greater range of
motion before component impingement becomes a risk, and no reports have shown an increase in bearing wear.
With modem bearings, volumetric wear should remain low, despite the increase in head diameter. Large femoral
heads are available in all common bearing materials

REFERENCES: Peters CL, McPherson E, Jackson JD, et al: Reduction in early dislocation rate with large-diameter
femoral heads in primary total hip arthroplasty. J Arthroplasty 2007;22:140-144.
Inoue A, Asaumi K, Endo H, et al: Assessment of head wear more than ten years after total hip arthroplasty: 22-
mm zirconia vs metal heads. Acta Med Okayama 2006;60:311-318.

Figure 9

Question 9
A 68-year-old man underwent a primary total hip arthroplasty 2 years ago for a femoral neck fracture. His early
postoperative course was unremarkable, but he notes some aching in the thigh since surgery. His symptoms have
gotten worse over the last year, such that he now has activity-related thigh pain that limits his walking ability. An
AP hip radiograph is shown in Figure 9. What is the most appropriate surgical management?

1. No surgery is indicated
2. Revision of the acetabular component
3. Revision of the femoral component
4. Psoas tendon tenotomy

5. Strut grafting of the femur to stiffen the bone near the tip of the implant

PREFERRED RESPONSE: 3

DISCUSSION: The radiograph shows a loose femoral component with failure of osseous integration.
There is a reactive radiolucent line around the fiber mesh portion of the implant, hypertrophy of the bone under the
collar, and a pedestal formation at the distal tip of the implant, all of which indicate failure of bone ingrowth. If

AL-Madena Copy

8
2010 Adult Reconstructive S urgery of the Hip and Knee Examination Answer Book • 13

bone ingrowth had occurred, there would be stress shielding of bone under the collar.

REFERENCES: Engh CA, Massin P, Suthers KE: Roentgenographic assessment of the biologic fixation of
porous-surfaced femoral components. Clin Orthop Relat Res 1990;257:107-128.
Lieberman JR, Huo MH, Schneider R, et al: Evaluation of painful hip arthroplasties: Are technetium bone scans
necessary? J Bone Joint Surg Br 1993;75:475-478.
Vresilovic EJ, Hozack WJ, Rothman RH: Radiographic assessment of cementless femoral components:
Correlation with intraoperative mechanical stability. J Arthroplasty 1994;9:137-141.

Figure 10

Question 10
Figure 10 shows patellar radiographs of a 68-year-old woman who underwent bilateral total knee arthroplasty 2
months ago. Following a recent fall onto the left side, she now reports anterior pain in the left knee. A CT scan
shows that the femoral and tibial components are appropriately externally rotated and radiographs show acceptable
axial alignment and no evidence of loosening. What is the most appropriate treatment option?

1. Fracture fixation and bracing


2. Lateral retinacular release with proximal realignment
3. Tibial component revision
4. Distal realignment by medialization of the tibial tubercle
5. Revision of the patellar component

PREFERRED RESPONSE: 2

DISCUSSION: Treatment of patellofemoral instability after total knee arthroplasty (TKA) is directed by its
etiology. In instances of component malpositioning, revision of one or both components is indicated.
If the components are determined to be in satisfactory position, soft-tissue procedures can be pursued. Lateral
retinacular release is usually the first soft-tissue procedure used to improve patellofemoral mechanics. In this
patient, the patellar fracture fragment is so small that it can be excised. Distal realignment is not usually used as the
first line of treatment for patellar maltracking following TKA.

REFERENCES: Fehring TK, Christie MJ, Lavemia C, et al: Revision total knee arthroplasty: Planning,
management, and controversies. Instr Course Lect 2008;57:341-363.
Patel J, Ries MD, Bozic KJ: Extensor mechanism complications after total knee arthroplasty. Instr Course Lect
2008;57:283-294.

Question 11
Which of the following best describes the legal definition of standard of care?

1. Conforming to a majority norm


2. Providing the most minimally acceptable care

AL-Madena Copy

9
14 • American Academy of Orthopaedic S urgeons

3. Providing average care


4. Providing the care you would expect for yourself or a loved one
5. Reasonable treatment that exhibits knowledge, skill, diligence, and care

PREFERRED RESPONSE: 5
DISCUSSION: The standard of care is a legal concept that is elusive and amorphous, although the term is used
widely by physicians to mean different things. Different state courts across the United States have also applied
different meanings to the term “standard of care.” Most commonly, the standard of care is that which a reasonable
physician would have done under similar circumstances. Expert testimony from other physicians is often required
to educate a jury in a medical malpractice trial about the applicable standard of care. As a general rule, treatment
that exhibits knowledge, skill, diligence, and care on the part of the physician is likely to fall within the standard of
care, regardless of variations in the definition of this term.

REFERENCES: Lewis MH, Gohagan JK, Merenstein DJ: The locality rule and the physician’s dilemma: Local
medical practices vs the national standard of care. JAMA 2007;297:2633-2637.
AAOS Expert Witness Program, www3.aaos.org/member/expwit/expertwitaess.cfm

Question 12
Which of the following statements most accurately describes the risk of ileus following total joint arthroplasty?

1. Older age decreases risk.


2. Male gender decreases risk.
3. The risk is roughly 1% in total joint arthroplasty patients.
4. The risk more commonly occurs in total knee arthroplasty (TKA) patients than in total hip
arthroplasty (THA) patients.
5. A history of abdominal surgery has no effect on risk.

PREFERRED RESPONSE: 3

DISCUSSION: The risk of postoperative ileus is noted to be higher in patients undergoing THA than patients
undergoing TKA. Older age, male gender, and a history of abdominal surgery have been identified as risk factors.

REFERENCE: Parvizi J, Han SB, Tarity TD, et al: Postoperative ileus after total joint arthroplasty. J Arthroplasty
2008;23:360-365.
Question 13
During surgical hip dislocation for the management of femoral acetabular impingement, preservation of what
structure is paramount to maintaining vascularity to the femoral head?

1. Metaphyseal vessels
2. Medial epiphyseal artery
3. Superficial branch of the medial femoral circumflex artery
4. Deep branch of the lateral femoral circumflex artery
5. Deep branch of the medial femoral circumflex artery

PREFERRED RESPONSE: 5

AL-Madena Copy

10
2010 Adult Reconstructive S urgery of the Hip and Knee Examination Answer Book • 15

DISCUSSION: When a trochanteric osteotomy is performed with the desire to maintain vascularity to the
femoral head, as in the approach for a surgical hip dislocation, the deep branch of the medial femoral c ircumflex
artery must be maintained. This branch courses along the posterior aspect of the greater trochanter, posterior to
the tendon of obturator extemus, and anterior to the tendons of superior gemellus, obturator intemus, and inferior
gemellus. It perforates the capsule above the superior gemellus and distal to the tendon or piriformis, before
dividing into two to four terminal retinacular branches. Maintaining the attachment of the external rotators
maintains the blood supply to the femoral head. Additionally, the superior-lateral retinacular vessels must also
be maintained during femoral neck osteoplasty.

REFERENCES: Gautier E, Ganz K, Krugel N, et al: Anatomy of the medial femoral circumflex artery and its
surgical implications. J Bone Joint Surg Br 2000;82:679-683.
Ganz R, Gill TJ, Gautier E, et al: Surgical dislocation of the adult hip: A technique with full access to the femoral
head and acetabulum without the risk of avascular necrosis. J Bone Joint Surg Br 2001; 83:11191124.

Figure 14

Question 14
A 68-year-old woman who underwent left total hip arthroplasty 14 years ago now reports pain in her thigh. A
radiograph is shown in Figure 14. What is the most reliable method for reconstructing the femoral component?

1. Revision to a standard length cemented femoral component using third generation cementing
techniques
2. Revision to a proximal femoral replacement device (“tumor prosthesis”)
3. Revision to a proximally porous-coated, metaphyseal loading cementless femoral component
4. Revision to a diaphyseal engaging, extensively coated cementless femoral component
5. Revision to an allograft-prosthetic composite

PREFERRED RESPONSE: 4

DISCUSSION: The patient has a loose, cemented femoral component. The proximal femur is deficient and may
not have adequate strength to support a proximally porous-coated cementless implant. Cemented hip implants
have shown higher rates of failure when used for revision surgery. An extensively coated, cementless stem that
engages the diaphysis has been shown to have excellent long-term durability in patients such as this, where the
diaphysis is preserved and more than 4 cm of intact diaphysis is available for fixation. Allograft-prosthetic
composites and tumor replacement prostheses are used for patients with more severe bone loss of the proximal
femur and would not be indicated for this patient.

REFERENCES: Barrack RL, Booth RE Jr, Lonner JH, et al (eds): Orthopaedic Knowledge Update: Hip and

AL-Madena Copy

11
16 • American Academy of Orthopaedic S urgeons

Knee Reconstruction 3. Rosemont, IL, American Academy of Orthopaedic Surgeons, 2006, pp 457474.
Della Valle CJ, Paprosky WG: The femur in revision total hip arthroplasty: Evaluation and classification. Clin
Orthop Relat Res 2004;420:55-62.
Engh CA Jr, Ellis TJ, Koralewicz LM, et al: Extensively porous-coated femoral revision for severe femoral bone
loss: Minimum 10-year follow-up. J Arthroplasty 2002;8:955-960.

Figure 15

Question 15
A 71-year-old woman with coronary artery disease underwent an uncomplicated right total hip arthroplasty for
osteoarthritis 12 years ago. Her hip has functioned well until approximately 18 months ago when she noted the
spontaneous onset of groin, buttock, and proximal thigh pain that is present at rest and made worse with activity. A
radiograph is shown in Figure 15. What is the recommended management at this point?

1. Immediate admission to the hospital and emergent revision hip arthroplasty


2. Reassurance and follow-up if symptoms worsen
3. Repeat radiographs in 1 month
4. Protected weight bearing with urgent revision hip arthroplasty when the patient is medically
cleared
5. A prescription for alendronate and reevaluation in 1 year

PREFERRED RESPONSE: 4

DISCUSSION: The radiograph shows significant osteolysis with loosening of the femoral component. The patient
is symptomatic and surgery is indicated because of the extent of osteolysis and the loose femoral component.
Reassurance and follow-up if symptoms worsen places the patient at risk for further bone loss and periprosthetic
fracture. Emergent surgery is not required because the symptoms have been present for more than a year; however,
urgent revision hip arthroplasty is recommended when the patient is medically cleared. While there is data to
suggest that bisphosphonates may slow the progression of osteolysis in animal modes, there is no clear evidence
that bisphosphonate treatment prevents the progression of osteolysis in humans. Additionally, this patient has a
loose symptomatic femoral component.

REFERENCES: Chiang PP, Burke DW, Freiberg AA, et al: Osteolysis of the pelvis: Evaluation and treatment.
Clin Orthop Relat Res 2003;417:164-174.
Dunbar MJ, Blackley HR, Bourne RB: Osteolysis of the femur: Principles of management. Instr Course Lect
2001;50:197-209.
Rubash HE, Dorr LD, Jacobs JJ, et al: Does alendronate inhibit the progression of periprosthetic osteolysis? Trans

AL-Madena Copy

12
2010 Adult Reconstructive S urgery of the Hip and Knee Examination Answer Book • 17

Orthop Res Soc 2004;29:1888.

)
Figure 16a Figure 16b

Question 16
A 35-year-old man reports a 2-year history of right groin pain. The pain is made worse with hip flexion, prolonged
sitting, and cycling. A radiograph and MRI scan are shown in Figures 16a and 16b. Nonsurgical management has
failed to provide relief. What is the best surgical option?

1. Arthroscopic labral debridement


2. Reverse periacetabular osteotomy
3. Resurfacing hip arthroplasty
4. Femoral neck osteochondroplasty and resection of the detached labrum
5. Femoral neck osteochondroplasty and reattachment of the labrum
PREFERRED RESPONSE: 5

DISCUSSION: The patient has cam-type femoral acetabular impingement. He still has a well-maintained joint space
without significant degenerative changes, and given his age a joint preserving procedure would be the procedure of
choice. A reverse periacetabular osteotomy may be considered in a retroverted acetabulum; however, that is not the
case here. A femoral neck osteochondroplasty is required to remove the cam of bone and reshape the femoral head-
neck junction to improve the femoral head/neck ratio (femoral head offset). Typically, in isolated cam impingement,
cartilage damage in the anterior-superior acetabulum precedes labral damage. Labral debridement alone does not
address the pathology of impingement. In cases where labral detachment is present, reattachment has been shown to
be superior to labral resection.

REFERENCES: Espinosa N, Rothenfluh DA, Beck M, et al: Treatment of femoro-acetabular impingement:


Preliminary results of labral refixation. J Bone Joint Surg Am 2006;88:925-935.
Parvizi J, Leunig M, Ganz R: Femoroacetabular impingement. J Am Acad Orthop Surg 2007;15:561-570. Trousdale
RT: Acetabular osteotomy: Indications and results. Clin Orthop Relat Res 2004;429:182-187.

Figure 17a Figure 17b Figure 17c

Question 17

AL-Madena Copy

13
18 • American Academy of Orthopaedic S urgeons

A 51-year-old woman who underwent a total knee arthroplasty 14 months ago for severe degenerative arthritis
now reports progressive pain, swelling, and buckling of the knee. She must use crutches and is unable to
negotiate stairs. Laboratory testing reveals a normal erythrocyte sedimentation rate and C-reactive protein.
Radiographs of the patient are shown in Figures 17a through 17c. What is the most important test to further
evaluate this problem?

1. Long standing anterior-posterior radiograph of the hip-knee-ankle


2. Axial CT views from the supracondylar distal femur to the proximal tibia below the tibial tubercle
3. Fluoroscopic stress views to demonstrate the position of subluxation
4. Bone scan
5. MRI scan

PREFERRED RESPONSE: 2

DISCUSSION: The cause of subluxation in this patient is multifactorial, and includes a laterally positioned patellar
component, a tibial tray that is internally rotated and translated to the medial side of the proximal tibial surface, and a
femoral component that is markedly internally rotated about 10 degrees. All of these findings will be apparent on a
CT scan. The long standing radiograph may be helpful but does not show the particular rotational abnormalities of
both implants that are causing this problem. Fluoroscopic review may show how unstable the patella is, but the initial
Merchant’s view shows the basic problem. A bone scan does not provide information about component malposition.
An MRI scan is inferior to a CT scan because of image artifact.

REFERENCES: Stiehl JB: Patellar instability in total knee arthroplasty. J Knee Surg 2003;16:229-235. Berger RA,
Crossett LS, Jacobs JJ, et al: Malrotation causing patellofemoral complications after total knee arthroplasty. Clin
Orthop Relat Res 1998;356:144-153.
Question 18
Changes to the properties of ultra-high molecular weight polyethylene with increasing irradiation dose include
improved

1. resistance to oxidation.
2. ultimate tensile strength.
3. resistance to crack propagation.
4. fracture toughness.
5. volumetric wear.

PREFERRED RESPONSE: 5

DISCUSSION: Increased irradiation doses cause a decrease in the mechanical properties of the polyethylene,
resulting in a decrease in ultimate tensile strength, fracture toughness, and resistance to crack propagation.
Irradiation leads to the production of free radicals, requiring a step in the manufacturing process (melting,
annealing, vitamin E doping) to stabilize the free radicals and reduce the potential for oxidation. Wear resistance is
improved with irradiation; however, there is minimal benefit with doses of greater than 10 Mrads.

REFERENCES: Collier JP, Currier BH, Kennedy FE, et al: Comparison of cross-linked polyethylene materials for
orthopaedic applications. Clin Orthop Relat Res 2003;414:289-304.
Gordan AC, D’Lima DD, Colwell CW Jr: Highly cross-linked polyethylene in total hip arthroplasty. J Am Acad

AL-Madena Copy

14
2010 Adult Reconstructive S urgery of the Hip and Knee Examination Answer Book • 19

Orthop Surg 2006;14:511-523.


Jacobs CA, Christian CP, Greenwald AS, et al: Clinical performance of highly cross-linked polyethylenes in total
hip arthroplasty. J Bone Joint Surg Am 2007;89:2779-2786.

Question 19
A 72-year-old man with a history of Parkinson’s disease, stable coronary artery disease, and mild renal
insufficiency is seen for hip arthroplasty. Which of the following is considered the most appropriate bearing of
choice?

1. Ceramic-on-ceramic
2. Large diameter metal head on highly cross-linked polyethylene liner
3. Ceramic head on metal liner
4. Large diameter metal-on-metal total hip arthroplasty
5. Metal on conventional polyethylene liner (noncross-linked)

PREFERRED RESPONSE: 2
DISCUSSION: The most appropriate bearing of those listed would be a large diameter metal head on cross-linked
polyethylene. Because of the renal insufficiency, metal-on-metal should be avoided because metal ions are renally
excreted. Ceramic-on-ceramic implants are not necessary in this patient because of the patient’s limited activity
and life expectancy, as well as the downside of increased cost, the small but definite risk of ceramic fracture, and
the increased technical demands of inserting ceramic implants. A highly cross-linked liner is favored to allow for
maximum head size as the patient may be at increased risk of dislocation because of his neurologic disorder.

REFERENCES: Bragdon CR, Greene ME, Freiberg AA, et al: Radiostereometric analysis comparison of wear of
highly cross-linked polyethylene against 36- vs 28-mm femoral heads. J Arthroplasty 2007;22:125-129.
MacDonald SJ: Metal-on-metal total hip arthroplasty: The concerns. Clin Orthop Relat Res 2004;429:86- 93.

Question 20
A 52-year-old man who weighs 325 lb is wheelchair-bound from severe degenerative arthritis of the left hip.
Twenty-four hours after cementless total hip arthroplasty, he develops shortness of breath and evaluation shows a
saddle pulmonary embolus. The patient is started on enoxaparin sodium at 150 mg every 12 hours. Two days
later, the patient’s hematocrit is 20% despite four units of transfused packed cells, and he now has developed a
complete sciatic nerve palsy. What is the best course of action?

1. Emergent exploration of the sciatic nerve


2. Transfusion to raise the hematocrit to 30% and sequential neurovascular examinations
3. Placement of a vena cava filter, halt anticoagulation, blood transfusion, and exploration of the sciatic
nerve
4. Transfusion to raise the hematocrit to 30%, continued administration of enoxaparin, and sequential
neurovascular examinations
5. Placement of a temporary vena cava filter and exploration of the sciatic nerve

AL-Madena Copy

15
20 • American Academy of Orthopaedic S urgeons

PREFERRED RESPONSE: 3

DISCUSSION: The purpose of this question is to draw attention to the early risks of therapeutic anticoagulation
that will be instituted by an intensivist or pulmonologist to treat a life-threatening pulmonary embolus. The
temporary vena cava filter is a recent innovation but will effectively reduce the risk of further pulmonary emboli.
This requires reversal of anticoagulation for safe insertion of the filter and creates a safe situation for additional
surgical solutions. Sciatic nerve compromise was caused by the expanding hematoma in this patient, which could
be mitigated by exploration both to assess the nerve and to remove a large hematoma that presents its own long-
term risks.
REFERENCES: Della Valle CJ, Steiger DJ, Di Cesare PE: Thromboembolism after hip and knee
arthroplasty: Diagnosis and treatment. J Am Acad Orthop Surg 1998;6:327-336.
Weil Y, Mattan Y, Goldman V, et al: Sciatic nerve palsy due to hematoma after thrombolysis therapy for acute
pulmonary embolism after total hip arthroplasty. J Arthroplasty 2006;21:456-459.
American Academy of Orthopaedic Surgeons Guideline on the Prevention of Symptomatic Pulmonary
Embolism in Patients Undergoing Total Hip or Knee Arthroplasty, www.aaos.org/research/guidelines/
PEguide.asp

Question 21
What is the most common cause of reoperation following contemporary hip resurfacing for degenerative
coxarthrosis?

1. Limb-length discrepancy
2. Altered femoral offset
3. Snapping psoas tendon
4. Limitation of hip movement
5. Femoral neck fracture
PREFERRED RESPONSE: 5

DISCUSSION: Femoral neck fracture, necessitating revision surgery to a total hip arthroplasty, is the most likely
common early complication after hip resurfacing. Cortical notching, varus positioning, and other technical variables
appear to predispose to this complication. Limb lengths do not change appreciably after hip resurfacing; the same
applies to femoral offset. Hip movement is not limited by hip resurfacing, and there is no evidence that psoas
tendon impingement is increased by hip resurfacing.

REFERENCES: Steffen RT, Pandit HP, Palan J, et al: The five-year results of the Birmingham Hip
Resurfacing arthroplasty: An independent series. J Bone Joint Surg Br 2008;90:436-441.
Amstutz HC: Present state of metal-on-metal hybrid hip resurfacing. J Surg Orthop Adv 2008;17:12-16.
Question 22
A 77-year-old man with a history of mild renal insufficiency and atrial fibrillation on warfarin therapy is
scheduled to undergo a left total hip arthroplasty. He previously underwent a right total hip arthroplasty with
development of significant heterotopic bone that resulted in limitation of motion. What is the most appropriate
form of prophylactic treatment to minimize the formation of heterotopic bone on his left hip?

1. Postoperative indomethacin for 3 weeks

AL-Madena Copy

16
2010 Adult Reconstructive S urgery of the Hip and Knee Examination Answer Book • 21

2. Postoperative indomethacin for 6 weeks


3. No treatment indicated; can treat later if heterotopic bone forms
4. 800 centigrey of radiation given to the periprosthetic soft tissues preoperatively on the morning of
surgery

5. 400 centigrey of radiation given to the periprosthetic soft tissues day 2 postoperatively

PREFERRED RESPONSE: 4
DISCUSSION: This question centers on the prophylactic treatment to reduce the risk of heterotopic bone
formation. Prophylaxis is indicated because he has already demonstrated bone formation with his prior hip
arthroplasty, which places him at increased risk for developing heterotopic bone on the contralateral side. He is
on warfarin and has renal insufficiency, which makes the use of NSAIDs contraindicated. The recommended
dose is 600 to 800 centigrey of radiation given within 24 hours of surgery preoperatively or 72 hours
postoperatively.

REFERENCES: Kolbl O, Knelles D, Barthel T, et al: Preoperative irradiation versus the use of nonsteroidal
anti-inflammatory drugs for prevention of heterotopic ossification following total hip replacement: The results
of a randomized trial. Int J Radiat Oncol Biol Phys 1998;42:397-401.
Pakos EE, Ioannidis JP: Radiotherapy vs nonsteroidal anti-inflammatory drugs for the prevention of heterotopic
ossification after major hip surgery: A meta-analysis of randomized trials. Int J Radiat Oncol Biol Phys
2004;60:888-895.
Seegenschmiedt MH, Makoski HB, Micke O, et al: Radiation prophylaxis for heterotopic ossification about the
hip joint: A multicenter study. Int J Radiat Oncol Biol Phys 2001 ;51:756-765.

AL-Madena Copy

17
22 • American Academy of Orthopaedic S urgeons

Figure 23 a Figure 23b

Question 23
A 46-year-old male construction worker has right hip pain that has failed to respond to nonsurgical management.
His body mass index (BMI) is 32, he is 6’2” tall, and he has no other medical comorbidities. AP and lateral
radiographs of the right hip are shown in Figures 23a and 23b. The patient inquires about his suitability for metal-
on-metal hip resurfacing. The patient should be educated that he is at higher risk for failure secondary to which of
the following?

1. BMI >30
2. Presence of secondary changes of the acetabulum
3. Osteonecrosis of the femoral head
4. Age of younger than 55 years old
5. Male gender

PREFERRED RESPONSE: 3

DISCUSSION: This young patient has osteonecrosis of the femoral head with a large area of collapse.
The results of hip resurfacing arthroplasty have been reported to be best in young, male patients who are younger
than 55 years of age with a diagnosis of osteoarthritis. Although some authors advocate metal- on-metal hip
resurfacing as an option for patients with osteonecrosis of the femoral head, in this particular patient, given the
size of the necrotic segment, he would be at higher risk for failure and a conventional total hip arthroplasty would
be a more conservative option. As the acetabulum is resurfaced in metal- on-metal hip resurfacing, the secondary
changes of the acetabulum are not an issue and his BMI is in an acceptable range for the procedure.

REFERENCES: Mont MA, Ragland PS, Etienne G, et al: Hip resurfacing arthroplasty. J Am Acad Orthop Surg
2006;14:454-463.
Revell MP, McBryde CW, Bhatnagar S, et al: Metal-on-metal hip resurfacing in osteonecrosis of the femoral
head. J Bone Joint Surg Am 2006;88:98-103.
Buergi ML, Walter WL: Hip resurfacing arthroplasty: The Australian experience. J Arthroplasty 2007;22:61-65.
Question 24
A 31-year-old woman had disabling right knee pain. An arthroscopic assessment reveals chondromalacia of both
the lateral femoral condyle and tibial plateau. The standing femorotibial axis measures 10 degrees of valgus. The

AL-Madena Copy

18
2010 Adult Reconstructive S urgery of the Hip and Knee Examination Answer Book • 23

optimum treatment of this condition should include

1. distal femoral varus osteotomy.


2. osteoarticular transplant to the lateral femoral condyle.
3. unicondylar arthroplasty.
4. high tibial osteotomy.
5. Fulkerson tibial tubercle transfer.
PREFERRED RESPONSE: 1

DISCUSSION: The long-term outcome of a distal femoral varus osteotomy has been quite favorable and should
remain the primary choice for this young active woman. Sharma and associates have shown that a 5-degree
valgus malalignment has a five-fold chance of progressing at least one grade within 18 months, making a
corrective osteotomy the most important surgical maneuver.

REFERENCES: Sharma L, Song J, Felson DT, et al: The role of knee alignment in disease progression and
function decline in knee osteoarthritis. JAMA 2001 ;286:188-195.
Murray PB, Rand JA: Symptomatic valgus knee: The surgical options. J Am Acad Orthop Surg 1993; 1:19.

Figure 25a Figure 25b Figure 25c

Question 25
A healthy 78-year-old woman falls down a flight of stairs 2 years after undergoing left total hip arthroplasty.
Radiographs are shown in Figures 25a through 25c. Optimal management should include which of the
following?

1. Skeletal traction
2. Long stem femoral revision
3. Closed reduction and internal fixation with a retrograde femoral nail
4. Open reduction and internal fixation with a conventional plate and screws
5. Open reduction and internal fixation with a plate, proximal cables, and distal screws

PREFERRED RESPONSE: 5

DISCUSSION: The fracture occurs distal to the component, and does not jeopardize the cemented implant

AL-Madena Copy

19
24 • American Academy of Orthopaedic S urgeons
fixation. The cement mantle is intact. The component appears well fixed and without osteolysis. In such cases,
the femoral component can be left alone and the fracture fixed with internal fixation. There is no available
intramedullary space proximal to the fracture to allow for a retrograde nail. A standard plate cannot obtain
adequate fixation of the proximal fragment due to the presence of the femoral stem. Therefore, a plate that uses
screws distal to the fracture and cables proximal to the fracture is the treatment of choice. Skeletal traction is
usually reserved for patients unable to withstand surgery. This fracture is classified as a type C, according to the
Vancouver classification of postoperative femoral fractures.

REFERENCES: Barrack RL, Booth RE Jr, Lonner JH, et al (eds): Orthopaedic Knowledge Update: Hip and
Knee Reconstruction 3. Rosemont, IL, American Academy of Orthopaedic Surgeons, 2006, pp 475503.
Brady OH, Garbuz DS, Masri BA, et al: The reliability and validity of the Vancouver classification of femoral
fractures after hip replacement. J Arthroplasty 2000; 15:59-62.
Question 26
Osteolysis after total hip arthroplasty with polyethylene acetabular bearings is most closely correlated with which
of the following risk factors?

1. Patient weight
2. Femoral head bearing material
3. Linear wear rate
4. Decreased femoral offset
5. Increased femoral offset

PREFERRED RESPONSE: 3

DISCUSSION: The development of osteolysis appears to be multifactorial. Patient activity, component


positioning, polyethylene oxidation level, and bearing surface all appear to contribute. They contribute, however,
by increasing the rate of wear. Therefore, the one variable that correlates closest with the likelihood of osteolysis
(and the magnitude of osteolysis) is the wear rate of the bearing couple. Wear can be measured linearly or
volumetrically. Both correlate with the development of osteolysis.

REFERENCE: Barrack RL, Booth RE Jr, Lonner JH, et al (eds): Orthopaedic Knowledge Update: Hip and Knee
Reconstruction 3. Rosemont, IL, American Academy of Orthopaedic Surgeons, 2006, pp 521-528.

Question 27
Which of the following patients requires preoperative noninvasive cardiac testing?

1. 52-year-old man scheduled to undergo elective lumbar decompression and fusion, with a history of
hypercholesteremia, hypertension, and deep venous thrombosis following treatment of an ankle
fracture
2. 67-year-old woman scheduled to undergo total knee arthroplasty, with a history of myocardial
infarction, cerebrovascular accident, and diabetes mellitus
3. 68-year-old woman scheduled to undergo total hip arthroplasty for osteonecrosis, no known medical
comorbidities, but has not seen a physician in more than 20 years and drinks 7 to 10 alcoholic
beverages per week
4. 72-year-old man with an intertrochanteric hip fracture, with a history of chronic renal failure, colon

AL-Madena Copy

20
2010 Adult Reconstructive S urgery of the Hip and Knee Examination Answer Book • 25
cancer, and obesity
5. 81-year-old man with a fracture of the proximal femur, history of myocardial infarction, prostate
adenocarcinoma, and hypothyroidism, and prior to the fracture he was able to climb a flight of stairs

PREFERRED RESPONSE: 2

AL-Madena Copy

21
DISCUSSION: Noninvasive cardiac testing is recommended in the presence of the three or more of the following
risk factors in a sedentary patient: history of coronary artery disease or myocardial infarction, history of heart
failure, prior cerebrovascular accident, and diabetes mellitus or chronic renal disease. Preoperative stress testing is
not recommended if patients can perform moderate activities such as climbing a flight of stairs.

REFERENCES: Bushnell BD, Horton JK, McDonald MF, et al: Perioperative medical comorbidities in the
orthopaedic patient. J Am Acad Orthop Surg 2008;16:216-227.
Auerback A, Goldman L: Assessing and reducing the cardiac risk of noncardiac surgery. Circulation
2006;113:1361-1376.
Fischgrund JS (ed): Orthopaedic Knowledge Update 9. Rosemont, IL, American Academy of Orthopaedic
Surgeons, 2008, pp 105-113.

Figure 28

Question 28
A 62-year-old woman undergoes an uncomplicated primary metal-on-metal cementless hip arthroplasty. Her early
postoperative course is unremarkable. She returns at 1 year reporting groin pain. Her symptoms are made worse
with stair climbing and getting in and out of her vehicle. Examination reveals minimal discomfort with passive
range of motion, but straight leg raising reproduces her symptoms. A radiograph is shown in Figure 28. What is the
most appropriate surgical management?

1. Revision of the femoral component


2. Psoas tendon tenotomy
3. Exchange of the bearing to metal-on-polyethylene
4. Exchange of the bearing to ceramic-on-ceramic
5. Exchange of the bearing to a smaller femoral head and metal liner

PREFERRED RESPONSE: 2

DISCUSSION: The radiograph shows a cementless cup that is placed low within the native acetabulum and
appears large relative to the patient’s native acetabulum. The size and inferior position of the cup place it at risk
for causing psoas tendon irritation or impingement. Symptoms are typical for irritation of the psoas tendon and
not consistent with either a loose implant (more mechanical activity-related pain) or metal hypersensitivity
(more constant pain and not just associated with hip flexion activities). Controversy exists regarding tenotomy
versus acetabular revision.
REFERENCES: Dora C, Houweling M, Koch P, et al: Iliopsoas impingement after total hip replacement: The
results of non-operative management, tenotomy or acetabular revision. J Bone Joint Surg Br 2007;89:1031-1035.

22
2010 Adult Reconstructive S urgery of the Hip and Knee Examination Answer Book • 27

Heaton K, Dorr LD: Surgical release of iliopsoas tendon for groin pain after total hip arthroplasty. J Arthroplasty
2002; 17:779-781.
Malik A, Maheshwari A, Dorr LD: Impingement with total hip replacement. J Bone Joint Surg Am 2007;89:1832-
1842.
Trousdale RT, Cabanela ME, Berry DJ: Anterior iliopsoas impingement after total hip arthroplasty. J Arthroplasty
1995;10:546-549.

Question 29
Increasing tibial polyethylene conformity can have what effect on fixed bearing total knee arthroplasty?

1. Increased contact stress within the polyethylene


2. Increased risk of polyethylene delamination
3. Decreased mechanical forces transferred to the fixation surfaces
4. Decreased femoral rollback during flexion
5. Increased femoral rotation during flexion
PREFERRED RESPONSE: 4

DISCUSSION: Increasing conformity between the tibial polyethylene bearing and the femoral component has
desirable and undesirable consequences. Increased conformity increases the contact area and thereby reduces
contact stress within the polyethylene. This can lead to less risk of polyethylene wear, fracture, and delamination.
However, increasing conformity also limits the ability of the femur to roll back during flexion, and may transfer
increased shear stress to the fixation surfaces of the implants.

REFERENCES: D’Lima DD, Chen PC, Colwell CW Jr: Polyethylene contact stresses, articular congruity, and
knee alignment. Clin Orthop Relat Res 2001;392:232-238.
Pelicci PM, Tria AJ Jr, Garvin KL (eds): Orthopaedic Knowledge Update: Hip and Knee Reconstruction 2.
Rosemont, IL, American Academy of Orthopaedic Surgeons, 2000, pp 265-274.

Question 30
The anterior approach to total hip arthroplasty requires dissection between which of the following muscle planes?

1. Sartorius and gluteus maximus


2. Gluteus minimus and rectus femoris
3. Rectus femoris and sartorius
4. Tensor fascia lata and sartorius
5. Tensor fascia lata and rectus femoris
PREFERRED RESPONSE: 4
AL-Madena Copy
DISCUSSION: The anterior approach to the hip joint involves identifying the plane between the tensor fascia lata
and the sartorius muscles.

REFERENCES: Berger RA, Duwelius PJ: The two-incision minimally invasive total hip arthroplasty: Technique
and results. Orthop Clin North Am 2004;35:163-172.
Matta JM, Shahrdar C, Ferguson T: Single-incision anterior approach for total hip arthroplasty on an orthopaedic
table. Clin Orthop Relat Res 2005;441:115-124.
AL-Madena Copy

23
28 • American Academy of Orthopaedic S urgeons

Figure 3Id Figure 31e

Question 31
A 41-year-old female postal worker reports a 9-month history of left groin and lateral hip pain. She denies pain in
the right hip. Her social history reveals that she smokes and drinks on average five alcoholic beverages per week.
Her body mass index (BMI) is 26. Radiographs are shown in Figures 31a through 31c and coronal and axial MRI
scans are shown in Figures 3 Id and 31e, respectively. What is the most important factor that will determine if her
right hip will become symptomatic?

1. Patient age
2. Gender of the patient
3. Patient’s BMI and activity level
4. Size of the lesion
5. Association with alcohol use

PREFERRED RESPONSE: 4

DISCUSSION: Several factors have been examined as potentially predictive of progression in the patient with an
asymptomatic osteonecrotic lesion of the femoral head. While the presence of ongoing risk factors, lesion
location, lesion stage, age, gender, and BMI have all been suspected as important, the size of the lesion,
particularly when over one third of the size of the femoral head, is a significant risk factor for progression.
REFERENCES: Nam KW, Kim YL, Yoo JJ, et al: Fate of untreated asymptomatic osteonecrosis of the femoral
head. J Bone Joint Surg Am 2008;90:477-484.
Hungerford DS, Jones LC: Asymptomatic osteonecrosis: Should it be treated? Clin Orthop Relat Res
2004;429:124-130.

24
2010 Adult Reconstructive S urgery of the Hip and Knee Examination Answer Book • 29

Figure 32a Figure 32b Figure 32c Figure 32d

Question 32
A 66-year-old woman reports pain in both of her knees that has been recalcitrant to nonsurgical management.
Radiographs, including an AP of both knees, lateral and patellar views of the more symptomatic left knee and a
mechanical axis, are shown in Figures 32a through 32d. The lateral radiograph shows maximal knee extension.
When counseling the patient preoperatively regarding the risks of total knee arthroplasty, she should be educated
that she is at higher risk than the typical patient for which of the following complications?

1. Deep venous thrombosis


2. Peroneal nerve injury
3. Wound healing problems
4. Femoral component loosening
5. Quadriceps tendon rupture

PREFERRED RESPONSE: 2

DISCUSSION: The patient has a large valgus deformity and flexion contracture that places her at increased risk
for peroneal nerve injury. The peroneal nerve is tethered at the fibular head and correction of her valgus deformity
and flexion contracture at the time of total knee arthroplasty can stretch the nerve, leading to peroneal nerve palsy.

REFERENCES: Barrack RL, Booth RE Jr, Lonner JH, et al (eds): Orthopaedic Knowledge Update: Hip and Knee
Reconstruction 3. Rosemont, IL, American Academy of Orthopaedic Surgeons, 2006, pp 147155.
Asp JP, Rand JA: Peroneal nerve palsy after total knee arthroplasty. Clin Orthop Relat Res 1990;261:233- 237.
Schinsky MF, Macaulay W, Parks ML, et al: Nerve injury after primary total knee arthroplasty. J Arthroplasty
2001; 16:1048-1054.
AL-Madena Copy

Figure 33a Figure 33b

Question 33
A 22-year-old patient has had severe groin pain for many months and is unable to engage in any physical activity.
TheAL-Madena
AP radiograph
Copy of the pelvis shows minimal arthritis. The lateral radiograph of the hip is shown in Figure 33a.
An MR-arthrogram is shown in Figure 33b. What is the most appropriate treatment at this stage?

25
30 • American Academy of Orthopaedic S urgeons

1. Hip arthroscopy and labral debridement


2. Femoroacetabular osteoplasty and labral repair
3. Femoral osteotomy
4. Hemiarthroplasty
5. Total hip arthroplasty

PREFERRED RESPONSE: 2

DISCUSSION: The patient has femoroacetabular impingement. The prominence on the femoral neck has
resulted in a labral tear and detachment. An MR-arthrogram is the most appropriate modality for diagnosis of
a labral tear. The diagnosis of a labral tear per se is not an indication for surgical intervention because the
natural incidence of this condition is not known. Labral debridement without addressing the underlying
anatomic abnormality is likely to result in a suboptimal outcome. The most appropriate treatment, when
indicated, is shaving down of the femoral neck to remove the bony prominence and attachment of the labrum.
Femoral osteotomy has no role in the treatment of this condition. The patient has minimal arthritis; therefore,
arthroplasty is not indicated.

REFERENCES: Ganz R, Parvizi J, Beck M, et al: Femoroacetabular impingement: A cause for osteoarthritis of
the hip. Clin Orthop Relat Res 2003;417:112-120.
Espinosa N, Rothenfluh DA, Beck M, et al: Treatment of femoro-acetabular impingement: Preliminary results of
labral refixation. J Bone Joint Surg Am 2006;88:925-935.

AL-Madena Copy

26
2010 Adult Reconstructive S urgery of the Hip and Knee Examination Answer Book • 31

Question 34
Following total knee arthroplasty, a patient is noted to have asymmetrical absent pulses and poor capillary refill.
What is the next most appropriate step in management?

1. Observation of the limb for 4 hours to see if the arterial spasm resolves
2. Measurement of lower leg compartment pressures
3. Magnetic resonance angiogram
4. Emergent return to the operating room for wound exploration while the patient anesthesia is still under
5. Return to the operating room, obtain a vascular surgery consultation, and intraoperative
arteriogram perform an

PREFERRED RESPONSE: 5

DISCUSSION: An assessment of the location of the vascular compromise is necessary prior to surgical
exploration. Vascular repair will most likely require a separate surgical exposure. Vascular reperfusion may be
accomplished at the time of an arteriogram with the use of a stent in certain situations. Return to the operating
room with vascular surgical consultation and intraoperative arteriogram is appropriate.
An immediate postoperative compartment syndrome is unlikely. Magnetic resonance angiogram is not appropriate
because of the potential for a delay in diagnosis.

REFERENCE: Smith DE, McGraw RW, Taylor DC, et al: Arterial complications and total knee arthroplasty. J Am
Acad Orthop Surg 2001 ;9;253-257.

Question 35
With respect to the safety of allogeneic transfusions, which of the following infectious diseases is not tested for
during routine laboratory blood screening?

1. Human immunodeficiency virus (HIV)


2. Hepatitis C
3. Lyme disease
4. Syphilis

5. West Nile virus encephalitis PREFERRED RESPONSE: 3


DISCUSSION: Routine screening of donated blood usually includes screening for Hepatitis B, Hepatitis C, syphilis,
HIV-1 (responsible for AIDS), HIV-2, and West Nile virus. Several bloodbome diseases that are not routinely
screened for during blood donation include Lyme disease, malaria, babesiosis, and Chagas disease. Because routine
blood bank practices do not include screening for all potential bloodbome diseases, surgeons should be aware of the
possibility of disease transmission.

REFERENCE: Barrack RL, Booth RE Jr, Lonner JH, et al (eds): Orthopaedic Knowledge Update: Hip and Knee
Reconstruction 3. Rosemont, IL, American Academy of Orthopaedic Surgeons, 2006, pp 217-224.

AL-Madena Copy

27
32 • American Academy of Orthopaedic S urgeons

Question 36
A 57-year-old man undergoes total hip arthroplasty, has an uncomplicated early postoperative course, and a
normal neurovascular status is documented. However, on postoperative day two he develops a progressive foot
drop that increases over the next 24 hours. Postoperative repeat radiographs of the hip arthroplasty are
unrevealing. There is no suggestion of swelling of the thigh to suggest a subfascial wound hematoma. What is the
next most appropriate step?

1. MRI of the lumber spine


2. MRI of the knee joint
3. Electromyography and nerve conduction velocity studies of the sciatic nerve
4. Lumbar spine anterior/posterior and lateral radiographs
5. Venous Doppler of the lower extremity

PREFERRED RESPONSE: 1

DISCUSSION: Spinal stenosis is commonly found in patients with degenerative arthritis of the hip joint and
would be a strong possibility for causing a foot drop if the radiographs do not reveal a dislocation or other direct
mechanical cause for the sciatic nerve compromise.

REFERENCES: Dehart MM, Riley LH Jr: Nerve injuries in total hip arthroplasty. J Am Acad Orthop Surg
1999;7:101-111.
Pritchett JW: Lumbar decompression to treat foot drop after hip arthroplasty. Clin Orthop Rel Res 1994;303:173-
177.

Question 37
A 68-year-old woman undergoes an uncomplicated primary total knee arthroplasty with cement. Twelve days
later, the proximal wound appears healed and pain is decreased, but the distal wound continues to drain with slight
surrounding redness. What is the most appropriate management?

1. Prescribe an antibiotic specific for hospital-acquired bacteria.


2. Recommend antibiotic ointment and peroxide treatment.
3. Place the knee in an immobilizer and observe.
4. Place sutures to close up the wound in the clinic.
5. Irrigate and debride the knee in the operating room.
PREFERRED RESPONSE: 5

DISCUSSION: Persistent wound drainage after a total knee arthroplasty can lead to infection and other
complications. In this situation, antibiotics alone are not indicated, nor are topical wound treatment measures.
Prompt, aggressive treatment includes returning the patient to the operating room for irrigation and debridement,
with possible polyethylene exchange. Resection of components is not necessarily indicated at this early stage
because the arthroplasty can still be salvaged.
REFERENCES: Vince K, Chivas D, Droll KP: Wound complications after total knee arthroplasty. J Arthroplasty
2007;22:39-44.
Dennis DA: Wound complications in total knee arthroplasty. Orthopedics 1997;20:837-840.

AL-Madena Copy

28
2010 Adult Reconstructive S urgery of the Hip and Knee Examination Answer Book • 33

Question 38
A nondisplaced fracture of the proximal medial femoral neck proximal to the lesser trochanter is noted at the time
of insertion of a cementless tapered wedge-type femoral component in a total hip arthroplasty. Appropriate
perioperative management should include which of the following?

1. Cerclage cable placed proximal to the lesser trochanter with partial weight bearing for 6 weeks
postoperatively
2. No intraoperative or postoperative modifications are necessary
3. Non-weight-bearing for 6 weeks, retention of the femoral component, and no cerclage wire
4. Fracture exploration and repair with multiple cerclage cables, strut allograft and revision of the femoral
component with a long-stemmed implant
5. Revision with a cemented implant

PREFERRED RESPONSE: 1

DISCUSSION: The fracture should be explored in its entirety. If it remains in the intertrochanteric region, a single
cerclage cable passed above the lesser trochanter and tightened around the femoral component is appropriate. A
more distal or displaced fracture should be repaired with cerclage cables and consideration for revision of the
femoral component with a long-stemmed or cemented implant should be given.

REFERENCES: Sharkey PF, Hozack WJ, Booth RE, et al: Intraoperative femoral fractures in cementless total hip
arthroplasty. Orthop Rev 1992;21:337-342.
Sharkey PF, Wolf LR, Hume EL, et al: Insertional femoral fracture: A biomechanical study of femoral component
stability. Semin Arthroplasty 1990;1:91-94.

Figure 39a Figure 39b Figure 39c

Question 39
A 68-year-old woman reports pain and sensations of instability following a primary total knee arthroplasty
18 months ago. A preoperative radiograph is shown in Figure 39a and postoperative AP and patellar view
radiographs are shown in Figures 39b and 39c. A CT scan shows that the femoral component is internally rotated 8
degrees and the tibial component is internally rotated 4 degrees. Management should include which of the
following?

1. A structured physical therapy program


2. A custom patellar stabilizing brace
3. Modular polyethylene liner exchange to a thicker liner

AL-Madena Copy

29
34 • American Academy of Orthopaedic S urgeons

4. Revision of the femoral and tibial components


5. An open lateral retinacular release

PREFERRED RESPONSE: 4

DISCUSSION: The patient had a valgus knee preoperatively and in these patients, care must be taken to avoid
internal rotation of the femoral component that can lead to patellar instability. At the present time she has a
completely dislocated patella, evidence of coronal instability on her AP radiograph, and internally rotated femoral
and tibial components; therefore, management should consist of revision of both of her components to place them
in appropriate external rotation.

REFERENCES: Malo M, Vince KG: The unstable patella after total knee arthroplasty: Etiology, prevention, and
management. J Am Acad Orthop Surg 2003; 11:364-371.
Barrack RL, Booth RE Jr, Lonner JH, et al (eds): Orthopaedic Knowledge Update: Hip and Knee Reconstruction 3.
Rosemont, IL, American Academy of Orthopaedic Surgeons, 2006, pp 123-145.
Question 40
The term “paradoxical motion,” used to describe knee kinematics, is best described by which of the following
definitions?

1. The patella does not roll forward into the trochlear groove during knee extension.
2. The tibia rolls back on the femur during knee extension.
3. The tibiofemoral contact point moves anteriorly during knee flexion.
4. The posterior cruciate ligament rolls posteriorly with respect to the anterior cruciate ligament during
knee extension.
5. The femur rolls back on the tibia during knee flexion.
PREFERRED RESPONSE: 3

DISCUSSION: The term “rollback” describes the posterior movement of the tibiofemoral contact point with knee
motion from extension to flexion. Therefore, with “paradoxical rollback” this contact point moves anteriorly.
“Paradoxical rollback” is a term used to connote the inability of the anterior cruciate- deficient, posterior cruciate-
retaining total knee prosthesis to create normal posterior femoral rollback with knee flexion.

REFERENCES: Dennis DA, Komistek RD, Mahfouz MR: In vivo fluoroscopic analysis of fixed-bearing total knee
replacements. Clin Orthop Relat Res 2003;410:114-130.
Incavo SJ, Mullins ER, Coughlin KM, et al: Tibiofemoral kinematic analysis after total knee arthroplasty.
J Arthroplasty 2004;19:906-910.

Question 41
Patients with hip dysplasia have a series of anatomic abnormalities that most commonly include which of the
following?

1. Shallow, medialized acetabulum that is deficient anteriorly and superiorly


2. Large contact area between the femoral head and acetabulum
3. Large femoral head with long femoral neck
4. Excessive femoral neck anteversion and a posterior greater trochanter

AL-Madena Copy

30
2010 Adult Reconstructive S urgery of the Hip and Knee Examination Answer Book • 35

5. Decreased neck-shaft angle


PREFERRED RESPONSE: 4

DISCUSSION: Patients with developmental dysplasia of the hip share a common pattern of anatomic abnormalities
including an acetabulum that is shallow, lateralized, anteverted, and deficient anteriorly and superiorly. On the
femoral side, the head is usually small, the neck is short and anteverted with a posteriorly placed trochanter, and the
femoral canal is small. The neck-shaft angle is typically increased. The contact area of the femoral head and
acetabulum is typically decreased.

REFERENCE: Sanchez-Sotelo J, Trousdale RT, Berry DJ, et al: Surgical treatment of developmental dysplasia of
the hip in adults: I. Nonarthroplasty options. J Am Acad Orthop Surg 2002; 10;321-333.

Figure 42

Question 42
An 82-year-old man who underwent a primary total knee arthroplasty 11 weeks ago is now seen following a fall
from a standing height. A radiograph is shown in Figure 42. Examination reveals a small abrasion of the skin
overlying the anterior aspect of the knee. He is able to actively extend the the knee but has a 10-degree extensor
lag. Initial management should include which of the following?

1. A 4-week period of immobilization followed by protected range of motion in a hinged knee brace
2. Use of a cylinder cast for 12 weeks
3. Open reduction and internal fixation of the fracture using a tension band technique 4 Revision of
the patellar component

5. Resection of the proximal bony fragment and soft-tissue repair PREFERRED RESPONSE: 1
DISCUSSION: The patient has a periprosthetic fracture of the patella but is able to actively extend his knee.
Despite the wide displacement of the fracture fragments, nonsurgical management is recommended given the high
risk of complications and problems when open treatment of these fractures is undertaken.

REFERENCES: Ortiguera CJ, Berry DJ: Patellar fracture after total knee arthroplasty. J Bone Joint Surg Am
2002;84:532-540.
Parvizi J, Kim KI, Oliashirazi A, et al: Periprosthetic patella fractures. Clin Orthop Relat Res
2006;446:161-166.

AL-Madena Copy

31
36 • American Academy of Orthopaedic S urgeons

Figure 43a Figure 43b

Question 43
An active 72-year-old man underwent a right hybrid total hip arthroplasty for osteoarthritis 4 years ago. His hip
has functioned well until approximately 8 months ago. He now reports activity-related proximal thigh pain and
groin pain. A current radiograph is shown in Figure 43a. A radiograph obtained prior to the onset of symptoms is
shown in Figure 43b. What is the most likely cause of his symptoms?

1. Polyethylene wear with polyethylene-induced synovitis


2. Psoas tendon irritation secondary to psoas impingement over the anterior inferior edge of the
acetabular component
3. Loose femoral component
4. Osteolysis secondary to polyethylene wear particles
5. Modulus mismatch of the stem and femur

PREFERRED RESPONSE: 3

DISCUSSION: The radiograph shows a loose femoral component. The implant has debonded showing a
radiolucent line at the lateral shoulder (zone 1), and also increased radiolucency at the bone cement interface
medially. The patient’s symptoms are mechanical in nature with activity-related pain. Symptoms are not
characteristic of psoas tendon irritation. Although osteolysis is present, in the absence of fracture it is generally
asymptomatic. Additionally, in this patient the osteolysis involving the proximal femur is much more likely to be
secondary to cement particulate debris and less likely related to polyethylene wear particles.

REFERENCES: Jasty M, Maloney WJ, Bragdon CR, et al: The initiation of failure in cemented femoral
components of hip arthroplasties. J Bone Joint Surg Br 1991;73:551-558.
Kwong LM, Jasty M, Mulroy RD, et al: The histology of the radiolucent line. J Bone Joint Surg Br
1992;74:67-73.
Verdonschot N, Tanck E, Huiskes R: Effects of prosthesis surface roughness on the failure process of
cemented hip implants after stem-cement debonding. J Biomed Mater Res 1998;42:554-559.

AL-Madena Copy

32
2010 Adult Reconstructive S urgery of the Hip and Knee Examination Answer Book • 37

Figure 44

Question 44
A 64-year-old woman who underwent a successful total hip arthroplasty (THA) 2 years ago now reports a painful
hip. A radiograph is shown in Figure 44. Laboratory evaluation includes an erythrocyte sedimentation rate (ESR) of
65 mm/h (0-30 mm/h) and a C-reactive protein of 5.4 mg/L (< 0.8 mg/L). What is the next step in management?

1. Ultrasound examination
2. Technetium bone scan
3. Indium-labeled WBC scan
4. Hip joint aspiration
5. Two-stage revision surgery

PREFERRED RESPONSE: 4

DISCUSSION: Hip joint aspiration, with analysis of the cell count, differential, and culture greatly aids in the
diagnosis of infection in THA. Aspiration is indicated in this patient because both the ESR and C-reactive
protein are elevated, and the radiograph shows well-fixed components without evidence of loosening.
Technetium and Indium-labeled WBC scans have less specificity and sensitivity respectively, and may not be
necessary if the diagnosis is confirmed by joint aspiration. Two-stage revision surgery may be the treatment of
choice if infection is confirmed, but it is not the next step in this patient.

REFERENCE: Barrack RL, Booth RE Jr, Lonner JH, et al (eds): Orthopaedic Knowledge Update: Hip and Knee
Reconstruction 3. Rosemont, IL American Academy of Orthopedic Surgeons, 2006, pp 475-503.
Question 45
The American Academy of Orthopaedic Surgeons thrombophlebitis prophylaxis guidelines for patients undergoing
total joint arthroplasty include which of the following?

1. General as opposed to regional anesthesia


2. Preoperative assessment for risk of thromboembolic disease as well as bleeding
3. Routine use of inferior vena cava filters
4. Warfarin with a goal International Normalized Ratio (INR) of 2-3 for patients with standard risk of
bleeding and thrombophlebitis
5. Thromboembolic compression stockings (TEDs)
PREFERRED RESPONSE: 2

DISCUSSION: The 2007 AAOS guidelines for thrombophlebitis prophylaxis for patients undergoing total hip and
knee arthroplasty includes preoperative risk assesment for deep venous thrombosis, pulmonary embolism, and
AL-Madena Copy

33
38 • American Academy of Orthopaedic S urgeons

bleeding. Regional anesthesia when appropriate is suggested. Inferior vena cava filters may be appropriate in
selected patients. When warfarin is used as a chemoprophylactic agent, the goal INR is less than or equal to 2 to
minimize the risk of bleeding. This is in contrast to the 2004 ACCP guidelines for warfarin with a goal INR of 2-3.
-

REFERENCE: American Academy of Orthopaedic Surgeons Guideline on the Prevention of Symptomatic


Pulmonary Embolism in Patients Undergoing Total Hip or Knee Arthroplasty. www.aaos.org/Research/
guidelines/PEguide.asp

Figure 46

Question 46
Figure 46 shows the radiograph of a 65-year-old man who reports restricted range of motion and pain with sitting
18 months after undergoing right side revision total hip arthroplasty. What is the most appropriate management?

L Intensive physiotherapy
2. Alendronate
3. Indomethacin
4. Radiotherapy
5. Excision

PREFERRED RESPONSE: 5

DISCUSSION: The presence of Brooker grade 1 or 2 heterotopic ossification (HO) does not influence the
outcome of total hip arthroplasty, whereas restricted range of motion and pain may occur in patients with more
severe grade 3 or 4 HO. Treatment may be nonsurgical or surgical. Nonsurgical management includes intensive
physiotherapy during the maturation phase of the disease in an attempt to limit the final stiffness. There appears
to be no data regarding the effectiveness of this treatment. There is no role for NSAIDs or radiothe rapy as a
treatment for preexisting HO. Surgical treatment involves excision of the heterotopic bone and can be expected
to improve the functional outcome. Bisphosphonates have been used in the past, but their use has been
discontinued as they only postpone ossification until treatment is stopped.

REFERENCES: Board TN, Karva A, Board RE, et al: The prophylaxis and treatment of heterotopic
ossification following lower limb arthroplasty. J Bone Joint Surg Br 2007;89:434-440.
Harkess JW, Crockarell JR: Arthroplasty of the hip, in Canale ST, Beaty JH (eds): Campbell’s Operative
Orthopaedics, ed 11. Philadelphia, PA, Mosby Elsevier, 2008, vol 1, pp 314-483.
Question 47
What is the most common cause of early failure for patellofemoral arthroplasty?

AL-Madena Copy

34
2010 Adult Reconstructive S urgery of the Hip and Knee Examination Answer Book • 39

1. Progression of tibiofemoral arthritis


2. Loosening of the femoral trochlear component
3. Loosening of the patellar component
4. Patellar instability/maltracking
5. Rupture of the quadriceps tendon from trochlear component impingement secondary to
excessive anterior placement and flexion of the implant
PREFERRED RESPONSE: 4

DISCUSSION: In properly selected patients who have no or minimal tibiofemoral arthritis, the most common
cause of early failure is patellofemoral instability secondary to uncorrected patellar malalignment, soft-tissue
imbalance, or component malposition. Progression of tibiofemoral arthritis is also a leading cause of failure, but
occurs late in about 25% of patients. Loosening of components has occurred in less than 1% of knees with
cemented implants. A higher rate of loosening has been reported in cementless implants.

REFERENCES: Lonner JH: Patellofemoral arthroplasty. J Am Acad Orthop Surg 2007;15:495-506. Argenson JN,
Flecher X, Parratte S, et al: Patellofemoral arthroplasty: An update. Clin Orthop Relat Res 2005;440:50-53.

Figure 48

Question 48
An 88-year-old nursing home resident is seen in the emergency department after a fall. At the time of admission,
physical examination of the affected extremity reveals absent pulses and inadequate capillary refill. A radiograph
is seen in Figure 48. Appropriate management includes which of the following?

1. Emergent open reduction and internal fixation of the fracture


2. Emergent consultation with vascular surgery and a possible arteriogram
3. Revision of the femoral component followed by vascular surgical consultation
4. Open reduction and internal fixation of the fracture with intraoperative vascular consultation
5. Release traction until surgical management is performed

PREFERRED RESPONSE: 2

AL-Madena Copy

35
40 • American Academy of Orthopaedic S urgeons

DISCUSSION: The patient has a Vancouver type B3 periprosthetic femur fracture (fracture around a total hip
prosthesis with a loose stem and poor proximal bone stock). This type of periprosthetic fracture necessitates
revision of the femoral component as well as fracture fixation. This patient has diminished blood flow in the
extremity that necessitates emergent vascular surgical evaluation.

REFERENCES: Brady OH, Garbuz DS, Masri BA, et al: Classification of the hip. Orthop Clin North Am
1999;30;215-220,
Klein GR, Parvizi J, Rapuri V, et al: Proximal femoral replacement for the treatment of periprosthetic fractures. J
Bone Joint Surg Am 2005;87:1777-1781.
Question 49
An otherwise healthy 58-year-old woman reports hip pain after undergoing total hip arthroplasty 8 months ago. She
gives a history of prolonged wound drainage after surgery treated with antibiotics.
Hip aspiration is positive for methicillin-resistant coagulase-negative staphylococcus. Appropriate management at
this point includes which of the following?

1. Arthroscopic irrigation followed by appropriate antibiotic treatment


2. Treatment with 6 weeks of IV vancomycin and oral rifampin
3. Treatment with 6 weeks of IV vancomycin and oral rifampin, followed by indefinite oral
antibiotic suppression
4. Open debridement with exchange of the polyethylene insert, followed by appropriate antibiotic
treatment
5. Open debridement with removal of the implants and insertion of an antibiotic spacer

PREFERRED RESPONSE: 5

DISCUSSION: The patient has a late chronic periprosthetic infection. The history points to a missed postoperative
infection where the patient was treated with antibiotics for prolonged wound drainage. A low virulent organism,
such as coagulase-negative staphylococcus, is often the pathogen. In an otherwise healthy patient with a reasonable
life expectancy, the goal of treatment should be cure of the infection. Treatment for late chronic infection requires
removal of the implants. Treatment with antibiotics alone, or debridement without removal of the implants is
generally unsuccessful. Controversy exists regarding one- stage treatment with immediate reimplantation versus
two-stage treatment with delayed reimplantation. Generally, a higher cure rate is achieved with a two-stage
reimplantation (approximately 90% to 93%) than with one-stage reimplantation (75% to 80%).

REFERENCES: Hanssen AD, Spangehl MJ: Treatment of the infected hip replacement. Clin Orthop Relat Res
2004;420:63-71.
Isiklar ZU, Demirors H, Akpinar S, et al: Two-stage treatment of chronic staphylococcal orthopaedic
implant-related infections using vancomycin impregnated PMMA spacer and rifampin containing antibiotic
protocol. Bull Hosp Jt Dis 1999;58:79-85.

Question 50
Patients with ankylosing spondylitis undergoing total knee arthroplasty are likely to experience which of the
following complications?

1. Infection

AL-Madena Copy

36
2010 Adult Reconstructive S urgery of the Hip and Knee Examination Answer Book • 41

2. Instability
3. Heterotopic ossification
4. Periprosthetic fracture
5. Patellar loosening

PREFERRED RESPONSE: 3
DISCUSSION: Patients with ankylosing spondylitis (AS) are likely to have achieved significant pain relief
and improvement in function following total knee arthroplasty (TKA). These patients, however, are also likely
to experience a higher incidence of complications, particularly stiffness and heterotopic ossification (HO). In
one series, the incidence of HO following TKA in patients with AS was 20%.

REFERENCES: Parvizi J, Duffy GP, Trousdale RT: Total knee arthroplasty in patients with ankylosing
spondylitis. J Bone Joint Surg Am 2001;83:1312-1316.
Fintersbush A, Amir D, Vatashki E, et al: Joint surgery in severe ankylosing spondylitis. Acta Orthop Scand
1988;59:491-496.

Question 51
Posterior cruciate-retaining total knee arthroplasty has been studied by numerous methods including kinematic
video fluoroscopy. Which of the following is the best description of typical kinematic behavior?

1. Unpredictable anterior femoral condylar translation from full extension to 90 degrees of flexion
2. Predictable femoral rollback from full extension to 90 degrees of flexion
3. Medial femoral condyle pivot with minimal medial femoral condyle translation and lateral femoral
condyle posterior translation in flexion
4. Lateral femoral condyle pivot with minimal medial femoral condyle translation
5. Anterior tibial contact of both the medial and lateral femoral condyles in full extension
PREFERRED RESPONSE: 1

DISCUSSION: In vivo fluoroscopic studies have enlightened orthopaedic surgeons as to the actual kinematic
performance of the total knee implants. Importantly, many technical factors combine to result in abnormal
kinematics after total knee arthroplasty. The typical kinematics of the normal knee are negated by osteoarthritis and
the various changes created by the surgeon’s technique. Dennis and associates observed that anterior femoral
condylar translation during deep knee flexion was most commonly observed in posterior cruciate-retaining knees.

REFERENCES: Dennis DA, Komistek RD, Mahfouz MR, et al: Multicenter determination of in vivo
kinematics after total knee arthroplasty. Clin Orthop Relat Res 2003;416:37-57.
Freeman MA, Pinskerova V: The movement of the normal tibio-femoral joint. J Biomech 2005-38'197- 208.

AL-Madena Copy

37
42 • American Academy of Orthopaedic S urgeons

Figure 52
Question 52
A 52-year-old woman has a 60-degree extensor lag following a right total knee arthroplasty performed 16
months ago. Since the time of her primary total knee arthroplasty she has undergone primary repair of a patellar
tendon rupture that occurred after a fall 8 months ago. A lateral radiograph of the knee is shown in Figure 52. A
CT scan obtained to determine component rotation showed that the femoral component is internally rotated 9
degrees and the tibial component is internally rotated 12 degrees. Appropriate management at this time should
include

1. a structured physical therapy program to increase quadriceps muscle strength.


2. a hinged knee brace locked in extension while ambulating.
3. exchange of the modular polyethylene spacer to a thicker insert and reconstruction of the
patellar tendon using hamstring augmentation.
4. exchange of the modular polyethylene spacer to a thicker insert and reconstruction of the
patellar tendon using an extensor mechanism allograft tensioned tightly in full extension.
5. revision of the tibial and femoral components and reconstruction of the patellar tendon using an
extensor mechanism allograft tensioned tightly in full extension.
PREFERRED RESPONSE: 5

DISCUSSION: A chronic patellar tendon rupture is a difficult complication to manage. Patients typically present
with both inability to extend their leg and instability of the extremity, oftentimes associated with multiple falls.
Attempts at secondary repair have been associated with high failure rates whereas the use of an extensor
mechanism allograft has been shown to more effectively restore active extension in a substantial percentage of
patients. Important aspects of the technique include fully tensioning the graft in full extension and
immobilization of the extremity for 6 to 8 weeks postoperatively to allow for graft healing. Nonsurgical
management will not result in an acceptable outcome for a young patient, and attempted secondary repair is
associated with a high rate of failure, even when augmented with local tissues. This patient has gross rotational
malalignment of the components and the surgeon faced with this problem should consider obtaining a CT scan to
determine component rotation preoperatively.

REFERENCES: Burnett RS, Berger RA, Paprosky WG, et al: Extensor mechanism allograft reconstruction after
total knee arthroplasty: A comparison of two techniques. J Bone Joint Surg Am 2004;86:2694-2699.
Nazarian DG, Booth RE: Extensor mechanism allografts in total knee arthroplasty. Clin Orthop Relat Res
1999;367-123-129. , „. H
AL-Madena Copy ^Lj-^M) <UjJlU

38
2010 Adult Reconstructive S urgery of the Hip and Knee Examination Answer Book • 43

Figure 53

Question 53
Figure 53 shows the radiograph of a 48-year-old man who has a left side periprosthetic femoral fracture around the
femoral stem of a previous revision hip arthroplasty. What is the most appropriate treatment?

1. Open reduction and internal fixation with a plate


2. Open reduction and internal fixation with a cable
3. Revision with a short stem and plate fixation
4. Revision with allograft prosthesis composite

5. Removal of the proximal femoral bone and replacement with a cemented segmental prosthesis

PREFERRED RESPONSE: 4
DISCUSSION: In type B3 fractures, the proximal femur is so deficient that it cannot be treated with open
reduction and internal fixation or support a new femoral component. In younger patients, the femur can be
reconstructed with allograft prosthesis composite to restore bone stock. Removal of the distal stem with trephines
would compromise fixation with cement. Elderly and low-demand patients can be treated more simply with a
cemented segmental replacement prosthesis, such as that used for tumor reconstruction.

REFERENCES: Parvizi J, Tarity TD, Slenker N, et al: Proximal femoral replacement in patients with non-
neoplastic conditions. J Bone Joint Surg Am 2007;89:1036-1043.
Harkess JW, Crockarell JR: Arthroplasty of the hip, in Canale ST, Beaty JH (eds): Campbell’s Operative
Orthopaedics, ed 11. Philadelphia, PA, Mosby Elsevier, 2008, vol 1, pp 314-483.
Lee SR, Bostrom MP: Periprosthetic fractures of the femur after total hip arthroplasty. Instr Course Lect
2004;53:111-118.

Figure 54

Question 54

AL-Madena Copy

39
44 • American Academy of Orthopaedic S urgeons

A 66-year-old woman who underwent a right total knee arthroplasty 13 years ago now has pain, knee effusion,
squeaking, and grinding in the operated knee. The patellar view radiograph is shown in Figure 54. What is most
likely to be encountered during revision arthroplasty?

1. Loose femoral and/or tibial component


2. Fractured patella
3. Extensive metallosis
4. Patellar clunk
5. Ruptured extensor mechanism

PREFERRED RESPONSE: 3

DISCUSSION: Wear of the polyethylene insert of a metal-backed patellar or tibial component is a well-
recognized complication following total knee arthroplasty. If the polyethylene insert completely wears out, the
underlying metal-backed base plate will be in direct contact with the femoral component, resulting in scratching
of the components and release of metal debris. The released metal debris results in black “tattooing” of the
tissues or so-called metallosis. In addition, reactive synovitis and osteolysis occur secondary to a host response to
both polyethylene debris and metal particles. Patients may report knee pain, swelling, squeaking, and an unstable
feeling during walking and standing. Patellar clunk syndrome is characterized by the development of a fibrous
nodule on the deep aspect of the quadriceps tendon that falls into the intercondylar notch of the femoral
component during knee flexion.

REFERENCES: Chang FY, Tseng KF, Chen WM, et al: Metal-backed patellar component failure in total knee
arthroplasty presenting as a giant calf mass. J Arthroplasty 2003;18:227-230.
Leopold SS, Berger RA, Patterson L, et al: Serum titanium level for diagnosis of a failed, metal-backed patellar
component. J Arthroplasty 2000;15:938-943.
Patel J, Ries MD, Bozic KJ: Extensor mechanism complications after total knee arthroplasty. Instr Course Lect
2008;57:283-294.

Figure 55a Figure 55b


Question 55
A 56-year-old man who underwent a left total hip arthroplasty 8 years ago is seen following a fall from a standing
height. A radiograph obtained at 2 years postoperatively is shown in Figure 55a and a current radiograph obtained
in the emergency department is shown in Figure 55b. On further questioning, he reports pain in this thigh for the
past 3 years that has been increasing in intensity. Appropriate management at this time includes which of the
following?

AL-Madena Copy

40
2010 Adult Reconstructive S urgery of the Hip and Knee Examination Answer Book • 45

1. Nonsurgical management with the use of a cast-brace


2. Nonsurgical management with skeletal traction
3. Open treatment with a locked plate with or without strut allograft
4. Revision of the femoral component to a cemented femoral component that bypasses the
fracture site by two cortical diameters
5. Revision of the femoral component to a cementless femoral component that bypasses the fracture site
by at least two cortical diameters

PREFERRED RESPONSE: 5

DISCUSSION: The patient has a Vancouver type B2 periprosthetic femoral fracture with a loose, cementless
femoral component. Radiographs show subsidence of the femoral component and the patient reports pain in the
thigh for several years prior to the fall. When the femoral component is loose, revision is mandated to treat both
the loose component and the fracture. Nonsurgical management is associated with a high risk of medical
complications related to extended recumbency as well as a high rate of malunion and nonunion. Cementless
femoral component revision has been shown to fare better than cemented femoral components, particularly for
treating periprosthetic fractures of the femur.

REFERENCES: Masri BA, Meek RM, Duncan CP: Periprosthetic fractures evaluation and treatment.
Clin Orthop Relat Res 2004;420:80-95.
Springer BD, Berry DJ, Lewallen DB: Treatment of periprosthetic femoral fractures following total hip
arthroplasty with femoral component revision. J Bone Joint Surg Am 2003;85:2156-2162.
Question 56
A 62-year-old woman who underwent a primary total knee arthroplasty under a combined spinal-epidural
anesthetic presents 5 hours postoperatively with severe pain in the extremity that is unresponsive to narcotic pain
medication. A tourniquet was used during the procedure. On examination, the patient is unable to dorsiflex or
plantar flex the foot and the pulses appear to be asymmetric. What is the next most appropriate step in
management?

1. Discontinuation of the epidural and serial neurologic exams


2. Loosening of the surgical bandages and elevation of the extremity
3. MRI of the spine to evaluate for an epidural hematoma
4. Return to the operating room for angiography and vascular bypass
5. Return to the operating room for angiography, vascular bypass, and four-compartment fasciotomy

PREFERRED RESPONSE: 5

DISCUSSION: The patient has clinical evidence of vascular injury and compartment syndrome. An emergent
vascular consult and a return to the operating room for an angiogram and revascularization are needed. In these
situations, thrombectomy alone is often not sufficient and a bypass is typically required. A four-compartment
fasciotomy should be done following revascularization of the extremity.

REFERENCES: Calligaro KD, Dougherty MJ, Ryan S, et al: Acute arterial complications associated with total
hip and knee arthroplasty. J Vase Surg 2003;38:1170-1177.
Smith DE, McGraw RW, Taylor DC, et al: Arterial complications and total knee arthroplasty. J Am Acad Orthop
Surg 2001;9:253-257.
Ninomiya JT, Dean JC, Goldberg VM: Injury to the popliteal artery and its anatomic location in total knee

AL-Madena Copy

41
46 • American Academy of Orthopaedic S urgeons

arthroplasty. J Arthroplasty 1999;14:803-809.

Question 57
What mechanical properties are observed in polyethylene used for total knee arthroplasty after the material
undergoes oxidation?

1. Increased elastic modulus and decreased strength


2. Increased elastic modulus and increased ductility
3. Decreased elastic modulus and decreased strength
4. Decreased strength and increased ductility
5. Decreased elastic modulus and decreased ductility

PREFERRED RESPONSE: 1

DISCUSSION: When polyethylene undergoes oxidation, the material undergoes a decrease in strength and
ductility, and an increase in the elastic modulus. This makes the material more brittle, and leaves it vulnerable to
delamination, fracture, and pitting.
REFERENCE: Barrack RL, Booth RE Jr, Lonner JH, et al (eds): Orthopaedic Knowledge Update: Hip and Knee
Reconstruction 3. Rosemont, IL, American Academy of Orthopaedic Surgeons, 2006, pp 163-176.

Question 58
What is the most common cause for late revision (> 2 years post op) total knee arthroplasty?

i. Infection
2. Polyethylene wear
3. Instability
4. Patellar complications
5. Malalignment

PREFERRED RESPONSE: 2

DISCUSSION: There are multiple causes for failure of total knee arthroplasty, and more than one may exist at
the same time. Sharkey and associates reviewed a series of revision total knee arthroplasties, and found that
polyethylene failure was the most common cause of failure followed closely by component loosening. The
most common cause of early failure (< 2 years post op) was infection. Instability and malalignment are both
complications of surgical technique, and if these categories are combined, they would be the most common
cause of all total knee failures.

REFERENCE: Sharkey PF, Hozack WJ, Rothman RH, et al: Insall Award paper: Why are total knee
arthroplasties failing today? Clin Orthop Relat Res 2002;404:7-13.

AL-Madena Copy

42
2010 Adult Reconstructive S urgery of the Hip and Knee Examination Answer Book • 47

Figure 59a Figure 59b

Question 59
Figures 59a and 59b show the radiographs of a 63-year-old woman who is seen in the emergency department after
a minor twisting episode. History reveals that she underwent a successful mobile-bearing total knee arthroplasty
for severe lateral compartment arthritis and valgus deformity 4 months ago. What is the most likely cause of this
problem?

1. Flexion instability
2. Posterior cruciate ligament rupture
3. Lateral collateral ligament rupture
4. Chronic patellar instability
5. Prosthetic rotational malalignment

PREFERRED RESPONSE: 1

DISCUSSION: All of the instabilities listed may cause the above-noted radiographic appearance of a dislocated
mobile-bearing total knee arthroplasty; however, the most likely scenario with reasonable placement of the
implants and a lack of traumatic history, such as a fall, is flexion instability. Surgeons who consider the use of
mobile-bearing implants must recognize that flexion and extension gap balancing is important to avoid insert
subluxation.

REFERENCES: Thompson NW, Wilson DS, Cran GW, et al: Dislocation of the rotating platform after low
contact stress total knee arthroplasty. Clin Orthop Relat Res 2004;425:207-211.
Stiehl JB: Clinical results and complications in mobile-bearing total knee arthroplasty. Instr Course Lect
2005;54:233-240.
Question 60

AL-Madena Copy

43
48 • American Academy of Orthopaedic S urgeons

Which of the following abnormalities has been observed in a higher than expected frequency in patients with metal-
on-metal hip bearings?

1. Renal cell carcinoma


2. Leukocyte chromosomal aberrations
3. Carcinomas of the gastrointestinal tract
4. Soft-tissue sarcomas
5. Thyroid carcinoma
PREFERRED RESPONSE: 2

DISCUSSION: Metal-on-metal hip bearings have been associated with very low rates of wear and are commonly
used in North America and Europe. Patients with these bearings have higher levels of metal ions (particularly
cobalt and chromium) in the bloodstream than patients with bearings made of other materials. Although many
researchers have been concerned that these ions may predispose to cancer, there has been no evidence that patients
with metal-on-metal bearings are developing sarcomas or carcinomas with higher frequency than the general
population. However, there has been mixed data as to whether hematopoietic malignancies are slightly more
prevalent in patients with metal-on-metal bearings. Two recent reports have found chromosomal abnormalities,
such as translocations and aneuploidy, to be increased in patients with metal-on-metal hip bearings. The clinical
consequences of these changes, if any, remain unknown.

REFERENCES: Dunstan E, Ladon D, Whittingham-Jones P, et al: Chromosomal aberrations in the peripheral


blood of patients with metal-on-metal hip bearings. J Bone Joint Surg Am 2008;90:517-522. Ladon D, Doherty A,
Newson R, et al: Changes in metal levels and chromosome aberrations in the peripheral blood of patients after
metal-on-metal hip arthroplasty. J Arthroplasty 2004:19:78-83.
Visuri T, Pukkala E, Paavolainen P, et al: Cancer risk after metal on metal and polyethylene on metal total hip
arthroplasty. Clin Orthop Relat Res 1996:329:S280-S289.
Question 61
A 73-year-old woman with a history of type II diabetes mellitus undergoes a total hip arthroplasty for
osteoarthritis. She continues to have serosanguinous wound drainage from the midportion of the incision 12 days
after surgery. What is the most appropriate treatment at this time?

1. Return to the operating room for debridement and irrigation with removal of all implants and
immediate reimplantation
2. Return to the operating room for open debridement and irrigation, exchange of the polyethylene
insert, followed by appropriate antibiotics based on intraoperative culture results
3. Hip spica pressure dressing and a 2-week course of oral antibiotics
4. Hip aspiration for culture, followed by a 2-week course of appropriate IV antibiotics based on

AL-Madena Copy

44
2010 Adult Reconstructive S urgery of the Hip and Knee Examination Answer Book • 49

culture results
5. Hip aspiration for culture, followed by a 6-week course of appropriate IV antibiotics based on
culture results

PREFERRED RESPONSE: 2

DISCUSSION: This patient has an acute postoperative deep infection, with prolonged drainage at 12 days
postoperatively. Even without other signs or symptoms of infection, the prolonged drainage this far out after
surgery is concerning for an acute postoperative infection. The most appropriate treatment at this point is open
debridement with retention of the implants. Modular parts, if present, are exchanged. Oral or IV antibiotics alone
are generally inadequate to treat the infection, and this form of management alone would result in a missed
opportunity to potentially cure the infection. Removal of all implants at this point is not required because early
aggressive debridement within the first 2 weeks after surgery offers a reasonable outcome. After 4 to 6 weeks of
symptoms, results of debridement are less favorable and removal of the implants is usually required.

REFERENCES: Crockarell JR, Hanssen AD, Osmon DR, et al: Treatment of infection with debridement and
retention of the components following hip arthroplasty. J Bone Joint Surg Am 1998;80:1306-1313. Hanssen AD,
Osmon DR: Assessment of patient selection criteria for treatment of the infected hip arthroplasty. Clin Orthop
Relat Res 2000;381:91 -100.

Figure 62

Question 62
A 63-year-old woman reports pain in her groin, particularly when rising from a chair and when taking her first
steps out of bed in the morning. History reveals that she underwent a left primary total hip arthroplasty 19 years
ago. An AP radiograph is shown in Figure 62 and revision surgery is planned. What is the most reliable method
for reconstruction of the acetabulum?

1. Use of a cemented, all polyethylene acetabular component


2. Use of a cementless, porous-coated acetabular component with adjunctive screw fixation
3. Use of an antiprotrusio cage
4. Use of a bilobed, cementless acetabular component
5. Use of a bipolar head placed directly into the acetabulum

PREFERRED RESPONSE: 2

AL-Madena Copy

45
50 • American Academy of Orthopaedic S urgeons

DISCUSSION: The patient has severe wear and loosening of her cemented, all polyethylene acetabular component
with a Paprosky type 2 acetabular defect. Optimal long-term results have been achieved with the use of a
cementless, porous-coated acetabular component with adjunctive screw fixation. Cemented, all polyethylene
components have been shown to have a high rate of failure when used in the revision setting. Bilobed components
and antiprotrusio cages are unnecessary for this straightforward defect and have had mixed results reported in the
literature. The use of a bipolar head placed directly into the acetabulum is associated with high rates of persistent
pain and progressive acetabular erosion.

REFERENCES: Barrack RL, Booth RE Jr, Lonner JH, et al (eds): Orthopaedic Knowledge Update: Hip and Knee
Reconstruction 3. Rosemont, IL, American Academy of Orthopaedic Surgeons, 2006, pp 457474.
Della Valle CJ, Schuaipaj T, Berger RA, et al: Revision of the acetabular component without cement after total hip
arthroplasty: A concise follow-up, at fifteen to nineteen years, of a previous report. J Bone Joint Surg Am
2005;87:1795-1800.
Templeton JE, Callaghan JJ, Goetz DD, et al: Revision of a cemented acetabular component to a
cementless acetabular component: A ten to fourteen-year follow-up study. J Bone Joint Surg Am
2001;83:1706-1711.
Question 63
A 72-year-old man undergoes an uncomplicated cementless total hip arthroplasty for advanced osteoarthritis. At his
6-week postoperative follow-up, he has minimal pain and is progressing well with his mobility. Radiographs show
early formation of Brooker grade III heterotopic bone around his hip. What is the best treatment of the heterotopic
bone at this time?

1. Observation, repeat radiographs, and reexamination in 6 weeks


2. A 14-day course of indomethacin
3. A 4-week course of indomethacin
4. Plan for a return to the operating room at 10 weeks for excision of the heterotopic bone
5. Arrange urgently for 800 centigrey of radiation to the soft tissues and areas of heterotopic bone around
the hip, with shielding of the implants

PREFERRED RESPONSE: 1

DISCUSSION: The development of heterotopic bone occurs early after hip arthroplasty. The process begins within
days after surgery; therefore, prophylactic treatment must be in the early postoperative period (preoperative radiation
given within 24 hours of surgery, or postoperative radiation given within 72 hours of surgery, or nonsteroidal anti-
inflammatory drugs (NSAIDs) given postoperatively for 7 to 21 days - longer duration has not been shown to be of
any additional benefit). At 6 weeks, prophylactic treatment with NSAIDs or radiation is no longer effective. Surgery
at 10 weeks would be premature because the patient is currently asymptomatic with regards to the heterotopic bone,
and surgery prior to full maturation of the bone may increase the risk for more abundant recurrence of bone.

REFERENCES: Balboni TA, Gobezie R, Mamon HJ: Heterotopic ossification: Pathophysiology, clinical features,
and the role of radiotherapy for prophylaxis. Int J Radiat Oncol Biol Phys 2006;65:1289-1299. Fransen M, Neal B:
Non-steroidal anti-inflammatory drugs for preventing heterotopic bone formation after hip arthroplasty. Cochrane
Database Syst Rev 2004;3:CD001160.
Neal BC, Rodgers A, Clark T, et al: A systematic survey of 13 randomized trials of non-steroidal antiinflammatory

AL-Madena Copy

46
2010 Adult Reconstructive S urgery of the Hip and Knee Examination Answer Book • 51

drugs for the prevention of heterotopic bone formation after major hip surgery. Acta Orthop Scand 2000;71:122-128.

Figure 64
Question 64
Figure 64 shows the radiograph of a 61-year-old man with ankylosing spondylitis. He is scheduled to
undergo left total hip arthroplasty. Which of the following perioperative interventions should be
considered?

1. Intraoperative sciatic nerve monitoring


2. Preoperative radiation therapy with 700 centigray
3. Postoperative radiation therapy with 1,500 centigray
4. Postoperative continuous passive motion
5. Soft cervical collar during postoperative physical therapy

PREFERRED RESPONSE: 2

DISCUSSION: Patients with ankylosing spondylitis are at high risk for heterotopic ossification after total hip
arthroplasty, and perioperative prophylaxis with nonsteroidal anti-inflammatory drugs or radiation therapy has
been effective at reducing the risk. Radiation prophylaxis given within 24 hours preoperatively has been shown
to be as effective as radiation given postoperatively. Doses of 700 to 800 centigray have been shown to be as
effective as higher doses with less potential risk. Continuous passive motion has not been shown to improve the
postoperative flexibility of these patients. Although these patients may have stiff spines and may be at risk for
neurologic injury in the event of spinal fracture, a soft cervical collar is not expected to alter the risk should a
catastrophic spinal injury occur. Since there has been no shortening of the limb, the sciatic nerve should not be at
significantly higher risk for neurapraxia than with a more routine hip arthroplasty.

REFERENCES: Pellegrini VD Jr, Gregoritch SJ: Preoperative irradiation for prevention of heterotopic
ossification following total hip arthroplasty. J Bone Joint Surg Am 1996;8:870-881.
Pellegrini VD Jr, Konski AA, Gastel JA, et al: Prevention of heterotopic ossification with irradiation after total
hip arthroplasty: Radiation therapy with a single dose of eight hundred centigray administered to a limited field. J
Bone Joint Surg Am 1992;74:186-200.
Question 65
Which of the following statements best describes the process of articular cartilage degeneration in osteoarthritis?

1. In the second stage there is decreased catabolic activity with less matrix breakdown.
2. In the second stage there is less chondrocyte proliferation and decreased matrix production.
3. Matrix degradation includes increased proteoglycan production, more proteoglycan

AL-Madena Copy

47
52 • American Academy of Orthopaedic S urgeons

production, and longer glycosaminoglycan chains.


4. Cartilage degeneration may be initiated by inflammation, overload, or decreased matrix production.
5. Chondrocyte repair responses improve with aging.

PREFERRED RESPONSE: 4

DISCUSSION: Inflammation, overload, or decreased matrix production may lead to cartilage degeneration.
During the second stage of articular cartilage degeneration with osteoarthritis, there is increased chondrocyte
activity with proliferation and increased production of extracellular matrix. At the same time, there is an increase
in catabolic activity with removal of damaged matrix to facilitate matrix remodeling. Chondrocyte repair response
decreases with aging. Matrix degradation includes decreased proteoglycan production, less aggregation, and
shorter glycosaminoglycan chains.

REFERENCE: Buckwalter JA, Mankin HJ, Grodzinsky AJ: Articular cartilage and osteoarthritis. Instr Course
Lect 2005;54:465-480.

Question 66
An active 38-year-old male carpenter reports activity-related medial knee pain. Arthroscopy performed 3 years
ago revealed a torn medial meniscus that was debrided and mild condylar changes of the medial femoral condyle
and medial tibial plateau. Current standing radiographs reveal Ahlback stage II changes with mild medial femoral
joint space narrowing and a 5-degree varus deformity. What is the best treatment option?

1. High tibial osteotomy


2. Arthroscopic debridement and condylar “microfracture”
3. Osteoarticular transplant to the medial femoral condyle
4. Unicondylar arthroplasty
5. Total knee arthroplasty

PREFERRED RESPONSE: 1

DISCUSSION: The choice of high tibial osteotomy is appropriate to correct a deformity that has a very high risk
of leading to progressive arthritis. It should be more important than either the arthroscopic method or the
osteoarticular transplantation, though one may also consider these options. Prosthetic choices are limited for a
young active male in favor of less “end stage” options.
REFERENCES: Sharma L, Song J, Felson DT, et al: The role of knee alignment in disease progression and
function decline in knee osteoarthritis. JAMA 2001;286:188-195.
Wright JM, Crockett HC, Slawski DP, et al: High tibial osteotomy. J Am Acad Orthop Surg 2005-13 279289.

Question 67
A patient with a severe nickel allergy and degenerative joint disease of the hip would be best served by which of
the following prosthetic options?

AL-Madena Copy

48
2010 Adult Reconstructive S urgery of the Hip and Knee Examination Answer Book • 53

1. Cemented titanium stem, ceramic (alumina) head, and press-fit titanium cup
2. Cemented cobalt-chrome stem, ceramic (alumina) head, and press-fit cobalt-chrome cup
3. Press-fit titanium stem, cobalt-chrome head, and press-fit titanium cup
4. Press-fit titanium stem, titanium head, and press-fit titanium cup
5. Press-fit titanium stem, ceramic (alumina) head, and cementless titanium cup
PREFERRED RESPONSE: 5

DISCUSSION: Nickel is present in cobalt-chrome and stainless steel alloys used in orthopaedic surgery; therefore,
these materials are not well suited for nickel-sensitive patients. Nickel is not present in titanium alloys or in
ceramic components. Titanium is therefore the material of choice for the femoral and acetabular component.
Titanium is a poor option for the femoral head due to its susceptibility to abrasive wear. Titanium is also poorly
suited to cemented applications in hip arthroplasty because it is less stiff than cobalt-chrome (and stainless steel),
and therefore transmits greater stresses to the cement column. Titanium’s poor abrasion resistance can also leave
the component susceptible to increased abrasive wear in the event of loosening and micromotion. Therefore, of the
options available, a cementless titanium stem and socket with a ceramic head is the best choice. In nickel-sensitive
total knee arthroplasty patients, cemented oxidized zirconium femoral components can be used in place of cobalt-
chrome, and titanium tibial components (press-fit or cemented) have been used with success.

REFERENCE: Barrack RL, Booth RE Jr, Lonner JH, et al (eds): Orthopaedic Knowledge Update: Hip and Knee
Reconstruction 3. Rosemont, IL, American Academy of Orthopaedic Surgeons, 2006, pp 345-368.
Question 68
Implant position at the time of primary total knee arthroplasty to optimize patellar tracking includes which of the
following?

1. External rotation of the femoral and tibial components


2. Internal rotation of the femoral component and external rotation of the tibial component
3. Internal rotation of the femoral and tibial components
4. Medialization of the femoral component
5. Lateralization of the patellar component

PREFERRED RESPONSE: 1

DISCUSSION: Component rotation is a critical factor to optimizing patellar tracking at the time of primary and
revision total knee arthroplasty. Both the femoral and tibial components should be externally rotated whereas the
patellar components should be medialized.

REFERENCES: Barrack RL, Booth RE Jr, Lonner JH, et al (eds): Orthopaedic Knowledge Update: Hip and Knee
Reconstruction 3. Rosemont, IL, American Academy of Orthopaedic Surgeons, 2006, pp 123145.
Berger RA, Crossett LS, Jacobs JJ, et al: Malrotation causing patellofemoral complications after total knee
arthroplasty. Clin Orthop Relat Res 1998;356:144-153.
Malo M, Vince KG: The unstable patella after total knee arthroplasty: Etiology, prevention and management. J
Am Acad Orthop Surg 2003; 11:364-371.

AL-Madena Copy

49
54 • American Academy of Orthopaedic S urgeons

Question 69
The placement of supplemental screw fixation with acetabular component fixation is a typical adjunct measure
but carries the greatest risk of vascular injury if placed in which of the following positions?

1. Superior
2. Posterior superior
3. Posterior inferior
4. Superior lateral
5. Anterior superior

PREFERRED RESPONSE: 5

DISCUSSION: Anatomic studies indicate that the safe areas for screws are superior and posterior. The external
illiac artery is at risk with anteromedial placement, and the sciatic nerve may be compromised by posterior
inferior screw placement.

REFERENCES: Lavemia CJ, Cook CC, Hernandez RA, et al: Neurovascular injury in acetabular reconstruction
cage surgery: An anatomical study. J Arthroplasty 2007;22:124-132.
Wasielewski RC, Galat DD, Sheridan KC, et al: Acetabular anatomy and transacetabular screw fixation at the
high hip center. Clin Orthop Relat Res 2005 ;43 8:171 -176.
AL-Madena Copy
Question 70
A 71-year-old woman has a failed revision hip arthroplasty and is undergoing a re-revision hip arthroplasty. Her
last hip surgery was 4 years ago with revision of the acetabular component. Radiographs show a well-fixed
extensively porous-coated femoral component and a failed acetabular component with proximal and medial
migration through the floor of the acetabulum. Preoperative laboratory studies reveal an erythrocyte
sedimentation rate (ESR) of 70 mm/h (normal 0-29 mm/h), a C-reactive protein (CRP) of 23.3 (normal 0.2-8.0),
and a negative hip aspiration. At the time of surgery, tissues look inflamed and a frozen section shows 20 WBC
per high power field; however, a Gram stain is negative. What is the most appropriate action at this point?

1. Proceed with the revision as planned


2. Obtain cultures and proceed with revision of the acetabulum only
3. Obtain cultures and proceed with revision of the femur only
4. Obtain cultures, remove the implants, and insert an antibiotic spacer
5. Obtain cultures and close

PREFERRED RESPONSE: 4

DISCUSSION: Despite the negative aspiration preoperatively, intraoperative findings are suspicious for infection.
Additionally, the preoperative blood work is also concerning for infection with an elevated CRP and ESR. The
frozen section is also positive. Most important is the unreliability of the Gram stain. Numerous investigators have
show high false negative rates for Gram stain in chronic periprosthetic infection. The Gram stain should not be
relied on for decision-making in revision surgery, particularly when other investigations point to infection. With
the information available, the diagnosis is deep infection. The best course of action is to obtain cultures, remove
the implants, and insert an antibiotic spacer. Only obtaining cultures and closing would require a second operation
to remove the implants if the cultures are positive.

REFERENCES: Sanzen L, Sundberg M: Periprosthetic low-grade hip infections: Erythrocyte sedimentation rate
and C-reactive protein in 23 cases. Acta Orthop Scand 1997;68:461-465.
Spangehl MJ, Hanssen AD, Osman DR: Diagnosis and treatment of the infected hip arthroplasty, in Morrey
AL-Madena
BF(ed): Copy
Joint Replacement Arthroplasty, ed 3. Philadelphia, PA, Churchill Livingstone, 2003, pp 856-874.
Question 71

50
2010 Adult Reconstructive S urgery of the Hip and Knee Examination Answer Book • 55

A 79-year-old patient has a history of peripheral vascular disease and reports chronic knee pain. She has had
coronary artery disease treated with angiography and stents on two occasions. Peripheral pulses are absent in
both lower extremities, but the patient is disabled by advanced chronic degenerative arthritis in her right knee
and would like to proceed with a total knee arthroplasty. The next most appropriate evaluation should include
which of the following?

1. Ankle-brachial index of the affected lower extremity


2. Femoral popliteal angiography
3. Venous Dopplers of both lower extremities
4. MRI of the popliteal fossa

5. Radiographs to identify calcified plaques in the femoral artery

PREFERRED RESPONSE: 1
DISCUSSION: This question is designed to draw attention to the fact that peripheral vascular disease carries an
increased risk of complications for the patient and should be carefully evaluated. The vascular surgeon will
make the choice of revascularization or surgical clearance for knee reconstruction based on the initial results of
the ankle-brachial index.

REFERENCE: Smith DE, McGraw RW, Taylor DC, et al: Arterial complications and total knee arthroplasty. J
Am Acad Orthop Surg 2001 ;9:253-257.

Figure 72

AL-Madena Copy

51
56 • American Academy of Orthopaedic S urgeons

Question 72
A 56-year-old woman with rheumatoid arthritis who underwent total hip arthroplasty 17 years ago now reports pain
and progressive shortening of the extremity over the past year. An AP radiograph of the hip is shown in Figure 72.
Laboratory studies show an erythrocyte sedimentation rate (ESR) of 34 mm/h (normal 0 to 28 mm/h) and a C-
reactive protein of 10.2 (normal 0.2-8.0). She is presently taking oral antibiotics for a urinary tract infection. What
is the next most appropriate step in management?

1. Aspiration of the hip joint and if negative, revision of the acetabular component
2. Aspiration of the hip joint and culture once she has discontinued the antibiotics for 3 weeks
3. Triple phase bone scan
4. Indium-111 leukocyte scan

5. Serial plain radiographs and protected weight bearing PREFERRED RESPONSE: 2


DISCUSSION: The patient has a loose acetabular component, which explains her pain and progressive shortening.
She has a history of inflammatory arthritis, elevated ESR and C-reactive protein, and has recently been treated for
an infection. Thus, the suspicion for infection is high and must be ruled out. A triple phase bone scan can assist in
the identification of component loosening but cannot differentiate infection from noninfectious causes. Indium-111
scans have been shown to have limited utility, although a negative scan can be helpful in ruling out infection. The
selective preoperative use of aspiration of the hip joint has been shown to be effective and is most likely to identify
infection; however, the patient must be off of antibiotics for a minimum of 2 weeks prior to her aspiration to avoid
a false negative culture.

REFERENCES: Della Valle CJ, Zuckerman JD, Di Cesare PE: Periprosthetic sepsis. Clin Orthop Relat Res
2004;420:26-31.
Lachiewicz PF, Rogers GD, Thomason HC: Aspiration of the hip joint before revision total hip arthroplasty:
Clinical and laboratory factors influencing attainment of a positive culture. J Bone Joint Surg Am 1996;78:749-754.
Barrack RL, Booth RE Jr, Lonner JH, et al (eds): Orthopaedic Knowledge Update: Hip and Knee Reconstruction 3.
Rosemont, IL, American Academy of Orthopaedic Surgeons, 2006, pp 475-503.

AL-Madena Copy

52
2010 Adult Reconstructive S urgery of the Hip and Knee Examination Answer Book • 57

Figure 73

Question 73
An active 18-year-old patient reports severe left hip pain that prevents her from playing lacrosse. An AP
radiograph of the pelvis is shown in Figure 73. What is the most appropriate option for this patient?

1. Activity modification
2. Hip fusion
3. Periacetabular osteotomy
4. Femoral osteotomy
5. Total hip arthroplasty

PREFERRED RESPONSE: 3

DISCUSSION: The patient has developmental dysplasia of the hip (DDH). There is anterolateral deficiency of the
acetabulum as is evidenced by the increased acetabular index and the reduced center- edge angle. The patient has
some arthritis of the hip with narrowing of the joint space and cyst formation visible on the radiograph. Although
all of the mentioned choices may be acceptable treatments for dysplasia of the hip, periacetabular osteotomy is the
best and most appropriate option for this young patient. Periacetabular osteotomy allows correction of the problem
and can even improve the joint space as the new region of the acetabulum is rotated into the weight-bearing
region. Hip fusion is very poorly tolerated by young patients, especially women. Femoral osteotomy alone is
unlikely to address the problem because the major problem is on the acetabular side. Although the patient has
arthritis, because of the young age of the patient, prosthetic replacement is not an attractive option. The outcome
of periacetabular osteotomy even for patients with moderate arthritis has been favorable, deferring the need for
total hip arthroplasty by a mean of 6.5 years.

REFERENCES: Weinstein SL, Mubarak SJ, Wenger DR: Developmental hip dysplasia and dislocation: Part 1.
Instr Course Lect 2004;53:523-530.
Parvizi J, Burmeister H, Ganz R: Previous Bernese periacetabular osteotomy does not compromise the results of
total hip arthroplasty. Clin Orthop Relat Res 2004;423:118-122.

AL-Madena Copy

53
58 • American Academy of Orthopaedic S urgeons

Figure 74

Question 74
Figure 74 shows the radiograph of an 84-year-old woman who reports severe right knee pain. At the time of total
knee arthroplasty, she is found to have gross insufficiency and attenuation of the medial collateral ligament
(MCL) complex. Optimal management should consist of

1. primary repair of the MCL and use of a posterior stabilized total knee arthroplasty (TKA)
prosthesis.
2. augmentation of the MCL with a collagenous tissue scaffold and use of a posterior stabilized TKA
prosthesis.
3. complete release of the lateral collateral ligament (LCL) and use of a posterior stabilized TKA
prosthesis.
4. lateral unicompartmental arthroplasty.
5. use of a varus-valgus constrained TKA prosthesis.

PREFERRED RESPONSE: 5

DISCUSSION: Patients with severe valgus deformity may have near complete attenuation of the MCL.
Attempts at ligament repair or reconstruction at the time of TKA can have unpredictable outcomes, leading
to an unstable TKA. Although there may be a role for trying to reconstruct the ligament in conjunction with
a nonconstrained implant in young patients with long life expectancies, in elderly patients a constrained
prosthesis can provide varus-valgus stability with a predictable outcome. In younger patients, there is
concern that the extra prosthetic constraint may shorten the longevity of the prosthetic fixation. In older
patients, the constrained implant is likely to last a lifetime, with several studies documenting excellent
survivorship (96%) at 10 years. Complete release of the LCL will leave the knee grossly unstable medially
and laterally, and could necessitate a hinged prosthesis.

REFERENCES: Lachiewicz PF, Soileau ES: Ten-year survival and clinical results of constrained
components in primary total knee arthroplasty. J Arthroplasty 2006;21:803-808.
Anderson JA, Baldini A, MacDonald JH, et al: Primary constrained condylar knee arthroplasty without stem
extensions for the valgus knee. Clin Orthop Relat Res 2006;442:199-203.

AL-Madena Copy

54
2010 Adult Reconstructive S urgery of the Hip and Knee Examination Answer Book • 59

Figure 75

Question 75
Figure 75 shows the radiograph of a healthy 52-year-old woman who has severe right hip pain that has been
unresponsive to nonsurgical management. What is the most appropriate surgical procedure at this time?

1. Total hip arthroplasty


2. Hemiarthroplasty of the hip
3. Valgus femoral osteotomy
4. Periacetabular osteotomy
5. Varus femoral osteotomy

PREFERRED RESPONSE: 1

DISCUSSION: The patient has developmental dysplasia of the right hip, as can be noted by the shallow
acetabulum and lack of femoral head coverage. She has secondary osteoarthritis, manifested radiographically by
joint space narrowing, periarticular sclerosis, and subchondral acetabular cyst formation. Femoral and/or acetabular
redirectional osteotomies are most effective when performed before the onset of arthritis. After osteoarthritis has
set in, total hip arthroplasty is the most reliable procedure for reducing pain. Hemiarthroplasties are not indicated in
the presence of arthritic changes of the acetabulum.

REFERENCE: Barrack RL, Booth RE Jr, Lonner JH, et al (eds): Orthopaedic Knowledge Update: Hip and Knee
Reconstruction 3. Rosemont, IL, American Academy of Orthopaedic Surgeons, 2006, pp 403-414.
Question 76
Which of the following surgical maneuvers is most likely to enhance proper patellar tracking during total knee
arthroplasty and minimize the need for a lateral retinacular release?

1. Use of a mobile-bearing prosthesis


2. Anterior placement of the tibial tray
3. Internal rotation of the femoral component
4. Internal rotation of the tibial component
5. External rotation of the tibial component

PREFERRED RESPONSE: 5

DISCUSSION: Slight external rotation of the tibial component medializes the tibial tubercle and helps reduce the
tendency for lateral patellar maltracking. Mobile bearings have not been shown to improve patellar tracking.

AL-Madena Copy

55
60 • American Academy of Orthopaedic S urgeons

Internal rotation of the tibial or femoral component will accentuate patellar maltracking. AP positioning of the
tibial tray will affect the force across the patella, but it is not likely to affect patellar tracking.

REFERENCES: Pelicci PM, Tria AJ Jr, Garvin KL (eds): Orthopaedic Knowledge Update: Hip and Knee
Reconstruction 2. Rosemont, IL, American Academy of Orthopaedic Surgeons, 2000, pp 323-337. Pagnano MW,
Trousdale RT, Stuart MJ, et al: Rotating platform knees did not improve patellar tracking: A prospective
randomized study of 240 primary total knee arthroplasties. Clin Orthop Relat Res 2004:428:221-227.

Figure 77a Figure 77b

Question 77
An obese 62-year-old man (BMI 38) who underwent a cementless total hip arthroplasty 14 months ago fell from a
ladder and is now unable to bear weight on the extremity. A radiograph from his 3-month visit is shown in Figure
77a and a radiograph from the time of the injury is shown in Figure 77b. Appropriate management includes which

AL-Madena Copy

56
2010 Adult Reconstructive S urgery of the Hip and Knee Examination Answer Book • 61

of the following?

1. Nonsurgical management with the use of a cast-brace


2. Nonsurgical management with skeletal traction
3. Open treatment with a plate with or without strut allograft
4. Revision of the femoral component to a cemented femoral component that bypasses the fracture
site by two cortical diameters
5. Revision of the femoral component to a cementless femoral component that bypasses the
fracture site by at least two cortical diameters

PREFERRED RESPONSE: 3

DISCUSSION: The patient has a Vancouver type B1 fracture of the femur with a well-fixed femoral component.
When comparing the two radiographs, the proximal femoral component position has not changed and remodeling is
seen around the component. Nonsurgical management is associated with the complications of extended
recumbency. As the femoral component is well-fixed, revision of the femoral component is not necessary.

REFERENCES: Masri BA, Meek RM, Duncan CP: Periprosthetic fractures evaluation and treatment.
Clin Orthop Relat Res 2004;420:80-95.
Barrack RL, Booth RE Jr, Lonner JH, et al (eds): Orthopaedic Knowledge Update: Hip and Knee Reconstruction 3.
Rosemont, IL, American Academy of Orthopaedic Surgeons, 2006, pp 475-503.
Ricci WM, Bolhofner BR, Lofitus T, et al: Indirect reduction and plate fixation, without grafting, for
periprosthetic femoral shaft fractures about a stable intramedullary implant. J Bone Joint Surg Am
2005;87:2240-2245.
Question 78
An 83-year-old man with a history of diabetes mellitus reports abdominal pain on postoperative day number three
following a total hip arthroplasty. The patient reports having a bowel movement the prior evening. Examination
reveals that the abdomen is distended but nontender. What is the next step in management?

1. Rectal examination for occult blood


2. Insertion of a nasogastric tube with lavage to identify blood in the gastric contents
3. Radiograph of the abdomen
4. Ultrasound of the abdomen
5. CT of the abdomen
PREFERRED RESPONSE: 3

DISCUSSION: The patient has risk factors, symptoms, and signs of Ogilvie syndrome of acute colonic pseudo-
obstruction. This unusual but potentially catastrophic complication is characterized by functional colonic
obstruction without an associated mechanical blockage. This disorder has been associated with advanced age,
male gender, the use of narcotic pain medications, and patients who have undergone hip arthroplasty. The first
step in management of any complication is diagnosis, and the diagnosis is most rapidly made using radiographs
that show dilation of the large intestine.

REFERENCES: Nelson JD, Urban JA, Salsbury TL, et al: Acute colonic pseudo-obstruction (Ogilvie syndrome)
after arthroplasty in the lower extremity. J Bone Joint Surg Am 2006;88:604-610.
Clarke HD, Berry DJ, Larson DR: Acute pseudo-obstruction of the colon as a postoperative complication of hip
arthroplasty. J Bone Joint Surg Am 1997;79:1642-1647.

AL-Madena Copy

57
62 • American Academy of Orthopaedic S urgeons

Question 79
A 46-year-old man reports occasional squeaking of his hip 2 years after undergoing an uneventful total hip
arthroplasty. History reveals no pain, physical examination cannot reproduce audible squeaking, and radiographs
show appropriate implant position. What is the most appropriate management?

1. Revise the cup bearing to polyethylene


2. Revise the cup bearing to polyethylene and replace the femoral head with a metal design
3. Increase the cup abduction angle
4. Decrease the cup abduction angle
5. Continue routine follow-up and observation

PREFERRED RESPONSE: 5

DISCUSSION: In the absence of component malpositioning, hip pain, or other compelling reasons to reoperate, a
squeaking ceramic bearing is not an indication for revision surgery. The patient can be reassured and observed.
Hopefully, with a better understanding of acoustic phenomena following ceramic total hip arthroplasty, this
complication can be minimized.
REFERENCES: Yang CC, Kim RH, Dennis DA: The squeaking hip: A cause for concem-disagrees. Orthopedics
2007;30:739-742.
Walter WL, O’Toole GC, Walter WK, et al: Squeaking in ceramic-on-ceramic hips: The importance of acetabular
component orientation. J Arthroplasty 2007;22:496-503.

Figure 80a Figure 80b

Question 80
A healthy 74-year-old man reports right knee pain and swelling 6 years after undergoing primary total knee
arthroplasty. Radiographs are shown in Figures 80a and 80b. He states he had severe bilateral bowlegged deformity
for the last 20 years, but the right side got “a little better after the knee arthroplasty.” The infection work-up is
negative, and a review of a bone scan and serial radiographs reveals no signs of component loosening. Lateral
ligaments appear lax on examination. Optimal surgical management should consist of which of the following?

1. Tibial polyethylene exchange


2. Tibial polyethylene exchange and lateral collateral ligament reconstruction and
medial collateral ligament release
3. Revision of all components and soft-tissue balancing

AL-Madena Copy

58
2010 Adult Reconstructive S urgery of the Hip and Knee Examination Answer Book • 63

4. Isolated tibial revision

5. Femoral revision and tibial polyethylene exchange PREFERRED RESPONSE: 3


DISCUSSION: The prosthesis has failed 6 years after implantation. In this patient, the tibial component is in 5
degrees of varus, so the varus mechanical axis was never restored to neutral. The polyethylene bearing is
significantly narrowed medially, a sign of incomplete release of the tight medial structures and residual varus
angulation of the limb. A polyethylene exchange alone will not correct the varus malalignment of the tibia, nor will
a femoral revision with polyethylene exchange. An isolated tibial revision will correct the varus but will probably
render the posterior cruciate ligament incompetent, necessitating a femoral revision to a posterior cruciate -
substituting design. The procedure most likely to result in a successful outcome is a full revision. Soft-tissue
releases to balance the medial and lateral sides also are required.
REFERENCES: Lotke PA, Garino JP: Revision Total Knee Arthroplasty. New York, NY, Lippincott- Raven,
1999, pp 137-250.
Insall JN, Windsor RE, Scott WN, et al (eds): Surgery of the Knee, ed 2. New York, NY, Churchill Livingstone,
1993, pp 935-957.
Pellicci PM, Tria AJ Jr, Garvin KL (eds): Orthopaedic Knowledge Update: Hip and Knee Reconstruction
2. Rosemont, IL, American Academy of Orthopaedic Surgeons, 2000, pp 339-365.

Question 81
Which of the following organisms is most often found in a late (> 3 months) infection of a total hip arthroplasty?

1. Staphylococcus aureus
2. Staphylococcus epidermidis
3. Group B streptococcus
4. Eschericia coli
5. Streptococcus viridans
PREFERRED RESPONSE: 2

DISCUSSION: Staphylococcus epidermidis is the most common organism found in an infected total hip
arthroplasty greater than 3 months from the origional surgery. Staphylococcus aureus is more common in acute
postoperative infections, and E. coli is associated with infections of the urinary tract. Streptococcus species are
less common.

REFERENCE: Barrack RL, Booth RE Jr, Lonner JH, et al (eds): Orthopaedic Knowledge Update: Hip and Knee
Reconstruction 3. Rosemont, IL American Academy of Orthopedic Surgeons, 2006, pp 475-503.

AL-Madena Copy

59
64 • American Academy of Orthopaedic S urgeons

Figure 82

Question 82
A patient sustained a periprosthetic femoral fracture. The proximal femur is comminuted and the femoral
component is loose. The patient has absent pulses and poor capillary refill. An emergent arteriogram is shown in
Figure 82. What is the most appropriate management?

1. Open reduction and internal fixation of the fracture


2. Traction, followed by revision total hip arthroplasty and vascular repair
3. Revision total hip arthroplasty, four compartment fasciotomy, and sequential neurovascular
examination
4. Revision total hip arthroplasty, followed by vascular repair
5. Traction, followed by vascular shunting, revision total hip arthroplasty, and definitive vascular repair
PREFERRED RESPONSE: 5

DISCUSSION: The patient requires emergent revascularization with signs of ischemia of unknown duration. The
leg has shortened substantially and initial management requires traction to return the leg to length. This should be
followed by revascularization of the leg. A temporary shunt may be helpful until the fracture has been stabilized
(revision total hip arthroplasty) with the leg at full length. Definitive vascular bypass should then be performed.

REFERENCES: Calligaro KD, Dougherty MJ, Ryan S, et al: Acute arterial complications associated with total
hip and knee arthroplasty. J Vase Surg 2003;38:1170-1177.
Brady OH, Garbuz DS, Masri BA, et al: Classification of the hip. Orthop Clin North Am 1999;30;215- 220.
Question 83
During normal human knee flexion (beginning with the knee fully extended), which of the following statements
best describes tibial rotation with respect to the femur?

1. Rotation is constantly occurring in both directions during the flexion cycle.


2. The tibia initially externally rotates, then progressively internally rotates.
3. The tibia initially internally rotates, then progressively externally rotates.
4. The tibia initially internally rotates, then remains in that rotational position until deep flexion when
further internal rotation occurs.
5. The tibia initially externally rotates, then remains in that rotational position until deep flexion when
further external rotation occurs.

PREFERRED RESPONSE: 4

AL-Madena Copy

60
2010 Adult Reconstructive S urgery of the Hip and Knee Examination Answer Book • 65

DISCUSSION: During knee flexion, the tibia initially rotates internally in approximately the first 20 degrees and
generally maintains this rotational position until flexion past 90 degrees when significantly more internal rotation
occurs.

REFERENCE: Coughlin KM, Incavo SJ, Churchill DL, et al: Tibial axis and patellar position relative to the
femoral epicondylar axis during squatting. J Arthroplasty 2003;18:1048-1055.

Question 84
The anterior approach to the hip (iliofemoral or Smith-Peterson) puts which of the following anatomic structures
at greatest risk?

1. Femoral artery
2. Femoral nerve
3. Lateral femoral cutaneous nerve
4. Medial femoral circumflex artery
5. Obturator artery

PREFERRED RESPONSE: 3

DISCUSSION: The anterior approach to the hip involves a dissection between the sartorius and the tensor fascia
lata (TFL) superficially, followed by a deep dissection between the rectus femoris and gluteus medius. The
lateral femoral cutaneous nerve generally enters the top of the thigh overlying the sartorius, and then usually
crosses the interval between the sartorius muscle and the tensor fascia lata more distally. As the fascia between
the sartorius and the TFL is incised, the nerve is at risk. The ascending branch of the late ral femoral circumflex
artery is also at risk during this approach. The femoral nerve should not be in the plane of dissection as it lies
medial to the sartorius.

REFERENCE: Hoppenfeld S, DeBoer P: Surgical Exposures in Orthopaedics: The Anatomic Approach.


Philadelphia, PA, JB Lippinocott, 1984, pp 301-315.

AL-Madena Copy

61
66 • American Academy of Orthopaedic S urgeons

Question 85
Which of the following polyethylene manufacturing processes is expected to generate the greatest degree of
polyethylene oxidation?

1. Sterilization with ethylene oxide


2. Sterilization with gamma radiation in a vacuum condition
3. Sterilization with gamma radiation in air
4. Sterilization with gamma radiation in oxygen-free gasses
5. Cross-linking the polyethylene followed by melting
PREFERRED RESPONSE: 3

DISCUSSION: Oxidation of polyethylene has been associated with increased rates of polyethylene wear.
Oxidation occurs after polyethylene has been irradiated in the presence of oxygen. Gamma irradiation has been
commonly employed to sterilize the polyethylene prior to sterile packaging. Over the last decade, several
methods of reducing oxidation of polyethylene have been used. These include irradiation in an inert gas (such as
argon or nitrogen), irradiation in vacuum packaging, and avoiding irradiation altogether and sterilizing the
polyethylene with ethylene oxide, gas plasma, or vaporized hydrogen peroxide. Crosslinking polyethylene has
been done with gamma irradiation and electron beam irradiation. Heating/ melting the material after irradiation
allows the free radical chains within the polyethylene to cross-link together rather than oxidize.

REFERENCE: Barrack RL, Booth RE Jr, Lonner JH, et al (eds): Orthopaedic Knowledge Update: Hip and Knee
Reconstruction 3. Rosemont, IL, American Academy of Orthopaedic Surgeons, 2006, pp 333-344.

Question 86
When compared to total hip arthroplasty, hip resurfacing offers which of the following advantages?

1. Better patient compliance with precautions


2. Increased patient activity in sports
3. Increased mobility of the hip
4. Improved pain relief
5. Preservation of proximal femoral bone

PREFERRED RESPONSE: 5

DISCUSSION: When compared to the outcomes of conventional hip arthroplasty with large diameter femoral
heads, hip resurfacing does not result in better outcomes in terms of compliance with hip precautions, increased
hip mobility, pain relief, or improved patient activity. The advantage of hip resurfacing is in preserving proximal
femoral bone stock that may be advantageous in revision surgery, should that become necessary.

AL-Madena Copy

62
2010 Adult Reconstructive S urgery of the Hip and Knee Examination Answer Book • 67
REFERENCES: Schmalzried TP: Why total hip resurfacing. J Arthroplasty 2007;22:57-60. Mont MA,
Ragland PS, Etienne G, et al: Hip resurfacing arthroplasty. J Am Acad Orthop Surg 2006;14:454-463.

Figure 87a Figure 87b

Question 87
A 78-year-old woman underwent total hip arthroplasty 15 years ago. She reports a recent history of increasing
thigh pain prior to a fall and is now unable to ambulate. Radiographs are shown in Figures 87a and 87b. What is
the best treatment for this condition?

1. Surgical traction for 6 weeks followed by application of a cast brace


2. Application of a femoral cable plate
3. Femoral revision with a cemented long stem prosthesis
4. Application of cerclage wired double allograft femoral struts
5. Femoral revision with a cementless long taper fluted modular stem and proximal allograft
strut supplementation

PREFERRED RESPONSE: 5

DISCUSSION: Severe periprosthetic fractures after total hip arthroplasty with a loose implant and progressive
bone loss are difficult problems for orthopaedic surgeons, with a high complication rate. Recent literature favors
the use of long fluted tapered stems that have a long distal taper that may optimally engage the remaining femoral
shaft isthmus. Plating options are problematic because the ability to use screws with the plate is limited by the
intramedullary stem. Although not the only solution to this problem (such as allograft-prosthetic composites,
impaction grafting, tumor prostheses), long distally fixed stems circumvent this problem by enhancing fracture
healing and create a long-term prosthetic solution in these most difficult cases.

REFERENCES: Barrack RL, Booth RE Jr, Lonner JH, et al (eds): Orthopaedic Knowledge Update: Hip and Knee
Reconstruction 3. Rosemont, IL, American Academy of Orthopaedic Surgeons, 2006, pp 475503.
Kwong LM, Miller AJ, Lubinus P: A modular distal fixation option for proximal bone loss in revision total hip
arthroplasty: A 2- to 6-year follow-up study. J Arthroplasty 2003;18:94-97.
AL-Madena Copy

63
68 • American Academy of Orthopaedic S urgeons

Figure 88a Figure 88b

Question 88
A 45-year-old man underwent a femoral varus intertrochanteric osteotomy at age 19 years for Perthes disease. He
now reports intractable left hip pain, is unable to ambulate more than Vi block, and has pain on stairs. Adjunct
nonsurgical management, such as nonsteroidal anti-inflammatory drugs and physical therapy, has failed to
provide relief. Radiographs shown in Figures 88a and 88b reveal end-stage degenerative joint disease. What is
the most appropriate management of the proximal femoral deformity?

1. Femoral osteotomy with realignment of the femoral canal to accommodate a cementless prosthesis
2. Custom prosthesis that will accommodate the prior osteotomy deformity
3. Hip arthrodesis
4. Cemented femoral component without femoral osteotomy
5. Valgus femoral osteotomy

PREFERRED RESPONSE: 1

DISCUSSION: Whereas the choice of a particular option may seem controversial in this scenario because all
answers seem possible, the literature favors a femoral osteotomy of the femoral canal with a cementless
prosthesis that offers some form of distal fixation. Such implants are widely available and allow for healing of the
reconstructive osteotomy. Custom implants are problematic because bone assessing algorithms may not be
accurate with the dysplastic hip. Hip arthrodesis is not a good option in this age group because ipsilateral spine
and knee arthritis can be aggravated. Finally, cemented femoral stem placement has given way to cementless
prosthetics in younger patients because the results of cement in osteotomy cases are inferior.

REFERENCES: Parsch D, Jung AW, Thomsen M, et al: Good survival of uncemented tapered stems for failed
intertrochanteric osteotomy: A mean 16 years followup study in 45 patients. Arch Orthop Trauma Surg
2007;128:1081-1085.
Suzuki K, Kawachi S, Matsubara M, et al: Cementless total hip replacement after previous intertrochanteric
valgus osteotomy for advanced osteoarthritis. J Bone Joint Surg Br 2007;89:1155-1157. Zadeh HG, Hua J,
Walker PS, et al: Uncemented total hip arthroplasty with subtrochanteric derotational osteotomy for severe
femoral anteversion. J Arthroplasty 1999;14:682-688.

AL-Madena Copy

64
2010 Adult Reconstructive S urgery of the Hip and Knee Examination Answer Book • 69
Question 89
Which of the following has been associated with an increased likelihood of stress shielding after cementless total
hip arthroplasty?

1. Use of titanium alloy femoral components


2. Use of proximally fixed femoral components
3. Use of distally fixed femoral components
4. Use of press-fit stems in patients with narrow intramedullary canals
5. Use of plasma sprayed components

PREFERRED RESPONSE: 3

DISCUSSION: Although stress shielding has not been associated with adverse clinical outcomes to date, it is a
commonly observed process after cementless total hip arthroplasty. Factors that increase the magnitude of stress
shielding include the use of distally fixed cobalt-chrome stems, particularly in patients with osteoporosis. Large
diameter femoral components (larger than 18 mm in diameter) have also been associated with an increased
prevalence of stress shielding.

REFERENCE: Barrack RL, Booth RE Jr, Lonner JH, et al (eds): Orthopaedic Knowledge Update: Hip and Knee
Reconstruction 3. Rosemont, IL, American Academy of Orthopaedic Surgeons, 2006, pp 345-368.

Question 90
During total hip arthroplasty, neurologic injury most commonly occurs in which of the following structures?

1. Inferior gluteal nerve


2. Obturator nerve
3. Peroneal branch of the sciatic nerve
4. Tibial branch of the sciatic nerve
5. Femoral nerve

PREFERRED RESPONSE: 3

DISCUSSION: The incidence of nerve injury with total hip arthroplasty is approximately 1%. The sciatic nerve is
involved roughly 80% of the time, with the peroneal branch being almost always involved. Isolated tibial branch
involvement is reported to occur in only 1% of neurologic injuries related to hip arthroplasty. The superior gluteal
nerve may be injured in direct lateral approaches.

REFERENCES: Barrack RL: Neurovascular injury: Avoiding catastrophe. J Arthroplasty 2004; 19:104107.
Lewallen DG: Neurovascular injury associated with hip arthroplasty. Instr Course Lect 1998;47:275-283.
Schmalzried TP, Noordin S, Amstutz HC: Update on nerve palsy associated with total hip replacement. Clin
Orthop Relat Res 1997;344:188-206.
AL-Madena Copy

65
70 • American Academy of Orthopaedic S urgeons

Figure 91a Figure 91b

Question 91
A 71-year-old man underwent an uncomplicated hybrid ceramic-on-conventional polyethylene hip arthroplasty 8
years ago. He now has minimal hip symptoms, but radiographs show massive acetabular osteolysis. An AP pelvis
radiograph is shown in Figure 91a and a sagittal CT scan is shown in Figure 91b. What is the most appropriate
treatment?

1. Symptomatic treatment with analgesics and a follow-up radiograph in 2 years


2. Revision of the acetabular component and exchange of the ceramic head
3. Revision of both the acetabular and femoral components
4. Revision of the acetabular liner, bone grafting, and exchange of the femoral head to metal

5. Revision of the acetabulum and exchange of the femoral head to metal PREFERRED

RESPONSE: 4
DISCUSSION: The patient has massive osteolysis for which surgery is indicated. Follow-up in 2 years risks further
progression of osteolysis and the potential for catastrophic failure. The acetabular component appears well-fixed
(CT scan shows a pillar of bone extending down to the dome of the acetabulum) and well positioned and removal
should be avoided because the resulting defect may be extremely large and difficult to reconstruct. A liner
exchange is favored, provided that the cup is well positioned and of sufficient size to accept a new liner with
adequate thickness. In this case, bone grafting behind the socket (without destabilizing it) would be indicated,
although routine bone grafting of osteolytic defects is controversial. Revising a ceramic head to a new ceramic
head should also be avoided because ceramic head fractures have occurred with placement on a used taper. Metal
adapter sleeves are available for some implants if a ceramic head is desired.

REFERENCES: Barrack RL, Burak C, Skinner HB: Concerns about ceramic in THA. Clin Orthop Relat Res
2004;429:73-79.
Lombardi AV Jr, Berend ICR: Isolated acetabular liner exchange. J Am Acad Orthop Surg 2008; 16:243248.
Naudie DD, Engh CA Sr: Surgical management of polyethylene wear and pelvic osteolysis with modular
uncemented acetabular components. J Arthroplasty 2004; 19; 124-129.
Question 92
Advantages of a resurfacing metal-on-metal hip arthroplasty over a large diameter metal-on-metal total hip
arthroplasty include which of the following?

AL-Madena Copy

66
2010 Adult Reconstructive S urgery of the Hip and Knee Examination Answer Book • 71

1. Lower risk of femoral component loosening


2. Acetabular bone preservation
3. Lower reoperation rate
4. Femoral bone preservation
5. Lower wear rate
PREFERRED RESPONSE: 4

DISCUSSION: A resurfacing hip arthroplasty preserves bone stock in the proximal femur, at the expense of a
higher reoperation rate because of component loosening and femoral neck fracture. Wear rate is the same as both
types of hip arthroplasty use a large head metal-on-metal bearing surface.

REFERENCES: Shimmin A, Beaule PE, Campbell P: Metal-on-metal hip resurfacing arthroplasty. J Bone Joint
Surg Am 2008;90:637-654.
Buergi ML, Walter WL: Hip resurfacing arthroplasty: The Australian experience. J Arthroplasty
2007;22:61-65.

Question 93
Patients with fulminant disseminated intravascular coagulation (DIC) have which of the following
findings?

1. Patients frequently have elevated fibrinogen levels.


2. Patients frequently have decreased D-Dimer levels.
3. Rapid infusion of IV heparin is generally curative.
4. Prothrombin time (PT) is usually normal.
5. Activated partial thromboplastin time (aPTT) is frequently elevated.

PREFERRED RESPONSE: 5

DISCUSSION: Disseminated intravascular coagulation is a syndrome that spans a spectrum from relatively
asymptomatic to life-threatening. In its fulminant form, patients undergo widespread microvascular thrombosis
leading to overconsumption of coagulation factors and platelets, and then subsequent hemorrhage. End-organ
failure frequently results. The condition is therefore a “thrombohemorrhagic” disorder. The exact pathophysiology
remains poorly understood, but can be seen in conjunction with a variety of medical conditions, including massive
transfusions, sepsis, bums, crush injuries, liver disease, autoimmune disorders, hemolysis, obstetrical emergencies,
and malignancy. Laboratory abnormalities frequently include depressed levels of fibrinogen and platelets, increased
levels of fibrinogen degradation products and D-Dimer, and an elevation in the PT and aPTT. In fulminant DIC,
treatment is controversial and frequently unsuccessful, leading to death in affected patients. Heparin, although
commonly used, has not been shown to have beneficial effects in controlled trials. Low-grade DIC will frequently
improve with correction of the underlying medical disorder.
AL-Madena Copy
REFERENCE: Townsend CM, Beauchamp RD, Evers BM, et al (eds): Sabiston Textbook of Surgery: The
Biologic Basis of Modern Surgical Practice, ed 18. Philadelphia, PA, Saunders Elsevier, 2008, pp 122123.

67
72 • American Academy of Orthopaedic S urgeons

Figure 94

Question 94
Figure 94 shows the lateral radiograph of an 80-year-old woman who is an independent ambulator and has a
supracondylar periprosthetic fracture around the knee. What is the most appropriate management for this patient?

1. Closed reduction and nonsurgical management


2. Open reduction and internal fixation using a distal femoral locking plate
3. Open reduction and internal fixation using an intramedullary rod
4. Revision total arthroplasty using distal femoral allograft
5. Revision total knee arthroplasty using distal femoral replacement

PREFERRED RESPONSE: 5

DISCUSSION: A supracondylar femoral periprosthetic fracture that is a reducible or an irreducible fracture with
poor distal bone stock and in the vicinity of loose or malpositioned components (type III) is very difficult to treat.
One of the treatment options for patients with a loose femoral component and poor bone stock is the use of distal
femoral replacement. The distal femoral replacement can be performed with relative ease, expediency, and is best
suited for elderly or sedentary patients.

REFERENCES: Kassab M, Zalzal P, Azores GM, et al: Management of periprosthetic femoral fractures after total
knee arthroplasty using a distal femoral allograft. J Arthroplasty 2004;19:361-368.
Kim KI, Egol KA, Hozack WJ, et al: Periprosthetic fractures after total knee arthroplasties. Clin Orthop Relat Res
2006;446:167-175.
Question 95
Effective management of heterotopic ossification (HO) following total hip arthroplasty should include which of
the following?

1. Indomethacin treatment for 10 days postoperatively


2. Immediate excision of established heterotopic ossification followed by radiation therapy or
indomethacin
3. Postoperative administration of ethylhydroxydiphosphonate

AL-Madena Copy

68
2010 Adult Reconstructive S urgery of the Hip and Knee Examination Answer Book • 73

4. Preoperative administration of radiation therapy 1 week before surgery


5. Postoperative administration of radiation therapy
PREFERRED RESPONSE: 5

DISCUSSION: Postoperative administration of ethylhydroxydiphosphonate results in delay of mineralization of


osteoid, but ultimately HO formation is not decreased. In addition, the delay in mineralization does not improve
range of motion of involved hips. Indomethacin has proven to be an effective long-term therapy. To be most
effective, radiation therapy must be done in the immediate postoperative period.

REFERENCES: Iorio R, Healy WL: Heterotopic ossification after hip and knee arthroplasty: Risk factors,
prevention, and treatment. J Am Acad Orthop Surg 2002;10:409-416.
Pellegrini VD Jr, Gregoritch SJ: Preoperative irradiation for prevention of heterotopic ossification following total
hip arthroplasty. J Bone Joint Surg Am 1996;78:870-881.
Pellegrini VD Jr, Konski AA, Gastel JA, et al: Prevention of heterotopic ossification with irradiation after total hip
arthroplasty: Radiation therapy with a single dose of eight hundred centigray administered to a limited field. J
Bone Joint Surg Am 1992;74:186-200.

Question 96
Computer navigation in total knee arthroplasty (TKA) has demonstrated which of the following?

1. Decreased cost
2. Improved clinical outcomes
3. Fewer outliers in terms of component position
4. Improved longevity
5. Decreased surgical time
PREFERRED RESPONSE: 3

DISCUSSION: Computer navigation has been shown to improve the accuracy of placement of TKA
components with fewer outliers. However, the surgical time and cost are increased. No studies to date have
demonstrated improved clinical outcomes or implant survivorship.

REFERENCE: Haaker RG, Stockheim M, Kamp M, et al: Computer-assisted navigation increases precision of
component placement in total knee arthroplasty. Clin Orthop Relat Res 2005;433:152-159.
AL-Madena Copy
Question 97
Which of the following statements best describes the kinematic behavior of the knee during motion from full
extension to flexion?

1. Both the medial and lateral knee tibiofemoral contact points rotate and translate equally with
increasing knee flexion.
2. The medial femoral condyle translates much less than the lateral femoral condyle with knee flexion.
3. The lateral femoral condyle translates much less than the medial femoral condyle with knee flexion.
4. The medial compartment rotates internally whereas the lateral compartment rotates externally.
5. The lateral compartment rotates internally whereas the medial compartment rotates externally.

PREFERRED RESPONSE: 2

DISCUSSION: During normal knee flexion, knee kinematic analysis reveals that the medial tibiofemoral contact
point moves very little (translates) in the anterior-posterior direction, whereas the lateral contact point moves much

69
74 • American Academy of Orthopaedic S urgeons

greater in the anterior-posterior direction (translates), resulting in more lateral translation, rollback, and medial
pivoting.

REFERENCE: Churchill DL, Incavo SJ, Johnson CC, et al: The transepicondylar axis approximates the optimal
flexion axis of the knee. Clin Orthop Relat Res 1998;356:111-118.

Figure 98a Figure 98b Figure 98c

Question 98
A 71-year-old businessman reports medial knee pain recalcitrant to nonsurgical management.
Examination reveals that his body mass index (BMI) is 28 and he has a mild varus deformity with a range of
motion from 5 degrees to 130 degrees of flexion. Anterior drawer and Lachman’s test are negative. Radiographs are
shown in Figures 98a through 98c. For cultural and religious reasons, he is concerned about maintaining his range
of motion and kneeling ability. Which of the following options is best?

1. High tibial osteotomy


2. Unicompartmental knee arthroplasty
3. Cruciate-retaining total knee arthroplasty with a fixed bearing design
4. Posterior stabilized total knee arthroplasty with a fixed bearing design
5. Cruciate-sacrificing total knee arthroplasty with a rotating platform design

PREFERRED RESPONSE: 2

DISCUSSION: Unicompartmental arthroplasty of the knee is associated with better range of motion than either
total knee arthroplasty or high tibial osteotomy. In a prospective randomized trial of unicompartmental and total
knee arthroplasty for patients with medial compartment osteoarthritis, patients with the unicompartmental
prosthesis had better range of motion. The literature that compares range of motion in cruciate-retaining as opposed
to posterior stabilized and fixed bearing as opposed to mobile- bearing total knees suggests relatively equivalent
range of motion between these designs.

REFERENCES: Newman JH, Ackroyd CE, Shah NE: Unicompartmental or total knee replacement? Five- year
results of a prospective randomized trial of 102 osteoarthritic knees with unicompartmental arthritis.

AL-Madena Copy

70
2010 Adult Reconstructive S urgery of the Hip and Knee Examination Answer Book • 75

J Bone Joint Surg Br 1998;80:862-865.


Barrack RL, Booth RE Jr, Lonner JH, et al (eds): Orthopaedic Knowledge Update: Hip and Knee Reconstruction 3.
Rosemont, IL, American Academy of Orthopaedic Surgeons, 2006, pp 59-69.
Question 99
At the time of revision total knee arthroplasty, the surgeon is trialing the knee and finds that it extends fully and is
stable in flexion with a 23-mm trial spacer; however, the patella is impinging on the polyethylene spacer. No
augments were used on the femur or the tibia because the components fit well without them. What is the most
appropriate action at this time?

1. Proceed with implantation of the final components.


2. Perform a Z-lengthening of the patellar tendon.
3. Increase the size of the femoral component and use posterior femoral augments to decrease the size of
the flexion gap.
4. Increase the size of the femoral component and use augments both distally and posteriorly to lower
the joint line and decrease the size of the flexion gap.
5. Place distal femoral augments on the femoral component to lower the joint line.

PREFERRED RESPONSE: 4

DISCUSSION: The surgeon in this case is faced with a common scenario at the time of revision total knee
arthroplasty and the tendency is to elevate the joint line. Elevation of the joint line is associated with
deleterious effects including anterior knee pain, restricted knee flexion, and instability. The error that has been
made is resting the femoral component on the bone that is left behind after removal of the prior component;
this typically leads to a femoral component that is too small (leading to an enlarged flexion gap) and proximal
to where it should be (enlarging the extension gap). Although the flexion and extension gaps are equivalent,
joint line elevation has occurred. To correct this problem, the femoral component size should be increased or
offset posteriorly (to decrease the size of the flexion gap) and distal femoral augments should be used to
decrease the size of the extension gap and restore the joint line to the appropriate level.

REFERENCES: Laskin RS: Joint line position restoration during revision total knee replacement. Clin Orthop
Relat Res 2002;404:169-171.
Yoshii I, Whiteside LA, White SE, et al: Influence of prosthetic joint line position on knee kinematics and patellar
position. J Arthroplasty 1991;6:169-177.
Barrack RL, Booth RE Jr, Lonner JH, et al (eds): Orthopaedic Knowledge Update: Hip and Knee Reconstruction
3. Rosemont, IL, American Academy of Orthopaedic Surgeons, 2006, pp 123-145.
Question 100
A 68-year-old woman is undergoing a cementless medial/lateral tapered femoral placement during a total hip
arthroplasty and the surgeon notices a small crack forming in the anteromedial femoral neck with final implant
insertion. The most appropriate management should include which of the following?

1. Placement of a cerclage cable around the femoral neck above the lesser trochanter

AL-Madena Copy

71
76 • American Academy of Orthopaedic S urgeons

2. Removal of the implant, placement of a cable around the femoral neck above the lesser trochanter,
and reinsertion of the implant
3. Removal of the press-fit implant and cementing of the same femoral stem
4. Final seating of the cementless femoral component without additional measures
5. Removal of the cementless femoral component and placement of a revision modular taper- fluted
femoral stem

PREFERRED RESPONSE: 2

DISCUSSION: The recognized treatment of the proximal periprosthetic fracture is first to identify its extent and
then to optimize the correction. Removing the implant seems logical to accomplish the identification. Several
studies indicate that proximal cerclage wiring is adequate to create a “barrel hoop” stability of the proximal
femur. The postoperative management may also include protected weight bearing and periodic radiographs.

REFERENCES: Barrack RL, Booth RE Jr, Lonner JH, et al (eds): Orthopaedic Knowledge Update: Hip and Knee
Reconstruction 3. Rosemont, IL, American Academy of Orthopaedic Surgeons, 2006, pp 475503.
Warren PJ, Thompson P, Fletcher MD: Transfemoral implantation of the Wagner SL stem: The abolition of
subsidence and enhancement of osteotomy union rate using Dall-Miles cables. Arch Orthop Trauma Surg
2002;122:557-560.

AL-Madena Copy

72
AAOS
A MERICAN A CADEMY OF ORTHOPAEDIC SURGEONS

73
Your Source for Lifelong Orthopaedic Learning

74
2010 Sports Medicine Examination Answer Book • 7

Question 1
What is the most common maxillofacial/dental injury in ice hockey?

1. Temporomandibular
2. contusion
Lip laceration
3. Tooth avulsion
4. Crown fracture
5. Mandible fracture

PREFERRED RESPONSE: 4

DISCUSSION: Lahti and associates reported that the most common dental injury in a study of 479
injured ice hockey players was a noncomplicated crown fracture, which accounted for 43.5% of all
maxillofacial/ dental injuries. The most common cause of injury was a blow from an ice hockey stick. As
a cause of injury, the stick was approximately three times as common in games as in training, and only
10% of injured players wore some sort of protective guard. A tooth avulsion is a partial or complete
displacement of the tooth from aleveolar support. A crown fracture is an incomplete loss or fracture of the
tooth enamel without loss of the tooth. The other injuries (mandible fracture, lip laceration, tooth
avulsion, and temporomandibular contusion) occur but are not nearly as common.

REFERENCES: Lahti H, Sane J, Ylipaavalniemi P: Dental injuries in ice hockey games and training.
Med Sci Sports Exerc 2002;34:400-402.
Minkoff J, Stecker S, Varlotta GP, et al: Ice hockey, in Fu FH, Stone DA (eds): Sports Injuries, ed 2.
Philadelphia, PA, 2001, pp 516-517.

Figure 2a Figure 2b Figure 2c


Question 2
The MRI scans and diagnostic ultrasound shown in Figures 2a through 2c show what pathologic condition?

1. Articular-sided supraspinatus tendon tear


2. Bursal-sided supraspinatus tear
3. Superior labral tear
4. Humeral avulsion of the anterior glenoid ligament
5. Avulsion of the anterior inferior glenohumeral ligament
DISCUSSION: The MRI scans and ultrasound show an articular surface partial-thickness rotator cuff tear
of the supraspinatus tendon. This condition most commonly involves the supraspinatus tendon and is

PREFERRED RESPONSE: 1

75
8 • American Academy of Orthopaedic S urgeons

usually found on the articular surface where the blood supply is less robust. There are multiple intrinsic
and extrinsic factors contributing to this condition which include age-related metabolic and vascular
changes that lead to degenerative tearing, subacromial impingement, shoulder instability (typically
anterior), internal impingement, and repetitive microtrauma. Acute trauma is less often the cause. The
physical examination for this condition is often nonspecific and requires supplemental imaging studies for
diagnosis.

REFERENCES: Matava MJ, Purcell DB, Rudzki JR: Partial-thickness rotator cuff tears. Am J Sports Med
2005;33:1405-1417.
Wright SA, Cofield RH: Management of partial-thickness rotator cuff tears. J Shoulder Elbow Surg
1996;5:458-466.
McConville OR, Iannotti JP: Partial-thickness tears of the rotator cuff: Evaluation and management. J Am
Acad Orthop Surg 1999;7:32-43.

Question 3
Which of the following statements best describes the anatomy of the sartorial branch of the saphenous
nerve during medial meniscal repair?

1. The nerve is reliably extrafascial at the joint line.


2. The nerve is anterior to the sartorius.
3. The nerve becomes extrafascial between the gracilis and the semitendinosus.
4. The nerve is anterior to the semitendinosus with the knee in extension.
5. The sartorial branch exits the adductor canal and travels to the anteromedial aspect of the knee.

PREFERRED RESPONSE: 4

DISCUSSION: Dunaway and associates reported that the nerve was extrafascial in only 43% of their
cadaveric specimens. Therefore, in medial meniscal repair, the nerve may be present during deep
dissection. The sartorial branch of the saphenous nerve is posterior to the sartorius; dissection should
remain anterior to the sartorius. The branch becomes extrafascial between the gracilis and the
sartorius. The nerve is anterior to the semitendinosus with the knee in extension. The infrapatellar
branch of the saphenous nerve exits the adductor canal and travels to the anteromedial aspect of the
knee.

REFERENCES: Dunaway DJ, Steensen RN, Wiand W, et al: The sartorial branch of the saphenous nerve:
Its anatomy at the joint line of the knee. Arthroscopy 2005;21:547-551.
Rodeo SA: Arthroscopic meniscal repair with use of the outside-in technique. Instr Course Lect
2000;49:195-206.
Question 4
What portion of the pitching phase creates forces approaching the tensile limit of the medial collateral
ligament?

1. Early cocking phase

76
2010 Sports Medicine Examination Answer Book • 9

2. Late cocking phase


3. Early acceleration phase
4. Follow-through phase
5. Deceleration phase
PREFERRED RESPONSE: 2

DISCUSSION: The late cocking phase of the overhand throw places a marked valgus moment across the
medial elbow. This repetitive force reaches the tensile limits of the medial collateral ligament.

REFERENCES: Fleisig GS, Andrews JR, Dillman CJ, et al: Kinetics of baseball pitching with implications
about injury mechanisms. Am J Sports Med 1995;23:233-239.
Lynch JR, Waitayawinyu T, Hanel DP, et al: Medial collateral ligament injury in the overhand-throwing
athlete. J Hand Surg 2008;33:430-437.

Figure 5a Figure 5b

Question 5
Figures 5a and 5b show the radiographs of a 21 -year-old wrestler who reports that his leg was rolled over
while wrestling. The patient has decreased sensation and function in the distribution of the peroneal nerve,
and he has absent pulses. What is the most appropriate initial management at this time?

1. Acute reconstruction of all ligamentous structures


2. Emergency MRI and reconstruction of all ligamentous structures
3. Emergency arteriogram followed by MRI
4. Emergency surgery with open reduction and repair of all tom structures with vascular surgery
available
5. Closed reduction in the emergency room and reevaluation of the vascular status

PREFERRED RESPONSE: 5

DISCUSSION: The patient has an acute traumatic anteromedial dislocation of the knee with occlusion of
the popliteal artery with a possible tear. Treatment should include reduction and reevaluation of the
vascular status. At this time, if pulses are symmetric, observation may be appropriate without surgical

77
10 • American Academy of Orthopaedic S urgeons

intervention of the artery, but documentation with studies would be appropriate. Delayed reconstruction of
injured structures is appropriate.

REFERENCES: Fanelli GC, Orcutt DR, Edson CJ: The multiple- ligament injured knee: Evaluation,
treatment, and results. Arthroscopy 2005;21:471 -486.
McDonough EB Jr, Wojtys EM: Multiligamentous injuries of the knee and associated vascular injuries.
Am J Sports Med 2009;37:156-159.
Wascher DC: High-velocity knee dislocation with vascular injury: Treatment principles. Clin Sports Med
2000;19:457-477.
Question 6
A 45-year-old distance runner has a hyaluronic acid injection to his knee because of degenerative arthritis.
He immediately develops a severe rash and a systemic hypersensitivity reaction. This patient most likely is
also allergic to which of the following?

1. Penicillin
2. Sulfur
3. Shellfish
4. Chicken or eggs
5. Lidocaine

PREFERRED RESPONSE: 4

DISCUSSION: Preparations of hyaluronic acid can be divided into low and high molecular weight
compounds. Contraindications to intra-articular hyaluronic acid include joint or skin infection, overlying
skin disease, and allergies to chicken or egg products if using a preparation derived from rooster comb.

REFERENCES: Gloyscen DN, Gillespie MJ, Schenek RC: The effects of medication in sports injuries, in
DeLee JC, Drez D Jr, Miller MD (eds): Orthopedic Sports Medicine: Principles and Practice, ed 2.
Philadelphia, PA, WB Saunders, 2003, vol 1, pp 121-124.
Schenck RC Jr: New approaches to the treatment of osteoarthritis: Oral glucosamine and chondroitin
sulfate. Instr Course Lect 2000;49:491-494.

Question 7
A 27-year-old male competitive soccer player reports a 1-year history of pain in the adductor region that
has prevented him from playing. Examination reveals tenderness about the adductor attachment to the
pelvis, and pain at the same site with resisted contraction of the adductors. There is no tenderness over the
hip joint and no signs of a sports hernia. Radiographs are normal. MRI does not show any evidence of
enthesopathy. What is the next best step in management?

1. Hip arthroscopy
2. Corticosteroid injection
3. Percutaneous adductor tenotomy

78
2010 Sports Medicine Examination Answer Book • 11

4. Bone scan
5. Rheumatology consultation

PREFERRED RESPONSE: 2
DISCUSSION: Schilders and associates reported their results of treating adductor-related groin pain in
competitive athletes. They reported that a single corticosteroid injection into the pubic cleft can be
expected to provide at least 1 year of relief of adductor-related groin pain in a competitive athlete with
normal findings on MRI. In contrast, when there is evidence of enthesopathy on MRI in this competitive-
athlete population, these injections are not therapeutic and are associated with a high likelihood of
recurrence of symptoms. Hip arthroscopy is generally reserved for intra-articular problems. Percutaneous
adductor tenotomy is not indicated for this condition. A bone scan is unlikely to provide any useful
information for clinical decision-making. Rheumatology consultation is also not indicated in the absence
of any evidence of inflammatory arthropathy.

REFERENCES: Schilders E, Bismil Q, Robinson P, et al: Adductor-related groin pain in competitive


athletes: Role of adductor enthesis, magnetic resonance imaging, and entheseal pubic cleft injections. J
Bone Joint Surg Am 2007;89:2173-2178.
Robinson P, Barron DA, Parsons W, et al: Adductor-related groin pain in athletes: Correlation of MR
imaging with clinical findings. Skelet Radiol 2004;33:451-457.

Question 8
A 23-year-old national team rower reports pain over the radial dorsum of the forearm that is made worse
with flexion and extension of the wrist during competition. His primary physician initially diagnosed de
Quervain’s tenosynovitis, and a subsequent corticosteroid injection into the first dorsal compartment at
the wrist provided no relief. The patient continues to report pain and audible crepitus that is noted 5 cm
proximal to the wrist joint, on the radial aspect. What structures are involved in the continued pathology?

1. Abductor pollicis brevis and extensor pollicis brevis


2. Abductor pollicis brevis and extensor pollicis longus
3. Abductor pollicis longus and extensor pollicis brevis
4. Abductor pollicis longus and extensor pollicis longus
5. Adductor pollicis and extensor pollicis longus

PREFERRED RESPONSE: 3

DISCUSSION: Intersection syndrome is also known as “squeakers wrist,” “oarsmen wrist,” and
crossover tendinitis. It occurs where the first and second dorsal wrist compartment structures pass over
one another, resulting in fibrosis, muscular changes, and inflammation of the bursa in this area. The
structures involved are the abductor pollicis longus and extensor pollicis brevis (first dorsal compartment)
that pass across the second compartment structures (extensor carpi radialis brevis and extensor carpi
radialis longus). An audible “squeak” is occasionally heard at the intersection point, which is
approximately 4 to 5 cm proximal to the proximal dorsal wrist crease.

79
12 • American Academy of Orthopaedic S urgeons

REFERENCES: Grundberg AB, Reagan DS: Pathologic anatomy of the forearm: Intersection syndrome.
J Hand Surg Am 1985; 10:299-302.
Thorson E, Szabo RM: Common tendinitis problems in the hand and forearm. Orthop Clin North Am
1992;23:65-74.
Williams JG: Surgical management of traumatic non-infective tenosynovitis of the wrist extensors. J
Bone Joint Surg Br 1977;59:408-410.
Wood MB, Dobyns JH: Sports-related extraarticular wrist syndromes. Clin Orthop Relat Res
1986;202:93-102.

Question 9
If the quality of the tendon is poor at the lateral attachment of a partial articular side rotator cuff tear
(more than 6 mm of footprint exposure or greater than 50% thickness), what should the surgeon do?

1. Use an autogenous fascial graft.


2. Use an allograft augmentation.
3. Complete the tear and then repair the tendon.
4. Perform a trans-tendon repair.
5. Biopsy the tissue.

PREFERRED RESPONSE: 3

DISCUSSION: Generally, partial articular side rotator cuff tears are treated by either debridement or
repair. The decision to repair depends on the “thickness” of the tear and the retraction of the undersurface
of the rotator cuff as well as the quality of the remaining tissue. More than 6 mm of footprint exposure
suggests a 50% thicknes tear. If it is poor quality as in this case, the surgeon should complete the tear and
repair the tendon as in a small full-thickness tear. Intrasubstance tears with an intact footprint can be
treated with trans-tendon repair.

REFERENCES: Wolff AB, Sethi P, Sutton KM, et al: Partial thickness rotator cuff tears. J Am Acad Or-
thop Surg 2006;14:715-725.
Mazzocca AD, Rincon LM, O’Connor RW, et al: Intra-articular partial-thickness rotator cuff tears:
Analysis of injured and repaired strain behavior. Am J Sports Med 2008:36:110-116.

80
13 • American Academy of Orthopaedic Surgeons

Question 10
A 32-year-old man underwent a total medial meniscectomy 2 years ago. He now reports pain and
recurrent swelling for the past 3 months. Work-up includes full standing hip-knee-ankle radiographs,
standing AP radiographs of both knees in full extension, an axial view of the patellofemoral joint, and a
45-degree flexion AP radiograph. Contraindication to meniscus allograft transplantation includes which
of the following?

1. 4 mm of tibiofemoral joint space on a 45-degree weight-bearing AP radiograph


2. Intact anterior cruciate ligament on MRI and physical examination
3. Recurrent effusions
4. Flattening of the femoral condyles
5. Healed high tibial osteotomy

PREFERRED RESPONSE: 4

DISCUSSION: Flattening of the femoral condyles indicates the onset of significant arthritis of the joint
and is a contraindication to meniscus allograft transplantation. Criteria to proceed with allograft
transplantation includes prior total meniscectomy, age of 50 years or younger, BMI of less than 30,
clinical symptoms of pain in the involved tibiofemoral compartment, 2 mm or more of tibiofemoral joint
space on a 45-degree weight-bearing AP radiograph, ligamentous stability, normal alignment, and no
radiographic evidence of advanced arthrosis. Recurrent effusions are associated with chronic meniscus
deficiency, and is one criteria for meniscal transplantation. High tibial osteotomy is often considered in
conjunction with meniscal transplantation to correct tibiofemoral malalignment.

REFERENCES: Noyes FR, Barber-Westin SD: Meniscus transplantation: Indications, techniques,


clinical outcomes. Instr Course Lect 2005;54:341-353.
Kang RW, Lattermann C, Cole BJ: Allograft meniscus transplantation: Background, indications,
techniques, and outcomes. J Knee Surg 2006;19:220-230.

Question 11
A college athlete on a scholarship has a medical condition that you feel presents a life-threatening risk to
him with participation in athletics. Because of the gravity of this decision and the potential effect it can
have on the student/athlete’s future, the college asks for your guidance. As the team physician for the
college, what is your ethical obligation?

1. Ban the athlete from sports participation.


2.
Allow the athlete to participate as it is his constitutional right to do so.
Advise the college to revoke the athlete’s college scholarship.
3.
4.
Offer no opinion as it is a matter strictly between the college and the athlete.
5.
Recuse yourself from all decision making and advise the athlete to get an opinion from a
third- party physician who is not employed by the college or university.
DISCUSSION: There is legal precedent for banning a scholarship athlete from participation in college

PREFERRED RESPONSE: 1

81
14 • American Academy of Orthopaedic S urgeons

athletics if the physician feels that it presents a significant physical risk to the athlete. The courts have
decided that the athlete has no constitutional right to participate in NCAA sports, and as a team physician
you must advise your athlete and the school as to the best course of action. The athlete must be allowed
to keep his or her college scholarship.

REFERENCES: Klossner D: NCAA Sports Medicine Handbook, ed 18. Indianapolis, IN, National
Collegiate Athletic Association, 2007.
Pearsall AW IV, Kovaleski JE, Madanagopal SG: Medicolegal issues affecting sports medicine
practitioners. Clin Orthop Relat Res 2005;433:50-57.

Question 12
A 16-year-old female gymnast reports a 2-month history of back pain since falling off the parallel bars,
and she has been unable to return to gymnastics. She has no numbness or tingling. Examination reveals
lower back tenderness, some paravertebral muscle spasm, range of motion of the lumbosacral spine is 20
degrees of flexion and 20 degrees of extension, and an equivocal straight leg raise. Lumbosacral spine
radiographs demonstrate Schomorl’s nodes but no evidence of spondylolisthesis. What is the next best
step in management?

1. Bone scan
2. MRI
3. Flexion-extension radiographs
4. Physical therapy
5. Lumbosacral corset
PREFERRED RESPONSE: 2

DISCUSSION: Injuries to the anterior and middle column in gymnasts occur but are far less common
than posterior column injuries such as spondylolysis and spondylolisthesis. The data on injuries to the
anterior and middle columns are more limited. Long-term gymnastics exercise is associated with disk
degeneration and other anterior and middle column abnormalities as reported by Katz and Scerpella.
They identified a series of anterior and middle column abnormalities, including vertebral compression
fractures, Schmorl’s nodes, disk degeneration, and disk herniation in young competitive female gymnasts
with back pain. Therefore, the differential diagnosis of back pain in these athletes should include
abnormalities of the anterior and middle column. Although diagnostic imaging should begin with
radiographs, MRI is the best way to diagnosis these abnormalities. A bone scan is more useful for
imaging bony abnormalities of the posterior elements. Flexion-extension radiographs are not indicated in
this patient. Treatment such as physical therapy or a lumbosacral corset should not be initiated prior to a
complete work-up.

REFERENCES: Katz DA, Scerpella TA: Anterior and middle column thoracolumbar spine injuries in
young female gymnasts: Report of seven cases and review of the literature. Am J Sports Med
2003;31:611-616.
Tertti M, Paajanen H, Kujala UM, et al: Disc degeneration in young gymnasts: A magnetic resonance
imaging study. Am J Sports Med 1990;18:206-208.

82
2010 Sports Medicine Examination Answer Book • 15

Question 13
A 19-year-old linebacker for a
collegiate football team
has had two episodes of
bilateral arm tingling and
weakness after tackling; the
symptoms resolved after
30 minutes of rest. Three
follow-up neurologic
examinations have been
normal. Cervical spine
CT and MRI scans Figure 13a are Figure 13b shown in Figure 13c Figures 13a through
13c. What is the next best step in management?

1. The addition of a neck roll to the helmet and continuation of play


2. Electrodiagnostic studies
3. A series of epidural steroid injections, followed by a return to play
4. Methylprednisolone dose pack, followed by a return to play in 1 week
5. No further participation in football

PREFERRED RESPONSE: 5

DISCUSSION: Cervical spinal stenosis is a contraindication to participation in collision and contact


sports. Previously, the risks of permanent quadriparesis from cervical spinal stenosis were thought to be
unclear and athletes with cervical spinal stenosis were often allowed to play contact sports. In 1996, Torg
and associates reported that developmental narrowing of the cervical canal in a stable patient does not
appear to predispose an individual to permanent catastrophic neurologic injury and therefore should not
preclude an athlete from participation in contact sports. However, the current understanding is that the
actual risks of permanent neurologic injury from cervical stenosis are significant. The Torg ratio was
previously used for diagnosis but is more recently thought to be of low predictive value as reported by
Cantu. Current methods for diagnosis of cervical spinal stenosis rely on MRI and CT. Current diagnosis
is based on comparisons of measurements with normal values. A cervical canal of less than 13 mm is
considered stenotic whereas a diameter of less than 10 mm is considered absolute stenosis as reported by
Crowl and Kong. This patient has symptomatic stenosis and should not be cleared for contact sports. A
neck roll will not prevent neurologic injury in the presence of cervical spinal stenosis. Electrodiagnostic
studies are not likely to add any additional significant findings with central canal stenosis. Cervical
traction is not of value in the long-term. Epidural steroid injections or a methylprednisolone dose pack
are not of value in this situation.

83
16 • American Academy of Orthopaedic S urgeons

REFERENCES: Torg JS, Naranja RJ Jr, Pavlov H, et al: The relationship of developmental narrowing
of the cervical spinal canal to reversible and irreversible injury of the cervical spinal cord in football
players. J Bone Joint Surg Am 1996;78:1308-1314.
Cantu RC: The cervical spinal stenosis controversy. Clin Sports Med 1998;17:121-126.
Crowl AC, Kong JF: Cervical Spine, in Johnson DL, Mair SD (eds): Clinical Sports Medicine.
Philadelphia, PA, Mosby Elsevier, 2006, pp 143-149.

Question 14
A 24-year-old professional male soccer player has lower abdominal pain on exertion. He has pain with
resisted hip adduction and with sit-ups. There is no palpable inguinal hernia with a Valsalva maneuver.
Nonsurgical management has failed to provide relief. After ruling out malignancies, what is the next
most appropriate step in management?

1. Additional nonsurgical management


2. Referral to a general surgeon
3. Ultrasound of the scrotum
4. CT of the pelvis
5. Cortisone injection

PREFERRED RESPONSE: 2

DISCUSSION: The patient has a sports hernia or athletic pubalgia. The exact nature of this pathology is
not well understood. MRI scans are not very helpful in making a diagnosis. In high-performance athletes
who have failed to respond to nonsurgical management, surgical intervention is needed to strengthen the
anterior pelvic floor. Additional nonsurgical management at this point will not improve symptoms.
Cortisone injection will not strengthen the pelvic floor. CT scan of the pelvis is valuable to rule out bony
injuries such as osteitis pubis. Ultrasound of the scrotum will have no additional diagnostic use in
management of this patient.

REFERENCES: Meyers WC, Foley DP, Garrett WE, et al: Management of severe lower abdominal or
inguinal pain in high-performance athletes: PAIN (Performing Athletes with Abdominal or Inguinal
Neuromuscular Pain Study Group). Am J Sports Med 2000;28:2-8.
Biedert RM, Wamke K, Meyer S: Symphysis syndrome in athletes: Surgical treatment for chronic lower
abdominal, groin, and adductor pain in athletes. Clin J Sport Med 2003;13:278-284.
Question 15
After normal menses has begun and in the absence of pregnancy, secondary amenorrhea is defined as
which of the following?

1. Increase in menstrual volume for 3 months


2. Absence of menstrual bleeding for 6 months
3. Excessive pain during menstrual bleeding for 2 months
4. Absence of menstrual bleeding in the month following peak training intensity
5. Decrease in menstrual volume for 2 consecutive months

PREFERRED RESPONSE: 2

84
2010 Sports Medicine Examination Answer Book • 17

DISCUSSION: Secondary amenorrhea is defined as the absence of menstrual bleeding for 6 months or the
absence of three to six consecutive menstrual cycles after normal menses has begun. The prevalence of
amenorrhea among female athletes is estimated at 10% to 20% in women who exercise vigorously and as
high as 40% to 66% in elite runners and professional ballet dancers.

REFERENCES: Garrick JG (ed): Orthopaedic Knowledge Update: Sports Medicine 3. Rosemont, IL,
American Academy of Orthopaedic Surgeons, 2004, pp 345-346.
Feingold D, Hame SL: Female athlete triad and stress fractures. Orthop Clin North Am 2006;37:575-583.

Question 16
A 17-year-old male football player is seen 1 week after developing symptoms of infectious mononucleosis
in the middle of the season. Examination reveals evidence of splenomegaly. He and his parents want to
know if he can play in a game the following day. What is the most appropriate recommendation?

1. It is safe to return to play right away.


2. It is safe to return to play if there is no airway obstruction from adenopathy.
3. It is safe to play due to the low risk of disease transmission to other players.
4. It is safe to play after 3 to 4 weeks.
5. It is unsafe to play the next season.

PREFERRED RESPONSE: 4

DISCUSSION: Infectious mononucleosis (IMN) is a self-limiting viral (Epstein-Barr virus) infection that
affects mostly adolescents. One of the clinical findings in IMN is splenomegaly. Unfortunately, the
splenomegaly is palpable only 50% of the time. The risk for spontaneous splenic rupture is highest 3
weeks after the onset of symptoms. Thus, most clinicians recommend return to contact sports after 4
weeks from the onset of symptoms. This patient presented 1 week after the onset of symptoms, so he can
return to play in 3-4 weeks from the time he was examined. The athlete should be afebrile, well hydrated,
and asymptomatic. Airway obstruction is usually not of concern. Disease transmission to teammates is
possible in the acute phases.
REFERENCES: Waninger KD, Harcke HT: Determination of safe return to play for athletes recovering
from infectious mononucleosis: A review of the literature. Clin J Sport Med 2005; 15:410-416.
Auwaerter PG: Infectious mononucleosis: Return to play. Clin Sports Med 2004;23:485-497.

Figure 17

85
18 • American Academy of Orthopaedic S urgeons

Question 17
A loose body is encountered during a left knee arthroscopy in the posterolateral compartment. In the
arthroscopic photograph shown in Figure 17, the posterior aspect of the lateral femoral condyle is shown
on the right and the posterolateral capsule is shown on the left. The arthroscope is placed in what
anatomic interval to visualize this loose body?

1. Between the anterior cruciate ligament (ACL) and the posterior cruciate ligament (PCL)
2. Between the ACL and the lateral femoral condyle
3. Between the PCL and the medial femoral condyle
4. Between the lateral collateral ligament (LCL) and the lateral femoral condyle
5. Between the medial collateral ligament (MCL) and the medial femoral condyle

PREFERRED RESPONSE: 2

DISCUSSION: The arthroscopic photo shows a grasper removing a loose body from the posterolateral
compartment through an accessory posterolateral portal. The blunt arthroscopic trocar is placed through
the intercondylar notch in the direction of the posterior horn of the lateral meniscus. The trocar passes
between the ACL and the posterior aspect of the lateral femoral condyle into the posterolateral
compartment.

REFERENCES: Wu WH, Richmond JC: Arthroscopy of the knee: Basic setup and techniques, in
McGinty JB (ed): Operative Arthroscopy, ed 3. Philadelphia, PA, Lippincott-Raven, 2003, pp 215-216.
Kramer DE, Bahk MS, et al: Posterior knee arthroscopy: Anatomy, technique, application. J Bone Joint
Surg Am 2006;88:110-121.
Question 18
Intramedullary screw fixation of a Jones fracture is a successful approach for the treatment of a difficult
proximal fifth metatarsal fracture. Recent studies suggest a statistically higher proportion of treatment
failures in which of the following?

1. Patients older than 40 years of age


2. Female patients
3. Patients with screw diameters of greater than 4.5 mm
4. Early unprotected weight bearing in athletes
5. Patients who did not undergo bone grafting
PREFERRED RESPONSE: 4

DISCUSSION: Proximal fifth metatarsal Jones fractures can be successfully treated by intramedullary
screw fixation. This technique may stimulate an early healing response when compared to nonsurgical
management. When studies were done to look at the failure rate in patients undergoing this procedure, elite
athletes appeared to have the highest rate of failure. This most likely does not represent a failure in the
surgical procedure as much as it is a failure in the treatment of the patient. If an athlete is allowed to return
back to playing their sport before the fracture is healed, they are putting themselves at risk for hardware
failure. Bone grafting has not been shown to necessarily accelerate the healing process in these patients.

REFERENCES: Larson CM, Almekinders LC, Taft TN, et al: Intramedullary screw fixation of Jones
fractures: Analysis of failure. Am J Sports Med 2002;30:55-60.
Glasgow MT, Naranja RJ Jr, Glasgow SG: Analysis of failed surgical management of fractures of the base of

86
2010 Sports Medicine Examination Answer Book • 19

the fifth metatarsal distal to the tuberosity: The Jones fracture. Foot Ankle Int 1996;17:449-457.

Figure 19a Figure 19b Figure 19c

Question 19
Which of the following clinical findings is most often seen with the MRI scan findings shown in Figures 19a
through 19c?

1. Atrophy of the lateral shoulder


2. Atrophy of the posterior shoulder
3. Sensory deficit of the lateral shoulder
4. Sensory deficit of the posterior shoulder
5. Sensory deficit of the anterior shoulder

PREFERRED RESPONSE: 2

DISCUSSION: The MRI scans show a large superior labral cyst. Impingement of the cyst on the
suprascapular nerve is implied by the visible atrophy of the infraspinatus muscle as seen in Figure 19c.
Clinically, this is manifested by atrophy of the posterior aspect of the shoulder inferior to the scapular spine.
The suprascapular nerve provides only motor function and does not provide any sensory function to the
shoulder girdle; therefore, sensory deficits will not be present in this patient.

REFERENCES: Westerheide KJ, Dopirak RM, Karzel RP, et al: Suprascapular nerve palsy secondary to
spinoglenoid cysts: Results of arthroscopic treatment. Arthroscopy 2006;22:721-727.
Schroder CP, Skare O, Stiris M, et al: Treatment of labral tears with associated spinoglenoid cysts without
cyst decompression. J Bone Joint Surg Am 2008;90:523-530.
Piatt BE, Hawkins RJ, Fritz RC, et al: Clinical evaluation and treatment of spinoglenoid notch ganglion
cysts. J Shoulder Elbow Surg 2002; 11:600-604.
Question 20
A 23-year-old right-hand dominant professional baseball pitcher has right shoulder pain when releasing the
ball. He has noticed his velocity has decreased over the past 2 months. Examination reveals supine abducted
external rotation of 110 degrees compared to 100 degrees on the left side. His internal rotation is 30 degrees on
the right compared to 70 degrees on the left side. Rotator cuff strength is normal. All other clinical tests are
normal. MRI with contrast reveals no intra-articular lesions. What is the best course of treatment?

1. Arthroscopic capsular plication


2. Arthroscopic thermal shift
3. Arthroscopic subacromial decompression
4. Posterior capsular stretching
5. Selective external rotation stretching

87
20 • American Academy of Orthopaedic S urgeons

PREFERRED RESPONSE: 4

DISCUSSION: The examination reveals that the patient has posterior capsular tightness. Surgery should not
be considered until the patient has failed to respond to nonsurgical management. The internal rotation
contracture (GIRD - glenohumeral internal rotation deficit) should be addressed with appropriate posterior
capsular stretching. This should then be followed by appropriate rotator cuff and scapular stabilization
exercises. Only if this management fails to relieve the patient’s symptoms should surgery be considered.
This patient clearly does not need external rotation stretching given the fact that he has normal external
rotation.

REFERENCES: Meister K: Injuries to the shoulder in the throwing athlete. Part two: evaluation/ treatment.
Am J Sports Med 2000;28:587-601.
Liu SH, Boynton E: Posterior superior impingement of the rotator cuff on the glenoid rim as a cause of
shoulder pain in the overhead athlete. Arthroscopy 1993;9:697-699.
Tyler TF, Nicholas SJ, Roy T, et al: Quantification of posterior capsule tightness and motion loss in patients
with shoulder impingement. Am J Sports Med 2000;28:668-673.

88
2010 Sports Medicine Examination Answer Book • 21

Question 21
A 23-year-old woman has had a 3-year history of snapping and pain in her left hip. She notes that the
snapping started while marathon training and is only problematic about 15 minutes into a run. Examination is
consistent with a negative Stinchfield, negative logroll, negative flexion abduction/external rotation test
(FABER) of the hip; however, she has a positive Ober test as she has difficulty adducting her hip across the
midline in the lateral decubitus position. Management consisting of nonsteroidal antiinflammatory drugs and
stretching has failed to improve her snapping. What is the most reliable surgical treatment?

1. Hip arthroscopy with labral debridement


2. Hip arthroscopy with femoral acetabular impingement lesion debridement
3. Release of the iliopsoas tendon
4. Z-plasty of the iliotibial band
5. Release of the iliotibial band at Gerdy’s tubercle

PREFERRED RESPONSE: 4

DISCUSSION: The patient has external-type snapping hip (coxa saltans). It is not uncommon for patients to
have a very long duration of symptoms that limit running or other sporting activities, and commonly affects
the downward leg (usually the left leg when running on the left side of the road). The snapping causes a
profound bursitis at the greater trochanter, and occasionally corticosteroid injections may be helpful. Her
physical examination does not suggest an intra-articular process, and is not consistent with an internal-type
snapping hip, usually caused by the iliopsoas tendon as it moves over the iliopectineal eminence. Stretching is
the mainstay of treatment, as testing with a positive Ober signifies a tight iliotibial band as the thigh has
difficulty crossing the midline with adduction. Various iliotibial band lengthening procedures have been
described, including a Z-plasty near the proximal origin of the iliotibial band. Release at Gerdy’s tubercle has
not been described.

REFERENCES: Provencher MT, Hofmeister EP, Muldoon MP: The surgical treatment of external coxa
saltans (the snapping hip) by Z-plasty of the iliotibial band. Am J Sports Med 2004;32:470-476.
Faraj A A, Moulton A, Sirivastava VM: Snapping iliotibial band: Report of ten cases and review of the
literature. Acta Orthop Belg 2001;67:19-23.
Brignall CG, Stainsby GD: The snapping hip: Treatment by Z-plasty. J Bone Joint Surg Br 1991 ;73:253-
254.
Question 22
What structure is the primary restraint to inferior translation of the shoulder?

1. Middle glenohumeral ligament

89
22 • American Academy of Orthopaedic Surgeons

2. Subscapularis
3. Long head of the biceps
4. Coracohumeral ligament
5. Coracoacromial ligament

PREFERRED RESPONSE: 4

DISCUSSION: The coracohumeral ligament has been shown to be the primary restraint to inferior
translation of the shoulder. Although Bigliani and associates have demonstrated that the inferior capsule and
inferior glenohumeral ligaments also play a role, none of the other choices provide primary inferior stability
of the shoulder. The coracohumeral ligament is an important structure of the rotator interval of the shoulder
(the rotator interval contains the long head of the biceps, the superior glenohumeral ligament, the
coracohumeral ligament, and a thin layer of capsule). Harryman and associates demonstrated that an open
rotator interval closure via imbrication of the coracohumeral ligament improves inferior stability of the
glenohumeral joint.

REFERENCES: Harryman DTII, Sidles JA, Harris SL, et al: The role of the rotator interval capsule in
passive motion and stability of the shoulder. J Bone Joint Surg Am 1992;74:53-66.
Bigliani LU, Pollock RG, Soslowsky LJ, et al: Tensile properties of the inferior glenohumeral ligament. J
Orthop Res 1992;10:187-197.
Boardman ND, Debski RE, Warner JJ, et al: Tensile properties of the superior glenohumeral and
coracohumeral ligaments. J Shoulder Elbow Surg 1996;5:249-254.

Question 23
A collegiate division I football player ruptures his anterior cruciate ligament (ACL). After counseling
him, you agree to perform a double-bundle ACL reconstruction. Which of the following is a correct
statement for this technique?

1. The anteromedial (AM) bundle limits translation and the posterolateral (PL) bundle controls
rotation.
2. The PL bundle limits translation and the AM bundle controls rotation.
3. The anterolateral (AL) bundle limits translation and the posteromedial (PM) bundle controls
rotation.
4. Both the AL and the PM control rotation equally.
5. The AL bundle controls rotation and the PM bundle limits translation.
DISCUSSION: The ACL is composed of two anatomic bundles: the anteromedial (AM) and the
posterolateral (PL). They are both considered important to the stability of the knee. Although they work in
concert, the AM bundle controls translation, especially in flexion, whereas the PL bundle prevents rotation.

PREFERRED RESPONSE: 1

90
2010 Sports Medicine Examination Answer Book • 23

REFERENCES: Zelle BA, Vidal AF, Brucker PU, et al: Double-bundle reconstruction of the anterior
cruciate ligament: Anatomic and biomechanical rationale. J Am Acad Orthop Surg 2007;15:87-96.
Yasuda K, Ichiyama H, Kondo E, et al: An in vivo biomechanical study on the tension-versus-knee flexion
angle curves of 2 grafts in anatomic double-bundle anterior cruciate ligament reconstruction: Effects of
initial tension and internal tibial rotation. Arthroscopy 2008;24:276-284.

Question 24
Which of the following is the only nonreversible effect of anabolic steroids?

1. Muscle hypertrophy
2. Alterations in high density lipoprotein (HDL) and low density lipoprotein (LDL) ratios
3. Alopecia
4. Personality effects
5. Acne

PREFERRED RESPONSE: 3

DISCUSSION: The loss of hair or alopecia, is the only nonreversible effect of anabolic steroid use.
Once anabolic steroids are stopped, muscle hypertrophy and training gains are quickly lost and the HDL/
LDL ratios return to their preexisting levels. Fortunately, the personality effects and the acute acne are
reversible.

REFERENCES: Hartgens F, Kuipers H: Effects of androgenic-anabolic steroids in athletes. Sports Med


2004;34:513-554.
Evans NA: Current concepts in anabolic-androgenic steroids. Am J Sports Med 2004;32:534-542.

Question 25
What is the most common complication associated with surgical repair of an Achilles tendon rupture?

1. Infection
2. Neurologic injury
3. Loss of motion
4. Re-rupture
5. Skin healing problems
DISCUSSION: There are many reported advantages to performing surgical repair of an Achilles tendon
rupture. One advantage appears to be the decreased risk of re-rupture, and especially in the elite athlete, the
ability to return that individual to his or her pre-injury status. Clearly surgical intervention is not without
complications. The number one complication with Achilles tendon repair is skin healing problems. Infection is
also common, although it is not the most common problem. Neurologic injury is a rare complication. Re-
rupture and loss of motion are more commonly seen in cases treated without surgery.

91
24 • American Academy of Orthopaedic S urgeons

REFERENCES: Wong J, Barrass V, Maffulli N: Quantitative review of operative and nonoperative


management of Achilles tendon ruptures. Am J Sports Med 2002;30:565-575.
Lo IK, Kirkley A, Nonweiler B, et al: Operative versus nonoperative treatment of acute Achilles tendon
ruptures: A quantitative review. Clin J Sports Med 1997;7:207-211.

Question 26
Internal impingement is characterized by which of the following anatomic lesions?

1. Subscapularis tear
2. Bursal-sided rotator cuff tear
3. Articular-sided rotator cuff tear
4. Tight anterior capsule
5. Laxity of the posterior capsule

PREFERRED RESPONSE: 3

DISCUSSION: Internal impingement is characterized by articular-sided partial-thickness rotator cuff tears


and superior glenoid labral tears. The capsule is characterized by laxity anteriorly and tightness posteriorly.

REFERENCES: Garrick JG (ed): Orthopaedic Knowledge Update: Sports Medicine 3. Rosemont, IL,
American Academy of Orthopaedic Surgeons, 2004, p 82.
Burkhart SS, Morgan CD, Kibler WB: The disabled throwing shoulder: Spectrum of pathology. Part I:
Pathoanatomy and biomechanics. Arthroscopy 2003;19:404-420.

Figure 27

Question 27
The clinical photograph in Figure 27 shows a palsy of what nerve/associated muscle?

1. Long thoracic/rhomboid
2. Long thoracic/serratus anterior
3. Long thoracic/supraspinatus
4. Dorsal scapular/trapezius

92
2010 Sports Medicine Examination Answer Book • 25

5. Spinal accessory/trapezius

PREFERRED RESPONSE: 2

DISCUSSION: The clinical picture reveals medial scapular winging, which involves the serratus anterior
muscle, potentially due to an injury to the long thoracic nerve that innervates this muscle. Injury to the long
thoracic nerve is usually due to closed trauma, direct compression, traction or stretching injury, a direct
blow, or, very rarely, viral infection such as Parsonage-Tumer syndrome. The nerve is easily injured in
surgical dissection of the axilla, and is predisposed to injury due to its relatively long course, it is small in
diameter, and it has little surrounding connective tissue. If rehabilitation and time are unsuccessful, both
nerve and muscle transfers have been described with mixed results.

REFERENCES: Wiater JM, Flatow EL: Long thoracic nerve injury. Clin Orthop Relat Res 1999;368:17 -
27.
Warner JJ, Navarro RA: Serratus anterior dysfunction: Recognition and treatment. Clin Orthop Relat Res
1998;349:139-148.
Question 28
An 18-year-old collegiate football player injures his right shoulder during a tackle. He reports pain and
numbness in the shoulder and numbness radiating to his fingers. His symptoms improve within 15 minutes
and he has no residual symptoms. This condition is best known as

1. acute and transient spinal cord injury.


2. central cord syndrome.
3. nerve root avulsion.
4. Guillain-Barre syndrome.
5. stinger/burner.

PREFERRED RESPONSE: 5

DISCUSSION: The condition described in this case is known as a stinger or burner. It is caused by stretching
the upper trunk of the brachial plexus in the C5 and C6 nerve roots. The symptoms are temporary and last 15
to 20 minutes. There are no residual deficits, unless the patient has had multiple repetitive injuries. Once
motor and sensory examination findings and reflexes have normalized, the athlete can return to play. Acute
spinal cord injury may cause temporary complete paralysis in the upper and lower extremities with resolution
of symptoms within 24 hours. Central cord syndrome affects the upper more than lower extermities and
affects mostly elderly patients. Nerve root avulsions lead to permanent deficits and have a poor prognosis for
return of function. Guillain-BarDe syndrome is an autoimmune disease that presents as an ascending paralysis
with weakness in the legs that spreads to the upper limbs and the face along with complete loss of deep tendon
reflexes.

REFERENCES: Safran MR: Nerve injury about the shoulder in athletes. Part 2: Long thoracic nerve, spinal
accessory nerve, burners/stingers, thoracic outlet syndrome. Am J Sports Med 2004;32:1063-1076. Aval SM,
Durand P Jr, Shankwiler JA: Neurovascular injuries to the athlete’s shoulder: Part I. J Am Acad Orthop Surg
2007;15:249-256.

93
26 • American Academy of Orthopaedic S urgeons

Question 29
A 17-year-old basketball player sustained an inversion twisting injury to the left ankle with the foot plantar
flexed approximately 20 degrees. Which of the following ankle ligaments is most likely to be injured by this
mechanism?

1. Anterior tibiofibular
2. Posterior tibiofibular
3. Anterior talofibular
4. Calcaneofibular
5. Posterior talofibular
DISCUSSION: The lateral ankle is stabilized by the anterior talofibular, calcaneofibular, and posterior
talofibular ligaments. With the ankle in neutral flexion, the anterior talofibular ligament is perpendicular to
the long axis of the tibia. However, with ankle plantar flexion, this ligament becomes more parallel to the
tibia and is at risk with an inversion sprain that most commonly occurs with the ankle in a plantar flexed
position. The calcaneofibular ligament is parallel to the tibia in neutral ankle flexion and is usually injured
when the ankle is inverted in this neutral position. The posterior talofibular ligament is only injured in severe
sprains that also disrupt the other two lateral ligaments. The anterior tibiofibular and posterior tibiofibular
ligaments contribute to the ankle syndesmosis and are most commonly injured with ankle eversion and
external rotation.

REFERENCES: Colville MR, Marder RA, Boyle JJ, et al: Strain measurement in lateral ankle ligaments.
Am J Sports Med 1990;18:196-200.
Casillas M: Ligament injuries of the foot and ankle in adult athletes, in DeLee J, Drez D, Miller M (eds):
DeLee & Drez’s Orthopaedic Sports Medicine: Principles and Practice, ed 2. Philadelphia, PA, WB
Saunders, 2003, vol 2, pp 2323-2376.
Colville MR, Marder RA, Zarins B: Reconstruction of the lateral ankle ligaments: A biomechanical analysis.
Am J Sports Med 1992;20:594-600.

Question 30
In the anterior cruciate ligament-deficient knee, what structure provides an important secondary restraint to
anterior tibial translation?

1. Anterior horn of the lateral meniscus


2. Posterior cruciate ligament
3. Posterior horn of the medial meniscus
4. Popliteus tendon
5. Quadriceps muscle

PREFERRED RESPONSE: 3

94
2010 Sports Medicine Examination Answer Book • 27

DISCUSSION: Cadaveric studies have demonstrated the important role of the posterior horn of the medial
meniscus in stabilizing the anterior cruciate ligament-deficient knee with significantly greater resultant force
in the medial meniscus when subjected to anterior tibial loads. The posterior horn of the medial meniscus is
thought to limit anterior tibial translation by acting as a buttress by wedging against the posterior aspect of
the medial femoral condyle. The other soft tissues mentioned do not play any significant role in prevention
of anterior tibial translation in the anterior cruciate ligament-deficient knee.

REFERENCES: Garrick JG (ed): Orthopaedic Knowledge Update: Sports Medicine 3. Rosemont, IL,
American Academy of Orthopaedic Surgeons, 2004, p 200.
Allen CR, Wong EK, Livesay GA, et al: Importance of the medial meniscus in the anterior cruciate
ligament-deficient knee. J Orthop Res 2000;18:109-115.
Levy IM, Torzilli PA, Warren RF: The effect of medial meniscectomy on anterior-posterior motion of the
knee. J Bone Joint Surg Am 1982;64:883-888.

0 •v. .

jfp
!i
JRT
w

JM

mf
1

w
m
Figure 31
Question 31
A 23-year-old man reports a 6-year history of recurrent instability in the right dominant shoulder. He has
not undergone surgery and has essentially stopped all of his sporting activities. On examination, he has
instability and apprehension in the midrange of motion (abduction of 45 to 60 degrees with external
rotation) and a palpable clunk representing a transient dislocation over the anterior glenoid rim. A three-
dimensional CT scan is shown in Figure 31. What is the most appropriate surgical intervention to provide
him with reliable stability postoperatively?

1. Arthroscopic Bankart surgery


2. Bony glenoid augmentation procedure
3. Subscapularis advancement
4. Open capsular shift
5. Hemiarthroplasty

PREFERRED RESPONSE: 2

DISCUSSION: In the setting of significant anteroinferior glenoid bone deficiency (greater than 20% to
25%), both open and arthroscopic Bankart repairs have demonstrated higher rates of failure. Bony glenoid
augmentation procedures such as the Bristow-Lataijet, which describe coracoid transfers to reconstruct the
deficient glenoid, have led to decreased rates of recurrent shoulder instability. In this scenario, the patient
has a significant loss of glenoid bone. There are also several clues in the history to suspect bone deficiency:
multiple recurrences, a long history of recurrence, and instability in the midranges of motion.

95
28 • American Academy of Orthopaedic S urgeons

A bony augmentation procedure such as the Lataijet has been well-described to provide a well functioning and
stable shoulder joint. A hemiarthroplasty is not indicated in the absence of arthritis. Subscapularis
advancement will not address the bone loss.

REFERENCES: Hovelius L, Sandstrom B, Sundgren K, et al: One hundred eighteen Bristow-Latarjet repairs
for recurrent anterior dislocation of the shoulder prospectively followed for fifteen years: Study I— clinical
results. J Shoulder Elbow Surg 2004;13:509-516.
Schroder DT, Provencher MT, Mologne TS, et al: The modified Bristow procedure for anterior shoulder
instability: 26-year outcomes in Naval Academy midshipmen. Am J Sports Med 2006;34:778-786.
Itoi E, Lee SB, Berglund LJ, et al: The effect of a glenoid defect on anteroinferior stability of the shoulder
after Bankart repair: A cadaveric study. J Bone Joint Surg Am 2000;82:35-46.
Question 32
A 26-year-old man underwent excision of a ganglion cyst of the tibiofibular joint 1 year ago. It has now
recurred and is extremely symptomatic. Nonsurgical management has failed to provide relief. What type of
surgery provides the most predictable results for this patient?

1. Tibiofibular joint fusion


2. Repeat excision
3. Total fibular head excision
4. Interpositional arthroplasty
5. Partial fibular head excision

PREFERRED RESPONSE: 1

DISCUSSION: Recurrence of a ganglion cyst of the tibiofibular joint is most successfully treated by proximal
tibiofibular joint fusion. A repeat excision will most likely result in recurrence of the cyst. Total or partial
excision may lead to instability of the posterolateral structures of the knee. Interpositional arthroplasty has not
been proven to be effective for the treatment of recurrent ganglion cysts of the proximal tibiofibular joint.

REFERENCES: Miskovsky S, Kaeding C, Weis L: Proximal tibiofibular joint ganglion cysts: Excision,
recurrence, and joint arthrodesis. Am J Sports Med 2004;32:1022-1028.
Vatansever A, Bal E, Okcu G: Ganglion cysts of the proximal tibiofibular joint review of literature with three
case reports. Arch Orthop Trauma Surg 2006;126:637-640.

Question 33
A 20-year-old basketball player sustains a knee injury during a game and is seen in the orthopaedic clinic 3
days after injury. Examination reveals a positive Lachman, pivot shift, joint line tenderness, and moderate
effusion. Which of the following tissue injuries is most likely causing the jointline tenderness?

1. Medial meniscus tear


2. Popliteus tendon rupture
3. Lateral meniscus tear
4. Proximal tibia-fibula disruption
5. Pes anserine bursitis

96
2010 Sports Medicine Examination Answer Book • 29

PREFERRED RESPONSE: 3

DISCUSSION: The physical examination findings are consistent with an acute anterior cruciate ligament tear.
In the acute setting, a lateral meniscus tear is a more common secondary injury than a medial meniscus tear.
In one study of acute anterior cruciate ligament tears in alpine skiers, the incidence of lateral meniscus tears
was over four times that of medial meniscus tears. Medial meniscus tears are more common in the chronic
setting, most likely secondary to its role as a secondary restraint.
REFERENCES: Garrick JG (ed): Orthopaedic Knowledge Update: Sports Medicine 3. Rosemont, IL,
American Academy of Orthopaedic Surgeons, 2004, p 201.
Greis PE, Bardana DD, Holmstrom MC, et al: Meniscal injury: I. Basic science and evaluation. J Am Acad
Orthop Surg 2002;10:168-176.
Duncan JB, Hunter R, Purnell M, et al: Meniscal injuries associated with acute anterior cruciate ligament tears
in alpine skiers. Am J Sports Med 1995;23:170-172.

Question 34
Which of the following cohorts of patients is at highest risk of a future anterior cruciate ligament (ACL) tear?

1. Men with a hip abduction moment during landing


2. Men with a neutral hip abduction-adduction moment during landing
3. Men with varus knee abduction moment during landing
4. Women with a hip adduction moment during landing
5. Women with a knee valgus moment during landing

PREFERRED RESPONSE: 5

DISCUSSION: Hewett and associates reported in a study of 205 female athletes that female athletes, with
increased dynamic valgus and high abduction loads, were at increased risk of ACL injury. The same
investigators in an earlier study of 81 high school basketball players reported that female athletes landed with
greater total valgus knee motion and a greater maximum valgus knee angle than male athletes. Female athletes
were also found to have significant differences between their dominant and nondominant side in maximum
valgus knee angle. Lephart and associates reported that in single-leg landing and forward hop tasks that
female athletes had significantly less knee flexion and lower leg internal rotation maximum angular
displacement, and less knee flexion time to maximum angular displacement than males. Females with an
adduction moment during landing should have a lower incidence of ACL tears. Males in general have a lower
incidence of ACL tears.

REFERENCES: Hewett TE, Myer GD, Ford KR, et al: Biomechanical measures of neuromuscular control
and valgus loading of the knee predict anterior cruciate ligament injury risk in female athletes: A prospective
study. Am J Sports Med 2005;33:492-501.
Ford KR, Meyer GD, Hewett TE: Valgus knee motion during landing in high school female and male
basketball players. Med Sci Sports Exerc 2003;35:1745-1750.
Lephart S, Ferris CM, Riemann BL, et al: Gender differences in strength and lower extremity kinematics
during landing. Clin Orthop Relat Res 2002;401:162-169.

97
30 • American Academy of Orthopaedic S urgeons

Fig
ure
35

Question 35
A 36-year-old softball player sustains a shoulder dislocation making a diving catch. The shoulder is
successfully reduced in the emergency department. A postreduction MRI is shown in Figure 35. What
anatomic lesion is a result of the dislocation?

1. Bankart lesion
2. Humeral avulsion of the glenohumeral ligament (HAGL) lesion
3. Superior labrum anterior-posterior (SLAP) lesion
4. Hill-Sach deformity
5. Glenoid fracture (bony Bankart)

PREFERRED RESPONSE: 2

DISCUSSION: The MRI scan reveals a HAGL lesion. It more commonly affects older patients and is
associated with more violent trauma.

REFERENCES: Garrick JG (ed): Orthopaedic Knowledge Update: Sports Medicine 3. Rosemont, IL,
American Academy of Orthopaedic Surgeons, 2004, pp 53-54.
Bokor DJ, Conboy VB, Olson C: Anterior instability of the glenohumeral joint with humeral avulsion of the
glenohumeral ligament: A review of 41 cases. J Bone Joint Surg Br 1999;81:93-96.
Question 36
An 18-year-old woman injures her left knee playing soccer. At the time of anterior cruciate ligament (ACL)
reconstruction, she was noted to have an irreparable posterior horn medial meniscus tear. Partial
meniscectomy will have what primary effect?

1. Increase medial compartment peak loads


2. Increase medial compartment contact area
3. Decrease in situ forces in the ACL graft
4. Decrease anterior tibial translation
5. Increase posterior tibial translation
PREFERRED RESPONSE: 1

DISCUSSION: The medial meniscus distributes force through the medial compartment. Peak loads in the
affected compartment are increased by partial and complete meniscectomy. The posterior horn of the medial
meniscus is also an important secondary restraint to anterior tibial translation in the ACL-deficient knee. In
situ forces in the reconstructed ACL are increased with loss of the posterior horn of the medial meniscus.

REFERENCES: Garrick JG (ed): Orthopaedic Knowledge Update: Sports Medicine 3. Rosemont, IL,
American Academy of Orthopaedic Surgeons, 2004, pp 199-201.

98
2010 Sports Medicine Examination Answer Book • 31

Greis PE, Bardana DD, Holmstrom MC, et al: Meniscal injury: I. Basic science and evaluation. J Am Acad
Orthop Surg 2002; 10:168-176.

Question 37
A 45-year-old coach sustains a complete distal biceps tendon rupture at the elbow. Surgical repair is most
indicated to

1. restore full supination strength.


2. restore full elbow flexion strength.
3. restore full range of motion.
4. improve cosmesis.
5. prevent degenerative changes of the elbow.

PREFERRED RESPONSE: 1

DISCUSSION: The biceps is primarily responsible for supination of the forearm. The brachialis muscle is
primarily repsonsible for elbow flexion strength. Failure to repair the distal biceps tendon will result in loss
of 40% supination strength and 10% loss in flexion strength. Therefore, surgical repair of a complete distal
biceps tendon rupture is most indicated to maximize supination strength. Improved cosmesis should not be
the primary indication for surgical repair. Degenerative changes of the elbow have no bearing on whethe r the
distal biceps is repaired or not. Loss of terminal extension is common in distal biceps tendon repairs.
REFERENCES: Altcheck DW, Altcheck AJ: The Athlete’s Elbow. Philadelphia, PA, Lippincott Williams &
Wilkins, 2001, p 288.
Morrey BF, Askew LJ, An KN, et al: Rupture of the distal tendon of the biceps brachii: A biomechanical study.
J Bone Joint Surg Am 1985;67:418^4-21.

Question 38
The thumb metacarpophalangeal (MCP) joint should be flexed to what degree to properly assess ligamentous
stability?

1. 30 degrees of flexion to test the proper collateral ligament and full extension to test the accessory
collateral ligament and the palmar plate
2. 30 degrees of flexion to test the accessory collateral ligament and full extension to test the
proper collateral ligament and the palmar plate
3. 45 degrees of flexion to test the accessory collateral ligament, the proper collateral ligament, and
the palmar plate
4. 90 degrees of flexion to test the proper collateral ligament and full extension to test the acces
sory collateral ligament and the palmar plate
5. 90 degrees of flexion to test the accessory collateral ligament and full extension to test the
proper collateral ligament and the palmar plate

99
32 • American Academy of Orthopaedic S urgeons

PREFERRED RESPONSE: 1

DISCUSSION: The collateral ligaments of the MCP joint of the thumb can be isolated by flexing the joint to
30 degrees. Full extension is best to assess the accessory collaterals and the palmar plate. The ulnar collateral
ligament nearly always separates from the base of first phalanx of the thumb; it frequently becomes lodged
between adductor pollicis aponeurosis and its normal position (Stener lesion). The creation of a Stener lesion
requires significant radial deviation of the phalanx along with combined tears of the proper and accessory
collateral ligaments in order for the ligament to be displaced above the adductor aponeurosis.

REFERENCES: Koval KJ (ed): Orthopaedic Knowledge Update 7. Rosemont, IL, American Academy of
Orthopaedic Surgery, 2002, pp 339-358.
Stener B: Displacement of the ruptured ulnar collateral ligament of the MP joint of the thumb: A clinical and
anatomical study. J Bone Joint Surg Br 1962;44:869-879.

100
33 • American Academy of Orthopaedic Surgeons

Question 39
Anaerobic weight training has what effect in a prepubescent 10-year-old male athlete?

1. It can induce muscle hypertrophy.


2. It can increase efficiency of muscle action.
3. It has no effect on muscle performance.
4. It can cause injury to the growth plate.
5. It can lead to a higher risk of osteochondritis dissecans.

PREFERRED RESPONSE: 2

DISCUSSION: Although anaerobic weight training in this age group does not lead to muscle hypertrophy, it
can increase the efficiency of muscle action by increasing muscle memory. There is insufficient testosterone
in this patient population to allow for muscle hypertrophy. Proper techniques of weight training have been
shown to be safe and do not damage the growth plates or joints in these individuals.

REFERENCES: Blimkie CJ: Resistance training during preadolescence: Issues and controversies. Sports Med
1993;15:389-407.
American Academy of Pediatrics Council on Sports Medicine and Fitness, McCambridge TM, Strieker PR:
Strength training by children and adolescents. Pediatrics 2008;121:835-840.

Figure 40a Figure 40b Figure 40c

Question 40
What is the predominant type of collagen in the tissue resulting from the surgical procedure shown in
Figures 40a through 40c?

1. Type I
2. Type II
3. Type III
4. Type IX
5. TypeX

PREFERRED RESPONSE: 1

101
34 • American Academy of Orthopaedic S urgeons

DISCUSSION: The arthroscopic images show a microfracture procedure. Perforation of the subchondral
bone results in so-called “marrow stimulation” that results in the formation of fibrocartilage. This reparative
tissue is composed predominantly of type I collagen with a disorganized matrix lacking a true tidemark, as
opposed to hyaline cartilage which is composed primarily of type II collagen. This operation is indicated for
full-thickness chondral defects without associated degenerative arthrosis. Microfracture is most commonly
performed in the knee, though it has also been applied to other joints. Type III collagen is not a
predominant component of fibrocartilage. Type IX and X are minor collagenous components of cartilage.

REFERENCES: Magnussen RA, Dunn WR, Carey JL, et al: Treatment of focal articular cartilage defects in
the knee: A systematic review. Clin Orthop Relat Res 2008;466:952-962.
Williams RJ III, Hamly HW: Microfracture: Indications, technique, and results. Instr Course Lect
2007;56:419-428.
Mithoefer K, Williams RJ III, Warren RF, et al: Chondral resurfacing of articular cartilage defects in the
knee with the microfracture technique: Surgical technique. J Bone Joint Surg Am 2006;88:294-304.

Question 41
A 21-year-old female college athlete sustained a stress fracture of the fifth metatarsal 1 year ago which was
treated successfully with surgical stabilization and she returned to normal activities. She now has a tension-
sided femoral neck fracture. After surgical fixation of the fracture, what is the next step in management?

1. Obtain a menstrual history


2. Advise the athlete never to compete in high level endurance sports again
3. Obtain serum calcium levels
4. Obtain a psychiatric consultation
5. Recommend changes in training intensity

PREFERRED RESPONSE: 1

DISCUSSION: Stress fractures can be seen in female athletes who develop the female athletic triad including
amenorrhea, osteoporosis, and eating disorders. Any female athlete with a history of stress fractures should
undergo a workup for this disorder. Workup should include obtaining a menstrual history, obtaining a
nutritional consultation, and obtaining a bone density. When properly counseled, these athletes may return to
high endurance sports activities. Although these athletes may require a change in training intensity or
psychiatric consultation, it would not be the next step in management. Psychiatric consultation may not be
necessary unless an eating disorder has been diagnosed. Serum calcium levels are normal in these patients.
Tension-sided stress fractures of the femoral neck require surgical stabilization with internal fixation as
opposed to compression-sided stress fractures that can be treated with rest and nonsurgical management.

REFERENCES: Feingold D, Hame SL: Female athlete triad and stress fractures. Orthop Clin North Am
2006;37:575-583.
Joy EA, Campbell D: Stress fractures in the female athlete. Curr Sports Med Rep 2005;4:323-328.
Question 42
A 20-year-old college pitcher reports the recent onset of decreased velocity and posterior shoulder pain.
He states that it takes him longer to loosen up but denies any mechanical symptoms. When compared to his
non-throwing shoulder, glenohumeral examination of his throwing shoulder will most likely reveal which of

102
2010 Sports Medicine Examination Answer Book • 35

the following findings?

1. Coracoid tenderness
2. Supraspinatus muscle atrophy
3. Decreased internal rotation of greater than 25 degrees
4. Decreased external rotation of greater than 40 degrees
5. Decreased abduction of greater than 30 degrees

PREFERRED RESPONSE: 3

DISCUSSION: In symptomatic throwing shoulders, loss of internal rotation in abduction resulting from
posteroinferior capsular contraction exceeds adaptive gains in external rotation. Glenohumeral internal rotation
deficit (GIRD) is defined as the loss in degrees of glenohumeral internal rotation of the throwing shoulder
compared with the non-throwing shoulder. The pathologic cascade initially begins with decreased velocity and
command, followed by posterior stiffness and trouble loosening up. Posterior shoulder pain without
mechanical symptom occurs during late cocking and early acceleration phases due to the contracture of the
posterior-inferior capsule. This results in a posterosuperior shift of the glenohumeral contact, resulting in
internal impingement on the undersurface of the posterior superior rotator cuff and strain on the posterior
superior glenoid labral interface. The “slap event” is when the posterior superior labrum and biceps anchor fail
in tension. After the “slap event”, surgery is the likely solution. Prior to this event, however, posterior inferior
capsular stretches may result in resolution of symptoms.

REFERENCES: Burkhart SS, Morgan CD, Kibler WB: The disabled throwing shoulder: Spectrum of
pathology Part I: Pathoanatomy and biomechanics. Arthroscopy 2003;19:404-420.
Burkhart SS, Morgan CD, Kibler WB: The disabled throwing shoulder: Spectrum of pathology Part II:
Evaluation and treatment of SLAP lesions in throwers. Arthroscopy 2003;19:531-539.
Morgan CD: The throwing shoulder, in McGinty JB (ed): Operative Arthroscopy, ed 3. Philadelphia, PA,
Lippincott-Raven, 2003, pp 570-584.

103
36 • American Academy of Orthopaedic S urgeons

Figure 43a Figure 43b


Question 43
A 21-year-old collegiate wrestler and rugby player reports an 8-month history of groin pain. Examination
reveals a slight Trendlenburg gait, abductor weakness, hip flexion of 90 degrees, and internal rotation of 10
degrees. A radiograph and MRI arthrogram are shown in Figures 43a and 43b. What is the next most
appropriate step in management?

1. Abduction orthosis
2. DEXA scan
3. Bone scan
4. Hip osteotomy
5. Hip arthroscopy and osteoplasty

PREFERRED RESPONSE: 5

DISCUSSION: The patient has mixed cam and pincer-type femoroacetabular impingement. The radiograph
shows an aspherical femoral head, and an acetabulum with increasd lateral coverage. The MRI arthrogram
shows a degenerative labral tear and an aspherical femoral head. The triad of MRI- arthrographic findings
consists of anterosuperior labral tears, anterosuperior cartilage lesions, and an increased alpha angle as
reported by Kassarjian and associates. The pathoanatomy of cam-type impingement is characterized by a
pistol-grip deformity of the femoral head-neck junction and a relative retroversion of the femoral head. As a
result, there is convexity of this area that causes abutment against the normal hip acetabulum with range of
motion. Surgical treatment involves recontouring of the femoral head-neck. It can be performed
arthroscopically or as an open procedure. The early results of these procedures are promising. However,
Gerdeman and associates have pointed out that available evidence is very limited and long-term studies will
be needed as to whether treatment halts the natural history of hip arthritis. In contrast, pincer-type
impingement is characterized by retroversion of the acetabulum and abutment of the normal proximal part of
the femur against the abnormal acetabulum. An abduction orthosis has no role in the treatment of
femoroacetabular impingement. A DEXA scan is unlikely to provide any additional useful information. It is
unlikely that a bone scan will add any additional information. A hip osteotomy is not indicated.

REFERENCES: Clohisy JC, McClure JT: Treatment of anterior femoroacetabular impingement with
combined hip arthroscopy and limited anterior decompression. Iowa Orthop J 2005:25:164-171. Gerdeman
AC, Hogan MV, Miller MD: What’s new in sports medicine? J Bone Joint Surg Am 2009;91:241-256.
Kassaijian A, Yoon LS, Belzile E, et al: Triad of MR arthrographic findings in patients with cam-type
femoroacetabular impingement. Radiology 2005;236:588-592.

104
2010 Sports Medicine Examination Answer Book • 37

Question 44
A 13-year-old pitcher is hit in the left intercostal space by a line drive ball. He collapses, is apneic and
unresponsive, and his radial pulse is absent. What is the next step in management?

1. Protect the airway and use smelling salts


2. Protect the airway, move to the shade, and place in reverse Trendelenburg
3. Protect the airway and protect from seizure activity
4. Protect the airway, start CPR, and prepare to cardiovert
5. Protect the airway, and move patient slowly to a spine board

PREFERRED RESPONSE: 4

DISCUSSION: Sudden death in athletes without structural cardiac damage is referred to as commotio
cordis. This is an emergency. The immediate priorities are protection of the airway, starting CPR, and
early cardioversion as this patient has an arrythmia. It is hypothesized to occur from apnea, vasovagal
reflex, or ventricular arrhythmia as reported by Maron and associates from the direct impact of the
baseball during a vulnerable part of the cardiac rhythm. Janda and associates reported that soft-core
baseballs may not differ from standard baseballs with regard to the risk of fatal chest-impact injury while
playing baseball. High survival rates are associated with rapid treatment.

REFERENCES: Maron BJ, Poliac LC, Kaplan JA, et al: Blunt impact to the chest leading to sudden death
from cardiac arrest during sports activities. N Engl J Med 1995;333:337-342.
Janda DH, Bir CA, Viano DC, et al: Blunt chest impacts: Assessing the relative risk of fatal cardiac injury
from various baseballs. J Trauma 1998;44:298-303.

Question 45
Which of the following best describes the pathologic anatomy of cam impingement of the hip?

1. Retroversion of the acetabulum


2. Posteroinferior labral tears
3. Morphologic abnormality of the femoral head
4. Femoral anteversion
5. Femoral head osteonecrosis

PREFERRED RESPONSE: 3

DISCUSSION: Cam impingement creates shearing forces that result in an outside-in directed detachment of
the labrum in the anterosuperior quadrant. Retroversion of the acetabulum is associated with pincer
impingement. The impingement is exhibited with hip flexion. Cam impingement involves a morphologic
abnormality of the femoral head. Pincer lesions result from stresses of a normal femoral neck against an
abnormal acetabular rim. Cam impingement is not associated with osteonecrosis.
REFERENCES: Jaberi FM, Parvizi J: Hip pain in young adults: Femoroacetabular impingement. J
Arthroplasty 2007:22;37-42.
Byrd JW: The role of hip arthroscopy in the athletic hip. Clin Sports Med 2006:25;255-278.
Beck M, Kalhor M, Leunig M, et al: Hip morphology influences the pattern of damage to the acetabular

105
38 • American Academy of Orthopaedic S urgeons

cartilage: Femoroacetabular impingement as a cause of early osteoarthritis of the hip. J Bone Joint Surg Br
2005:87; 1012-1018.

Question 46
Emergent management of acute tooth displacement (luxation) includes

1. delaying replantation until a dentist is present.


2. scrubbing the root of the tooth clean with hydrogen peroxide.
3. transporting the tooth in a carbonated beverage.
4. emergency root canal.
§. immediate repositioning or replantation of the tooth.

PREFERRED RESPONSE: 5

DISCUSSION: Avulsed teeth must be replanted immediately to enhance viability of the periodontal ligament
cells on the root. With the tooth in place, the athlete should bite down on a towel to maintain stability. The
athlete should be taken emergently to a dentist’s office or emergency room. The avulsed tooth should not be
handled by the root or scrubbed to remove debris. If immediate replantation is not possible, the tooth should
be transported in saline solution, milk, or saliva on gauze.

REFERENCES: Flores MT, Andreasen JO, Bakland LK, et al: Guidelines for the evaluation and
management of traumatic dental injuries. Dent Traumatol 2001; 17:97-102.
Ranalli DN, Demas PN: Orofacial injuries from sport preventive measures for sports medicine. Sports Med
2002;2:409-418.

Question 47
Early failure of arthroscopic rotator cuff repair most commonly occurs by which of the following
mechanisms?

1. Anchor pull-out
2. Anchor fracture
3. Suture rupture
4. Knot failure
5. Tissue failure
DISCUSSION: Arthroscopic repair of the rotator cuff is becoming increasingly popular. Unfortunately, recent
objective evaluations have indicated high failure rates even in patients who are clinically improved. Early
failure can occur by failure of the suture anchor, suture, or knot. However, the most common cause of failure
is when the suture pulls through the tendon. This results in “stretching” of the repair that can lead to gap
formation between the repaired tendon and the osseous insertion, and subsequently, poor tendon-to-bone
healing.

106
2010 Sports Medicine Examination Answer Book • 39

Question 49
REFERENCES: Gerber C, Schneeberger AG, Beck M, et al: Mechanical strength of repairs of the rotator
cuff. J Bone Joint Surg Br 1994;76:371-380.
Cummins CA, Murrell GA: Mode of failure for rotator cuff repair with suture anchors identified at revision
surgery. J Shoulder Elbow Surg 2003;12:128-133.
Ma CB, MacGillivray JD, Clabeaux J, et al: Biomechanical evaluation of arthroscopic rotator cuff stitches. J
Bone Joint Surg Am 2004;86:1211-1216.

Question 48
Which of the following pieces of equipment currently offers the greatest opportunity for lowering the number
of equestrian injuries?

1. Knee pads
2. Wrist guards
3. Boots
4. Helmets
5. Quick release stirrups

PREFERRED RESPONSE: 4

DISCUSSION: Ball and associates reported that “horseback riding was more dangerous than motorcycle
riding.” In a 10-year study of major traumatic injuries, they reported that 151 (2%) of 7,941 trauma patients
had major equestrian injuries (injury severity score > or = 12). Injuries included the chest (54%), head (48%),
abdomen (22%), and extremities (17%). Only 9% of riders wore helmets, and 64% believed the accident was
preventable. The authors noted that “helmet and vest use will be targeted in future injury prevention
strategies.” In another study, Frankel and associates noted that helmet use was only documented in 34% of
riders. Although orthopaedic injuries are common, knee pads, wrist guards, boots, and quick release stirrups
would most likely have less impact on injury prevention.

REFERENCES: Ball CG, Ball JE, Kirkpatrick AW, et al: Equestrian injuries: Incidence, injury patterns, and
risks factors for 10 years of major traumatic injuries. Am J Surg 2007;193:636-640.
Frankel HL, Haskell R, Digiacomo JC, et al: Recidivism in equestrian trauma. Am Surg 1998;64:151-154.
A 38-year-old man is three quarters of the way through the Hawaiian Ironman events run in a temperature of
60°F. He is sweating profusely and suddenly collapses. Prior to this he had been drinking large amounts of
bottled water at every water stop. What is the most likely diagnosis?

1. Hypernatremia
2. Hypothermia
3. Hyponatremia
4. Subendocardial myocardial infarction
5. Ruptured berry aneurysm

PREFERRED RESPONSE: 3

107
40 • American Academy of Orthopaedic S urgeons

DISCUSSION: Hyponatremia is often seen in endurance athletes such as triathloners, ultramarathoners, and
marathoners after prolonged exertion. It is commonly attributed to excess free water intake that fails to replete
massive sodium losses that result from sweating as reported by O’Connor. Exercise-induced hyponatremia is
generally asymptomatic, particularly in patients in whom the sodium is only mildy reduced. Up to 10% of
ultradistance athletes have a sodium level of 135 mEq/L or less, but those who are symptomatic usually have
a sodium level of 125 mEq/L as reported by Noakes and O’Connor. The best way to prevent hyponatremia is
to maintain the proper volume and types of fluid intake to ensure fluid balance during exercise. Beverages
containing carbohydrates in concentrations of 4% to 8% (ie, “sports drinks”) are recommended for athletes
participating in exercise lasting more than an hour (eg, marathon runners, etc.) To avert brainstem herniation
and death, severe, acute hyponatremia requires rapid correction. Oral rehydration with salty solutions is safe
and effective in patients with mild symptoms.
Too rapid correction has been reported to cause central pontine myelinolysis; therefore, correction ought to
be performed slowly. Hypernatremia, hypothermia, subendocardial myocardial infarction, or ruptured berry
aneurysm are unlikely in this scenario.

REFERENCES: O’Connor RE: Exercise-induced hyponatremia: Causes, risks, prevention, and management.
Cleve Clin J Med 2006;73:S13-S18.
Noakes T: Hyponatremia in distance runners: Fluid and sodium balance during exercise. Curr Sports Med Rep
2002;1:197-207.
Laureno R, Karp BI: Myelinolysis after correction of hyponatremia. Ann Int Med 1997;126:57-62.
Question 50
A 20-year-old male tennis player reports the acute onset of ulnar-sided wrist pain after hitting a forehand
shot. Examination reveals dorsoulnar tenderness and minimal swelling. The pain is recreated with
supination, wrist flexion, and ulnar deviation. Radiographs are normal. What structure is most likely
involved?

1. Ulnar styloid
2. Flexor carpi radialis tendon
3. Extensor carpi ulnaris tendon
4. Scapholunate ligament
5. Transverse carpal ligament

PREFERRED RESPONSE: 3

DISCUSSION: Extensor carpi ulnaris (ECU) lesions produce pain at the dorsoulnar aspect of the wrist,
particularly during wrist supination, wrist flexion, and ulnar deviation. It has been frequently described in
tennis players. Most ECU tenosynovitis can be successfully treated nonsurgically with immobilization
techniques. Surgical treatment is generally indicated for ECU tenosynovitis or tendinopathy that does not
respond to rest. Anatomically, the ECU retinaculum can rupture and the tendon can leave its sheath. With
supination, the tendon can leave the sheath and then return to its position during pronation.

REFERENCES: Montalvan B, Parier J, Brasseur JL, et al: Extensor carpi ulnaris injuries in tennis players: A
study of 28 cases. Br J Sports Med 2006;40:424-429.
Allende C, Le Viet D: Extensor carpi ulnaris problems at the wrist: Classification, surgical treatment and
results. J Hand Surg Br 2005;30:265-272.

108
2010 Sports Medicine Examination Answer Book • 41

Question 51
A 16-year-old female gymnast reports a 9-month history of knee pain with activities of daily living and night
pain. Management consisting of nonsteroidal anti-inflammatory drugs and physical therapy has failed to
provide relief. Examination reveals posterior soft-tissue fullness just proximal to the popliteal fossa, no
effusion, 130 degrees of knee motion, no instability, negative meniscus signs, and a normal gait.
Radiographs are normal. What is the next best step in management?

1. Additional physical therapy


2. Corticosteroid injection
3. Sympathetic block
4. MRI
5. Bone scan
DISCUSSION: The phenomena of tumors misdiagnosed as athletic injuries has been termed “sports tumors”
by Lewis and Reilly. These authors presented a series of 36 patients who initially were thought to have a
sports-related injury but ultimately were diagnosed with a primary bone tumor, soft-tissue tumor, or tumor-
like condition. Muscolo and associates presented a series of 25 tumors that had been previously treated with
an intra-articular procedure as a result of a misdiagnosis of an athletic injury. Initial diagnoses included 21
meniscal lesions, one traumatic synovial cyst, one patellofemoral subluxation, one anterior cruciate ligament
tear, and one case of nonspecific synovitis. The final diagnoses were a malignant tumor in 14 patients and a
benign tumor in 11 patients. The authors noted that oncologic surgical treatment was affected in 15 of the 25
patients. The most frequent causes of erroneous diagnosis were initial poor quality radiographs and an
unquestioned original diagnosis despite persistent symptoms. Persistent symptoms warrant further
diagnostic studies, not additional treatment such as physical therapy, corticosteroid injection, or sympathetic
block. Although a bone scan may be useful, the possibility of a soft-tissue mass makes MRI the preferred
initial imaging modality in this patient.

REFERENCES: Muscolo DL, Ayerza MA, Makino A, et al: Tumors about the knee misdiagnosed as ath-
letic injuries. J Bone Joint Surg Am 2003;85:1209-1214.
Lewis MM, Reilly JF: Sports tumors. Am J Sports Med 1987;15:362-365.

Question 52
When performing elbow arthroscopy, it is often necessary to evaluate the posterior compartment. When
entering the posterior compartment of the elbow, what are the two safest and most commonly used portals?

1. The posterior portal created 3 cm proximal to the tip of the olecranon and the posterior medial
portal created 3 cm from the tip of the olecranon and medial to the triceps

109
42 • American Academy of Orthopaedic S urgeons

2. The posterior portal created 3 cm proximal to the tip of the olecranon and the posterior lateral portal
created 3 cm proximal from the tip of the olecranon and just lateral to the triceps
3. The posterior medial portal created 3 cm from the tip of the olecranon and medial to the tri ceps
and the posterior lateral portal created 3 cm from the tip of the olecranon and lateral to the triceps
4. The posterior medial portal created 3 cm from the tip of the olecranon and the lateral portal
made through the anconeus
5. The posterior portal created at the tip of olecranon and the posterior medial portal just medial to
the triceps

PREFERRED RESPONSE: 2
DISCUSSION: The posterior portal created 3 cm proximal to the tip of the olecranon and the posterior lateral
portal created 3 cm proximal from the tip of the olecranon and just lateral to the triceps are the “workhorse”
portals of the posterior compartment and although relatively safe, risks exist. The radial nerve proximity
averages 4.8 mm (3 to 8 mm) from the posterolateral portal. The central posterior portal close to 20 mm from
the ulnar nerve.

REFERENCES: Steinmann SP: Elbow arthroscopy. J Am Society of the Hand 2003 ;3:199-207.
Dodson CC, Nho SJ, Williams RJ III, et al: Elbow Arthroscopy. J Am Acad Orthop Surg 2008:16:574- 585.

Question 53
Biomechanical in vitro studies of double-row anchor fixation of rotator cuff tears show what initial advantage
over single-row anchor fixation?

1. Increased peak-to-peak elongation


2. Decreased stiffness
3. Higher ultimate tensile load
4. Decreased contact area
5. Increased conditioning elongation

PREFERRED RESPONSE: 3

DISCUSSION: Biomechanical in vitro studies of double-row fixation of rotator cuff tears during cyclic
loading and tensile loading to failure have demonstrated that double-row fixation results in a higher ultimate
tensile load when compared to single-row fixation. Peak-to-peak elongation, stiffness, and conditioning
elongation for double-row fixation were all similar to single-row fixation. These initial findings, however, may
or may not lead to improved clinical outcomes.

REFERENCES: Ma CB, Comerford L, Wilson J, et al: Biomechanical evaluation of arthroscopic rotator cuff
repairs: Double-row compared with single-row fixation. J Bone Joint Surg Am 2006;88:403-410. Kim DH, El

110
2010 Sports Medicine Examination Answer Book • 43

Attrache NS, Tibone JE, et al: Biomechanical comparison of single-row versus double-row suture anchor
technique for rotator cuff repair. Am J Sports Med 2006;34:407-414.
Question 54
A 17-year-old pitcher reports pain over the medial aspect of the elbow that occurs during the acceleration
phase of throwing, and it prevents him from throwing at the velocity needed to be competitive. What structure
is most likely injured in this patient?

1. Radial collateral ligament


2. Posterior bundle of the ulnar collateral ligament
3. Anterior bundle of the ulnar collateral ligament
4. Flexor carpi ulnaris
5. Pronation teres
PREFERRED RESPONSE: 3
DISCUSSION: The anterior bundle of the ulnar collateral ligament of the elbow is the primary constraint to
valgus force of the elbow. In pitchers and in overhead athletes, injury to this portion of the ligament results in
valgus instability. Reconstruction of the anterior band of the ulnar collateral ligament is necessary in many
elite athletic throwers to allow them to return to this competitive activity.
REFERENCES: Azar FM, Andrews JR, Wilk KE, et al: Operative treatment of ulnar collateral ligament
injuries of the elbow in athletes. Am J Sports Med 2000;28:16-23.
Cain EL, Dugas JR, Wolf RS, et al: Elbow injuries in throwing athletes: A current concepts review. Am J
Sports Med 2003;31:621-635.
Rettig AC, Sherrill C, Snead DS, et al: Nonoperative treatment of ulnar collateral ligament injuries in
throwing athletes. Am J Sports Med 2001 ;29:15-17.

111
44 • American Academy of Orthopaedic S urgeons

/
Figure 55a Figure 55b
Question 55
A 22-year-old male basketball player has had knee pain for the past 3
months. He denies any history of trauma. He has symptoms of catching but no locking. He has rested for 2
weeks but symptoms returned when he resumed sporting activities. T r and T2-weighted MRI scans are shown
in Figures 55a and 55b. What is the most likely diagnosis?

1. Locked lateral meniscus tear


2. Anterior cruciate ligament tear
3. Ganglion cyst of the anterior cruciate ligament
4. Synovial osteochondromatosis
5. Pigmented villonodular synovitis

PREFERRED RESPONSE: 3

DISCUSSION: The MRI scans show a cystic structure within the anterior cruciate ligament. It is fluid filled
as seen on the T2 sequence. Ganglion cysts of the cruciate ligaments are rare. The most common presentation
is pain with occasional loss of motion. Instability is not a chief complaint and often there is no evidence of
laxity on examination. If nonsurgical management fails, arthroscopic debridement of the cyst is the accepted
method of treatment.

REFERENCES: Liu SH, Osti L, Mirzayan R: Ganglion cysts of the anterior cruciate ligament: A case report
and review of the literature. Arthroscopy 1994; 10:110-112.
Parish EN, Dixon P, Cross MJ: Ganglion cysts of the anterior cruciate ligament: A series of 15 cases.
Arthroscopy 2005;21:445-447.

112
2010 Sports Medicine Examination Answer Book • 45

Figure 56

Question 56
Figure 56 shows an arthroscopic view of the long head of the biceps; it has an incompetent biceps sling and is
unstable, and an axial glenohumeral MRI scan reveals that it is dislocated medially out of the intertubercular
groove. What structure is also most likely injured?

1. Middle glenohumeral ligament


2. Supraspinatus
3. Infraspinatus
4. Subscapularis
5. Bankart tear

PREFERRED RESPONSE: 4

DISCUSSION: It is important to recognize that rotator cuff tears are a common finding in the setting of a
dislocated long head of the biceps tendon (LHB) from the intertubercular groove of the shoulder. If a LHB
tendon dislocation is found on examination or radiographic work-up (ultrasound or MRI), it is imperative to
rule out associated rotator cuff pathology, specifically of the subscapularis tendon. Although very rare, injury
to the lesser tuberosity should also be ruled out. There are a variety of methods to treat the dislocated biceps
(tenotomy versus tenodesis); however, the entire rotator cuff, especially the subscapularis, should be carefully
inspected and treated if necessary. The corollary is also true - if you find a tear of the subscapularis tendon
insertion, especially the superior half, the LHB should be carefully inspected to ensure that it is not unstable
as it exits the shoulder. If the LHB is unstable, this is also addressed surgically with either tenotomy or
tenodesis. The middle glenohumeral ligament and Bankart tears are not stabilizers of the biceps.

REFERENCES: Sethi N, Wright R, Yamaguchi K: Disorders of the long head of the biceps tendon. J
Shoulder Elbow Surg 1999;8:644-654.
Edwards TB, Walch G, Sirveaux F, et al: Repair of tears of the subscapularis: Surgical technique. J Bone
Joint Surg Am 2006;88:1-10.
Tung GA, Yoo DC, Levine SM, et al: Subscapularis tendon tear: Primary and associated signs on MRI. J
Comput Assist Tomogr 2001;25:417-424.

113
46 • American Academy of Orthopaedic Surgeons

Figure 57
Question 57
A 57-year-old man who plays recreational sports reports pain in his dominant shoulder. An MR arthrogram is
shown in Figure 57. During arthroscopy of the shoulder, what pathology is most likely to be found?

1. Complete disruption of the transverse humeral ligament


2. Acromioclavicular joint arthritis
3. Absent coracohumeral ligament
4. Subscapularis tear and biceps subluxation
5. Complete rupture of the short head of the biceps

PREFERRED RESPONSE: 4

DISCUSSION: The MR arthrogram shows medial subluxation of the biceps tendon out of the bicipital
groove and a subscapularis tendon tear. Biceps tendon subluxation is almost always associated with
subscapularis tears. Whereas other diagnoses can be associated, none of them is directly related to this
finding or seen on the MR arthrogram.

REFERENCES: Lafosse L, Jost B, Reiland Y, et al: Structural integrity and clinical outcomes after
arthroscopic repair of isolated subscapularis tears. J Bone Joint Surg Am 2007;89:1184-1193.
Tonino PM, Gerber C, Itoi E, et al: Complex shoulder disorders: Evaluation and treatment. J Am Acad Orthop
Surg 2009:17:125-136.

Question 58
Which of the following is considered an advantage of the tibial inlay fixation compared to transtibial tunnel
technique when used in posterior cruciate ligament reconstruction?

1. Less invasive
2. Superior published clinical results
3. Decreased surgical time
4. Elimination of the critical 90-degree turn at the tibial aperture of the tunnel
5. Improved cosmesis
DISCUSSION: One of the most difficult aspects of posterior cruciate ligament reconstruction is placement of
the tibial tunnel and passing of the graft through this tunnel. The tibial inlay technique requires a
posteromedial approach to the tibia whereby the graft is directly fixed to the posterior aspect of the tibia. This
obviates the need for a tibial tunnel. This technique has never been shown to be less invasive, more cosmetic,
PREFERRED RESPONSE: 1

114
2010 Sports Medicine Examination Answer Book • 47

or require decreased surgical time. It has also never been shown in a published level I study to have superior
clinical results. However, it does eliminate the need for the 90-degree critical “killer” turn and passing of the
tibial graft through the tibial tunnel which may lead to graft failure.

REFERENCES: McAllister DR, Petrigliano FA: Diagnosis and treatment of posterior cruciate ligament injuries.
Curr Sports Med Rep 2007;6:293-299.
Cosgarea AJ, Jay PR: Posterior cruciate ligament injuries: Evaluation and management. J Am Acad
Orthop Surg 2001;9:297-307.

Question 59
Figure 59 shows properties of a material being tested for use as an implant. What is represented by the portion of
the stress-strain curve from point A to point B?

1. Elastic limit
2. Nonproportional behavior
3. Plastic behavior
4. Elastic behavior
5. Fracture point

PREFERRED RESPONSE: 4

DISCUSSION: The figure is a stress-strain diagram representing specific metal subjected to increasing tensile
stress. The portion of the curve from A to B is a straight line demonstrating a proportional increase in strain for
each increase in tensile stress. If the stress is removed at any point between A and C, the material will return to
its original shape, returning back along the original curve without permanent deformation. This is termed elastic
behavior. If the applied stress causes strain beyond point C, then permanent deformation occurs and returns
along a different path to a different zero stress point. This is termed plastic behavior. The point C at which the
material stops behaving in an elastic manner and begins behaving in a plastic manner is the elastic limit or yield
point. Point D represents a point on the curve of plastic deformation. Point E is the fracture point when the stress
on the material creates enough strain that the material fractures.
REFERENCES: Vaccaro AR (ed): Orthopaedic Knowledge Update 8. Rosemont, IL, American Academy of
Orthopaedic Surgeons, 2005, pp 45-46.
El-Ghannam A, Ducheyne P: Biomaterials, in Mow VC, Huiskes R (eds): Basic Orthopaedic Biomechanics
and Mechano-Biology, ed 3. Philadelphia, PA, Lippincott-Raven, 2005, pp 501-503.

115
48 • American Academy of Orthopaedic S urgeons

Figure 60
Question 60
A patient competing in a professional motocross race sustained a direct blow to the knee after falling off his
bike at high speed. He sustained several lacerations as shown in Figure 60. He is able to actively extend his
knee painlessly and his Lachman examination is negative. What is the most likely injury?

1. Anterior cruciate ligament tear


2. Patella fracture
3. Patellar tendon tear
4. Tibial tubercle avulsion
5. Posterior cruciate ligament tear

PREFERRED RESPONSE: 5

DISCUSSION: It is important to recognize the injury pattern sustained by this motocross rider by inspection
of his traumatic scars present anteriorly over the proximal tibia and the dorsum of the ankle and dorsum of
the forefoot, indicating that his foot was in a plantar flexed position with a concomitant blow to the anterior
tibia. This is a classic mechanism for a posterior cruciate ligament injury, and external clues (the scars)
should not be overlooked when examining the knee. Occasionally, a posterior cruciate ligament injury is
overlooked; however, putting together the patient’s history, the examination (especially the posterior drawer
and quadriceps active tests) provide a reliable diagnosis. Additional pathology should also be ruled out, such
as a posterolateral corner injury and intra-articular pathology. Patella fracture, tibial tubercle avulsion, and
patella tendon tears are unlikely because the patient can actively extend the knee.
An anterior cruciate ligament tear is unlikely with a negative Lachman examination.
REFERENCES: Janousek AT, Jones DG, Clatworthy M, et al: Posterior cruciate ligament injuries of the knee
joint. Sports Med 1999;28:429-441.
McAllister DR, Petrigliano FA: Diagnosis and treatment of posterior cruciate ligament injuries. Curr Sports
Med Rep 2007;6:293-299.

Question 61
The sublime tubercle of the elbow serves as the insertion site of the

1. anterior bundle of the medial collateral ligament


2. posterior bundle of the medial collateral ligament.

116
2010 Sports Medicine Examination Answer Book • 49

3. transverse bundle of the medial collateral ligament.


4. annular ligament.
5. lateral collateral ligament.
PREFERRED RESPONSE: 1

DISCUSSION: The anterior bundle originates on the anteroinferior medial humeral epicondyle and inserts on
the medial portion of the coronoid, known as the sublime tubercle.

REFERENCES: O’Driscoll SW, Jaloszynski R, Morrey BF, et al: Origin of the medial ulnar collateral
ligament. J Hand Surg Am 1992; 17:164-168.
Grace SP, Field LD: Chronic medial elbow instability. Orthop Clin North Am 2008;39:213-219.

Question 62
During the cocking and acceleration phases of the overhand throw (pitch), there are several static and
dynamic restraints to provide medial elbow support and prevent valgus instability. The dynamic structures
found to be most important during these phases of the overhand throw are the flexor digitorum

1. profundus and extensor carpi radialis longus.


2. profundus and extensor carpi radialis brevis.
3. superficialis and extensor carpi radialis longus.
4. superficialis and flexor carpi ulnaris.
5. superficialis and flexor carpi radialis.
PREFERRED RESPONSE: 4

DISCUSSION: Biomechanical analysis has demonstrated that local dynamic stability of the elbow is provided
by the flexor digitorum superficialis and the flexor carpi ulnaris, especially during the cocking and
acceleration phases of the overhand throw. This provides dynamic joint compression across the elbow joint
and may be protective to the static restraints such as the ulnar collateral ligament. It also emphasizes the need
to strengthen distant muscles in the forearm to assist with elbow biomechanics and potentially prevent injury.

117
50 • American Academy of Orthopaedic S urgeons

REFERENCES: Davidson PA, Pink M, Perry J, et al: Functional anatomy of the flexor pronator muscle group
in relation to the medial collateral ligament of the elbow. Am J Sports Med 1995;23:245-250. Garrick JG (ed):
Orthopaedic Knowledge Update: Sports Medicine 3. Rosemont, IL, American Academy of Orthopaedic
Surgeons, 2004, pp 101-111.

Figure 63a Figure 63b


Question 63
A 17-year-old high school basketball player reports chronic pain in the posterior aspect of the right ankle. He
denies any injury. His pain is made worse by jumping and “taking off’ the right leg while doing layups.
Examination reveals no discernible swelling. He has full active and passive range of motion of the ankle,
although maximal passive plantar flexion is painful posteriorly. He is unable to do a single-leg toe raise. He
has no tenderness or palpable mass over the Achilles tendon. Motor function to the foot is within normal
limits, and his neurovascular examination is intact. A lateral radiograph is shown in Figure 63a and an MRI
scan is shown in Figure 63b. Which of the following treatment options has the highest likelihood of success
for this condition?

1. Physical therapy
2. Corticosteroid injection
3. Open excision
4. Arthroscopic fixation
5. Open reduction and internal fixation

PREFERRED RESPONSE: 3

DISCUSSION: The patient’s history, examination, and imaging studies are consistent with the os trigonum
syndrome. The os trigonum is an accessory ossification center of the posterior process of the talus. It can
become painful in some athletes after ankle dorsiflexion or jumping, and is most common in ballet dancers and
basketball players. Traumatic disruption of the synchondrosis between the os trigonum and the talus can lead
to pain. MRI imaging of these injuries typically shows fluid surrounding the os with associated marrow
edema. There is usually no marrow edema in the talus to suggest an acute fracture of the posterior process.
These injuries are definitively managed with excision (either open or arthroscopic) of the inflamed os
trigonum with return to sports expected within 2 to 3 months. Physical therapy may provide short-term relief
but is unlikely to provide permanent relief. Corticosteroid injection is not recommended in this location.
Surgical fixation is not indicated for this entity.

118
2010 Sports Medicine Examination Answer Book • 51

REFERENCES: Chao W: Os trigonum. Foot Ankle Clin 2004;9:787-796.


Kadel N: Excision of os trigonum. Operative Techniques in Orthopaedics 2004; 14:1-5.
Davies M: The os trigonum syndrome. Foot 2004; 14:119-123.

Question 64
An 18-year-old female Marine Corps recruit enters basic training. Her enlistment history and physical
examination showed that she was an elite high school cross country runner. What is her most significant risk
factor for a femoral or pelvic stress fracture during basic training?

1. Running mileage during the 2 months prior to basic training


2. Self-rated fitness
3. Running frequency during the 2 months prior to basic training
4. No menstrual bleeding during the year prior to basic training
5. Race/ethnicity

PREFERRED RESPONSE: 4

DISCUSSION: Approximately 5% of female recruits incur a stress fracture during the 13 weeks of Marine
Corps basic training. Approximately 40% of these were femoral or pelvic stress fractures that were more
severe than in civilian athletes or male military recruits. Only women who reported no menses during the
previous year had a greater likelihood of femoral or pelvic stress fractures than did women who reported 10 to
12 menses. The referenced study did not find a statistically significant increase in risk of stress fracture in those
recruits who had lesser menstrual irregularities in the year prior to recruit training, but there was a trend toward
increased risk of stress fracture.

REFERENCES: Shaffer RA, Rauh MJ, Brodine SK, et al: Predictors of stress fracture susceptibility in
young female recruits. Am J Sports Med 2006;34:108-115.
Garrick JG (ed): Orthopaedic Knowledge Update: Sports Medicine 3. Rosemont, IL, American Academy of
Orthopaedic Surgeons, 2004, pp 273-283.
Question 65
During preparation for the NCAA wrestling championships, a participant reports the development of
vesicular lesions on his right chest wall that are mildly painful; however, they have not affected his ability to
wrestle. How should this athlete be managed?

1. He may wrestle if his lesions are covered.


2. He may wrestle if he is on oral antiviral agents for 48 hours.
3. He may wrestle immediately with no other treatment.
4. He cannot wrestle until the lesions are scabbed over and there are no new lesions for at least 72
hours.
5. He cannot wrestle for 2 weeks.
PREFERRED RESPONSE: 4

DISCUSSION: Herpes simplex virus (HSV) can cause serious outbreaks on athletic teams, especially
wrestling. HSV is highly contagious; it is secreted from active blisters, saliva, and mucous membranes. For
wrestlers, the NCAA states that the athlete must be free from systemic symptoms and any new blisters for 72

119
52 • American Academy of Orthopaedic Surgeons

hours before being allowed to participate. Also, all lesions must be dry and crusted and at least 120 hours of
antiviral therapy should have been instituted.

REFERENCES: Garrick JG (ed): Orthopaedic Knowledge Update: Sports Medicine 3. Rosemont, IL,
American Academy of Orthopaedic Surgeons, 2004, pp 301-309.
Johnson R: Herpes Gladiatorium and other skin diseases. Clin Sports Med 2004;23:473-484.

Question 66
An 18-year-old high school basketball player is being treated for Achilles tendinitis. What type of
strengthening exercise has been shown to be helpful in the later phases of rehabilitation?

1. Eccentric
2. Isokinetic
3. Concentric
4. Isometric
5. Isotonic

PREFERRED RESPONSE: 1

DISCUSSION: Eccentric strengthening for tendinopathies has proved most helpful in the later stages of
rehabilitation. Although the exact mechanism of the effect on eccentric exercises is not known, the most
widely accepted theory is that the absence of concentric stretching disrupts the normal lengthing/shorten-
ing cycle which may cause shearing in the tendon and injury to the collagen. Isokinetic exercise maintains
a constant angular velocity of joint motion. Isotonic exercise maintains a constant force of contraction
while isometric contraction develops force without changing the length of the musculotendinous unit.
All three types of these exercises have not been shown to benefit Achilles tendinitis as much as eccentric
exercise.
REFERENCES: Jonsson P, Alfredson H, Sunding K, et al: New regimen for eccentric calf-muscle training in
patients with chronic insertional Achilles tendinopathy: Results of a pilot study. Br J Sports Med
2008;42:746-749.
Maffulli N, Walley G, Say ana MK, et al: Eccentric calf muscle training in athletic patients with Achilles
tendinopathy. Disabil Rehabil 2008;30:1677-1684.

Figure 67

Question 67
PREFERRED RESPONSE: 1

120
2010 Sports Medicine Examination Answer Book • 53

Which of the following clinical symptoms will result from disruption of the structure indicated by the probe
shown in Figure 67?

1. Increased inferior glenohumeral translation with the arm at the side


2. Increased posterior glenohumeral translation with the arm elevated and adducted
3. Increased posterior glenohumeral translation with the arm abducted and externally rotated
4. Increased anterior glenohumeral translation with the arm elevated and adducted
5. Increased anterior glenohumeral translation with the arm abducted and externally ro tated

PREFERRED RESPONSE: 5

DISCUSSION: The structure indicated by the probe is the inferior glenohumeral ligament - one of the
three glenohumeral ligaments. This ligament provides the primary ligamentous restraint to anterior
glenohumeral translation with the arm in an abducted and externally rotated position.

REFERENCES: Bigliani LU, Pollock RG, Soslowsky LJ, et al: Tensile properties of the inferior
glenohumeral ligament. J Orthop Res 1992;10:187-197.
O’Brien SJ, Neves MC, Amoczky SP, et al: The anatomy and histology of the inferior glenohumeral
ligament complex of the shoulder. Am J Sports Med 1990; 18:449-456.
Question 68
A 19-year-old college football player is tackled and hits his head on the ground. There is no loss of
consciousness and the player walks to the appropriate side and sits on the bench. As the team physician, you
evaluate him and he is disoriented at first but after a few minutes, he responds more appropriately. You
evaluate him with the Standardized Assessment of Concussion (SAC) instrument and note that his score is
similar to his baseline scores obtained during the preseason. The coach is asking if he can return to the game.
The next step in the management of this patient involves

1. repeat testing with the SAC following mild to moderate sideline exertion.
2. further observation for 15 more minutes before return to play.
3. further observation for 30 more minutes before return to play.
4. restricting this player from further play this game.
5. immediate transportation to an emergency department for further evaluation.

PREFERRED RESPONSE: 1

DISCUSSION: A patient must be symptom-free before return to play. It is necessary to test each player
following exertion to verify that symptoms do not recur following mild to moderate exertion. When
evaluating athletes with the Standardized Assessment of Concussion (SAC) instrument, it is necessary to
compare to the baseline scores. Since his SAC score was similar to baseline testing and he is
asymptomatic, restriction from play and immediate transport to the emergency department are not
indicated.

REFERENCES: Practice parameters: The management of concussion in sport: Report of the Quality
Standards Subcommittee. Neurology 1997;48:581-585.
Garrick JG (ed): Orthopaedic Knowledge Update: Sports Medicine 3. Rosemont, IL, American Academy of
Orthopaedic Surgeons, 2004, pp 29-44.
Marion DW: Head injuries, in Fu FH, Stone DA (eds): Sports Injuries: Mechanisms, Prevention,

121
54 • American Academy of Orthopaedic Surgeons

Treatment, ed 2. Philadelphia, PA, Lippincott, Willia ms and Wilkins, 2001, pp 925-943.


Wojtys EM, Hovda D, Landry G, et al: Current concepts: Concussion in sports. Am J Sports Med
1999;27:676-687.

Question 69
During a knee arthroscopy on a 38-year-old patient with isolated medial knee pain and no lateral
symptoms, a routine examination of the lateral compartment reveals a discoid lateral meniscus. The
discoid lateral meniscus is not torn. Based on these findings, what is the most appropriate action?

1. Complete lateral meniscectomy


2. Lateral meniscal repair
3. Saucerization of the lateral meniscus
4. Microfracture of the lateral femoral condyle
5. Do nothing surgically to the lateral meniscus
DISCUSSION: The most appropriate action is to note this finding in the surgical report but do nothing
surgically in the lateral compartment. Multiple studies have shown that asymptomatic discoid lateral me nisci
seen on routine knee arthroscopies for other pathology need not be addressed surgically. They do not cause
problems later in life and do not need to be treated prophylactically.

REFERENCES: Smith CF, Van Dyk GE, Jurgutis J, et al: Cautious surgery for discoid menisci. Am J Knee
Surg 1999;12:25-28.
Kelly BT, Green DW: Discoid lateral meniscus in children. Curr Opinion Pediatr 2002;14:54-61.

Question 70
A high school football player asks you about an oral supplement that increases body mass and improves sprint
times. He would like to use it to improve performance. What is the most likely agent?

1. Creatine
2. Caffeine
3. Testosterone
4. Human growth hormone
5. Ephedrine
PREFERRED RESPONSE: 1

DISCUSSION: The supplement is creatine. Approximately 17% of high school athletes and about 30% of
high school football players use creatine. Creatine is a protein synthesized in the liver and the kidney,
circulates in the bloodstream, and is incorporated into muscle. Its use is associated with increased muscle
mass, short-term improvement in sprinting, and may allow for increased anaerobic resistance performance.
Caffeine and ephedrine are taken orally but do not increase muscle mass. Testosterone and human growth
hormone are both associated with increased body mass but must be injected.

PREFERRED RESPONSE: 1

122
2010 Sports Medicine Examination Answer Book • 55

REFERENCES: McGuine TA, Sullivan JC, Bernhardt DT: Creatine supplementation in high school football
players. Clin J Sports Med 2001 ;11:247-253.
Rawson ES, Gunn B, Clarkson PM: The effects of creatine supplementation on exercise-induced muscle
damage. J Strength Cond Res 2001; 15:178-184.
Branch JD: Effect of creatine supplementation on body composition and performance: A meta-analysis.
Int J Sport Nutr Exerc Metab 2003;13:198-226.

123
56 • American Academy of Orthopaedic Surgeons

Question 71
An 11-year-old boy who is a Little League pitcher has a 3-month history of right elbow pain, made worse after
several innings of pitching. The pain is in the posterior and medial aspect of the elbow joint but is without
clicking or mechanical symptoms. There are no signs of infection or swelling, and range of motion is full.
There is tenderness over the medial aspect of the elbow distal to the humeral epicondyle over the proximal
olecranon. Valgus stress testing of the elbow is normal. What is the most likely diagnosis?

1. Olecranon bursitis
2. Osteochondritis dissecans of the capitellum
3. Ulnar collateral ligament insufficiency
4. Medial epicondylitis
5. Olecranon stress fracture

PREFERRED RESPONSE: 5

DISCUSSION: The patient has an olecranon stress fracture due to overuse injury from pitching. The
repetitive forceful contraction of the triceps coupled with varus and valgus torques about the elbow are felt
to cause the olecranon epiphysis to separate from the adjacent epiphyseal plate as reported by Torg and
Moyer. This may persist into late adolescence; Charlton and Chandler described five throwing athletes
between the ages of 16 to 20 years with delayed closure of the olecranon epiphysis and inability to throw.
The ulnar collateral ligament was intact in all. The patients in their study underwent open reduction and
internal fixation with tension band wire, screw fixation, and autogenous bone graft in some of the cases.
At 32 months, all were asymptomatic despite a prolonged preoperative course (> 30 months) of limiting pain.
It is important to recognize stress fractures about the elbow in a young pitching population and treat
accordingly first with rest and cessation of throwing activities. If prolonged, surgical fixation provides reliable
results.

REFERENCES: Charlton WP, Chandler RW: Persistence of the olecranon physis in baseball players: Results
following operative management. J Shoulder Elbow Surg 2003;12:59-62.
Torg JS, Moyer RA: Non-union of a stress fracture through the olecranon epiphyseal plate observed in an
adolescent baseball pitcher. J Bone Joint Surg Am 1977;59:264-265.
Rettig AC, Wurth TR, Mieling P: Nonunion of olecranon stress fractures in adolescent baseball pitchers: A
case series of 5 athletes. Am J Sports Med 2006;34:653-656.

Question 72
What is the most common physical finding in a patient with femoroacetabular impingement (FAI)?

1. Increased external rotation


2. Increased abduction
3. Decreased external rotation
4. Decreased flexion and internal rotation
5. Decreased adduction
DISCUSSION: A loss of flexion and internal rotation are hallmarks of FAI. With the hip flexed 90 degrees,
maximal internal rotation testing is also known as the anterior impingement test, causing deep groin pain and

PREFERRED RESPONSE: 1

124
2010 Sports Medicine Examination Answer Book • 57

reproduction of symptoms. Occasionally, a posterior impingement test will be positive with extension and
external rotation. There are a variety of causes of FAI; however, the pathology limits motion as the femur
(cam) and acetabulum (pincer) contact one another. Also, only one location needs to be present, such as cam-
type or pincer-type versus both cam-pincer lesions to cause symptoms.

REFERENCES: Philippon MJ, Stubbs AJ, Schenker ML, et al: Arthroscopic management of femoroac-
etabular impingement: Osteoplasty technique and literature review. Am J Sports Med 2007;35:1571 -1580,
Siebenrock KA, Schoeniger R, Ganz R: Anterior femoroacetabular impingement due to acetabular retro-
version: Treatment with periacetabular osteotomy. J Bone Joint Surg Am 2003;85:278-286. Kubiak-Langer
M, Tannast M, Murphy SB, et al: Range of motion in anterior femoroacetabular impingement. Clin Orthop
Relat Res 2007;458:117-124.

Question 73
Which of the following diseases has documented transmission by allograft tissue transplantation in the last 20
years?

1. Tuberculosis
2. Hepatitis B
3. HIV
4. West Nile virus
5. Clostridium

PREFERRED RESPONSE: 5

DISCUSSION: The only reported cases of HIV transmission with tissue transplantation occurred more than
20 years ago. The only reported cases of tuberculosis and hepatitis B occurred more than 50 years ago. The
donor-associated clostridium infection occurred in 2001. The facility was not AATB-accredited (American
Association of Tissue Banks) and the local A ATB facility refused the graft. It is necessary for the surgeon
using the allograft tissue to be aware of the current status of tissue regulation, and procurement and
processing procedures.

REFERENCES: McAllister DR, Joyce MJ, Mann BJ, et al: Allograft update: The current status of tissue
regulation, procurement, processing, and sterilization. Am J Sports Med 2007;35:2148-2158.
Safety of tissue transplants. American Association of Tissue Banks, 2006.
Question 74
Which of the following types of intra-articular pathology is associated with lateral meniscal cysts?

1. Discoid meniscus
2. Posterolateral comer injury
3. Vertical meniscal tears
4. Middle third lateral meniscal tears
5. Popliteus tendon tears

125
58 • American Academy of Orthopaedic S urgeons

PREFERRED RESPONSE: 4

DISCUSSION: Lateral meniscal cysts often arise from myxoid degeneration that progresses from the meniscal
center and then outside the meniscus. Horizontal cleavage tears are commonly associated with the condition.
Cysts of the lateral meniscus are most commonly the consequence of a tear located in the medial third. If the
tear communicates with the joint, arthroscopic partial meniscectomy and cyst decompression are indicated. If
the tear does not open into the joint, arthroscopy should be followed by an open cystectomy.

REFERENCES: Hulet C, Souquet D, Alexandre P, et al: Arthroscopic treatment of 105 lateral meniscal cysts
with 5-year average follow-up. Arthroscopy 2004;20:831-836.
Ferrer-Roca O, Vilalta C: Lesions of the meniscus: Part I. Macroscopic and histologic findings. Clin Orthop
Relat Res 1980;146:289-300.
Ferrer-Roca O, Vilalta C: Lesions of the meniscus: Part II. Horizontal cleavages and lateral cysts. Clin Orthop
Relat Res 1980:146:301-307.

Figure 75

Question 75
A 20-year-old male military recruit reports a 5-day history of progressive deep groin pain that is made
worse with weight-bearing activities and running. His initial coronal T2-weighted MRI scan is shown in
Figure 75. His initial treatment should consist of which of the following?

1. Bed rest with skeletal traction (distal femur traction pin)


2. Calcium supplements
3. Crutches with protected weight bearing
4. Open reduction and internal fixation
5. Pulsed ultrasound treatment
PREFERRED RESPONSE: 3

DISCUSSION: The MRI scan findings and patient history demonstrate a compression-sided femoral neck
stress fracture. The stress fracture on the coronal MRI scan involves about one third of the width of the
femoral neck. Surgical treatment would be recommended for tension-sided fractures of the femoral neck. The
most appropriate initial treatment is protected weight bearing, with close examination and imaging follow-up.
Skeletal traction is not currently used for nondisplaced femoral neck stress fractures, and there is no indication

126
2010 Sports Medicine Examination Answer Book • 59

for open reduction and internal fixation. There is no documented role for pulsed ultrasound or calcium
supplements in the acute treatment of stress fractures.

REFERENCES: Shin AY, Morin WD, Gorman JD, et al: The superiority of magnetic resonance imaging in
differentiating the cause of hip pain in endurance athletes. Am J Sports Med 1996;24:168-176.
Pihlajamaki HK, Ruohola JP, Weckstrom M, et al: Long-term outcome of undisplaced fatigue fractures of the
femoral neck in young male adults. J Bone Joint Surg Br 2006;88:1574-1579.
Valimaki VV, Alfthan H, Lehmuskallio E, et al: Risk factors for clinical stress fractures in male military
recruits: A prospective cohort study. Bone 2005;37:267-273.

127
60 • American Academy of Orthopaedic S urgeons

Question 76
A college athlete has a knee injury requiring surgery. He has acne, gynecomastia, and well-developed
muscles related to the use of anabolic steroids. What association with steroid use is concerning for
surgery and anesthesia?

1. Fluid and electrolyte imbalance


2. Increased bleeding time
3. Impaired liver function
4. Lowered oxygen requirements
5. Splenomegaly
PREFERRED RESPONSE: 1

DISCUSSION: Anabolic steroids increase procoagulant factors VII and IX and thromboxane, all of which
lead to hypercoagulability which would decrease bleeding time. Liver function is usually upregulated as oral
steroids induce hepatic enzymes and patients are therefore less sensitive to anesthetic agents. Anabolic
steroids have a mineralocorticoid effect and users frequently use diuretics to mask this effect. Both can lead to
fluid and electrolyte imbalances. Cardiovascular effects include hypertension, left ventricular hypertrophy,
impaired diastolic filling, and thrombosis. Large muscle mass and high calorie intake lead to high ventilatory
requirements caused by increased oxygen consumption and carbon dioxide production. Anabolic steroids
have no effect on the spleen.

REFERENCES: Kam PC, Yarrow M: Anabolic steroid abuse: Physiological and anesthetic considerations.
Anaesthesia 2005;60:685-692.
Ansell JE, Tiarks C, Fairchild VK: Coagulation abnormalities associated with the use of anabolic steroids.
Am Heart J 1993;125:367-371.

Figure 77

Question 77
Figure 77 shows the clinical photograph of a 21-year-old male ice hockey player who sustained a blow to
the jaw from another player’s hockey stick. Examination reveals an unstable jaw, mild bleeding with
exposed bone, and malocclusion. What is the most serious acute complication of this injury?

128
2010 Sports Medicine Examination Answer Book • 61

1. Blood loss
2. Airway obstruction
3. Cerebrovascular accident
4. Periodontal disease
5. Hearing loss

PREFERRED RESPONSE: 2

DISCUSSION: The most serious, acute complication of severe maxillofacial trauma is airway obstruction that
can result in early death. It is most likely to be associated with multiple mandibular fractures or combined
maxillary, mandibular, and nasal fractures as reported by Seyfer and Hansen and Rohrich and Shewmaker.
The mandible suspends the tongue anteriorly. When the mandible is fractured and the patient is supine, the
tongue falls posteriorly and obstructs the airway. Soft-tissue swelling around the injured oronasal structures
can also result in a loss of airway patency. Endotracheal or nasotracheal intubation is often impossible and a
surgical airway may often have to be created to prevent death by asphyxiation. Other injuries that may require
immediate attention include head or cervical spine injury and hemorrhage. A cerebrovascular accident is also
less common but is associated with injury to the common carotid artery or its branches. Periodontal disease is
generally a long-term complication from dental injuries. Hearing loss is not a common complication of dental
and facial trauma.

REFERENCES: Seyfer AE, Hansen JE: Facial trauma, in Moore EF, Feliciano DV, Maddox KL (eds):
Trauma, ed 5. New York, NY, McGraw-Hill, 2004, pp 423-444.
Rohrich RJ, Shewmake KB: Evolving concepts of craniomaxillofacial fracture management. Clin Plast
Surg 1992;19:1-10.
Question 78
An 18-year-old high school football player exits the field after making a tackle on the opening kickoff. He
reports “feeling out of it” and states that he has a headache. He does not recall any loss of consciousness and
has no amnesia. He is unable to list the months of the year in reverse order on questioning. He does not return
to the game and feels normal at the completion of the game. What is the most sensitive test in assessing
deficits after mild traumatic brain injury?

1. Head CT
2. MRI of the head
3. Neuropsychologic testing
4. Radiographs of the skull
5. Sideline assessment

PREFERRED RESPONSE: 3

DISCUSSION: Most imaging studies in mild traumatic brain injury will be normal. Neuropsychologic
testing is the most sensitive test in assessing mild deficits after traumatic brain injury. Sideline assessment is
important but less sensitive in assessing deficits. The precise role of neuropsychologic testing in determining
return to play has not been fully defined.

REFERENCES: Garrick JG (ed): Orthopaedic Knowledge Update: Sports Medicine 3. Rosemont, IL,
American Academy of Orthopaedic Surgeons, 2004, pp 37-38.

129
62 • American Academy of Orthopaedic Surgeons

Maroon JC, Lovell MR, Norwig J, et al: Cerebral concussion in athletics: Evaluation and
neuropsychological testing. Neurosurgery 2000;47:659-672.

Figure 79

Question 79
What is the primary function of the structure at the tip of the probe in Figure 79?

1. Internal tibial rotation


2. External tibial rotation
3. Posterior tibial translation
4. Anterior tibial translation
5. Femoral internal rotation
DISCUSSION: The structure shown in the figure is the popliteus tendon. This structure is a continuation of
the popliteus muscle belly and attaches more proximally through its hiatus in the lateral meniscus onto the
lateral femoral epicondyle anterior and distal to the insertion of the lateral collateral ligament. The popliteus
is a dynamic internal rotator of the tibia. The popliteus complex reinforces the posterior third of the lateral
capsule and plays a major role in the dynamic and static stabilization of the lateral tibia on the femur,
including restriction of external tibial rotation, posterior tibial translation, and varus rotation of the tibia.

REFERENCES: Veltri DM, Deng XH, Torzilli PA, et al: The role of the cruciate and posterolateral
ligaments in stability of the knee: A biomechanical study. Am J Sports Med 1995;23:436-443.
Seebacher JR, Inglis AE, Marshall JL, et al: The structure of the posterolateral aspect of the knee. J Bone
Joint Surg Am 1982;64:536-541.
LaPrade RF, Terry GC: Injuries to the posterolateral aspect of the knee: Association of anatomic injury
patterns with clinical instability. Am J Sports Med 1997;25:433-438.

Question 80
Which of the following findings helps to distinguish between stress fractures of the tibia and shin splints?

1. With shin splints, a bone scan shows the posterior tibial cortex in a diffuse, longitudinal orientation.
2. With tibial shin splints, the bone scan is more intense.
3. A more diffuse area of tenderness is seen in tibial stress fractures.
4. A three-phase bone scan is positive in all phases with shin splints, but only positive in delayed

PREFERRED RESPONSE: 1

130
2010 Sports Medicine Examination Answer Book • 63

images with tibial stress fractures.


5. After activity, pain persists longer with tibial stress fractures.

PREFERRED RESPONSE: 1

DISCUSSION: Anterior tibial pain can often be difficult to diagnose. A bone scan showing the tibial
cortex in a diffuse, longitudinal orientation is consistent with shin splints compared to a more discreet,
localized uptake more commonly seen with a stress fracture. Bone stress injuries are due to cyclical
overuse of the bone. They are relatively common in athletes and military recruits but are also seem in
otherwise healthy people who have recently started new or intensive physical activity. Diagnosis of bone
stress injuries is based on the patient’s history of increased physical activity and on imaging findings.
The general symptom of a bone stress injury is stress-related pain. Bone stress injuries are difficult to
diagnose based only on a clinical examination because the clinical symptoms may vary depending on the
phase of the pathophysiological spectrum in the bone stress injury. Imaging studies are needed to ensure an
early and exact diagnosis. If the diagnosis is made early, most bone stress injuries heal well without
complications.

REFERENCES: Mubarak SJ, Gould RN, Lee YF, et al: The medial tibial stress syndrome: A cause of shin
splints. Am J Sports Med 1982;10:201-205.
Knobloch K, Yoon U, Vogt PM: Acute and overuse injuries correlated to hours of training in master running
athletes. Foot Ankle Int 2008:29:671-676.
Kiuru MJ, Pihlajamaki HK, Ahovuo JA: Bone stress injuries. Acta Radiol 2004;45:317-326.

131
64 • American Academy of Orthopaedic S urgeons

Question 81
Histologic studies of surgically resected tissue in lateral epicondylitis demonstrate which of the following
findings?

1. Chondroblastic proliferation
2. Angiofibroblastic tendinosis
3. Significant active inflammation
4. Primarily calcium deposition
5. No normal tendon histology

PREFERRED RESPONSE: 2

DISCUSSION: The extensor carpi radialis brevis is most often cited as the anatomic location of pathology in
lateral epicondylitis. Histologic examination demonstrates noninflammatory tissue, primarily
angiofibroblastic tendinosis though normal tendon histology is also present. There is usually no evidence of
acute inflammation or chondroblastic tissue, or significant calcium deposition.

REFERENCES: Nirschl RP, Ashman ES: Tennis elbow tendinosis (epicondylit is). Instr Course Lect
2004;53:587-598.
Lo MY, Safran MR: Surgical treatment of lateral epicondylitis: A systematic review. Clin Orthop Relat Res
2007;463:98-106.
Calfee RP, Patel A, DaSilva MF, et al: Management of lateral epicondylitis: Current concepts. J Am Acad
Orthop Surg 2008;16:19-29.

Question 82
A 22-year-old male soccer player reports left hip and groin pain. He states that symptoms began before a
preseason tournament but have worsened steadily for the past 2 weeks. He denies any recent fever or
sickness and is otherwise healthy. Examination reveals tenderness over the symphysis pubis and pain with
resisted rectus abdominus testing. Radiographs are negative. What is the next step in the proper management
of this patient?

1. Rest, nonsteroidal anti-inflammatory drugs, rehabilitation, and gradual return to play


2. Aspiration of the symphysis pubis followed by an appropriate course of antibiotics
3. Referral to a general surgeon for hernia evaluation
4. Rigid plating across the symphysis to address instability
5. MRI evaluation of the symphysis

PREFERRED RESPONSE: 1

DISCUSSION: Appropriate management of osteitis pubis includes rest, nonsteroidal anti-inflammatory


drugs, directed rehabilitation, and gradual return to sports. Lack of fever or chills excludes osteomyelitis as a
source of pain. Examination with tenderness over the symphysis pubis and pain with resisted rectus
abdominus testing is consistent with osteitis pubis as opposed to a sports hernia, where a patient would be
tender in the abdomen, not the pubis. There is no symphyseal instability that would require symphyseal
plating.
REFERENCES: Fricker PA, Taunton JE, Ammann W: Osteitis pubis in athletes. Sports Med 1991 ;12:266-

132
2010 Sports Medicine Examination Answer Book * 65

279.
Williams PR, Thomas DP, Downes EM: Osteitis pubis and instability of the pubic symphysis: When
nonoperative measures fail. Am J Sports Med 2000;28:350-355.

Question 83
Closed chain kinetic exercises are differentiated from open chain exercises by which of the following?

1. Increased j oint shear


2. Maximally rehabilitate individual muscles
3. Achieve normal motion in all the joints of the kinetic chain
4. Maximize j oint di stracti on
5. The compressive nature of applied loads
PREFERRED RESPONSE: 5

DISCUSSION: Closed chain kinetic exercises confer a margin of safety and are protective of healing or
repaired tissues by the compressive nature of the applied forces. Closed chain kinetic exercise is associated
with decreased shear, translation, and distraction of the joints within the chain. Because of patterns of motion
with closed chain kinetic exercises, individual muscles may not be maximally strengthened or all joint
motion returned to normal. Closed chain kinetic exercises may be used earlier in the rehabilitation process.
REFERENCES: Kibler WB, Livingston B: Closed-chain rehabilitation for upper and lower extremities. J Am
Acad Orthop Surg 2001;9:412-421.
Garrick JG (ed): Orthopaedic Knowledge Update: Sports Medicine 3. Rosemont, IL, American Academy of
Orthopaedic Surgeons, 2004, pp 131-132.

Question 84
Endurance training stimulates which of the following physiologic adaptations in the athlete?

1. Selective hypertrophy of type II muscle fibers


2. Decreased concentration of Krebs cycle enzymes
3. Increased rate of glycogen depletion
4. Increased sympathetic nervous system activity
5. Increased storage and utilization of intramuscular lipids
DISCUSSION: Endurance training causes selective hypertrophy of type I muscle fibers. It stimulates an
increase in the enzymes of the Krebs cycle which increases the capacity for aerobic ATP resynthesis during
exercise. There is a decrease in the rate of glycogen depletion. Depletion of glycogen has been linked to

PREFERRED RESPONSE: 5

133
66 • American Academy of Orthopaedic S urgeons

fatigue during endurance exercise. Endurance training blunts the catecholamine response and may
contribute to the reduction in heart rate observed for the same exercise intensity following training. The
greater use of lipid reduces the contribution of carbohydrate to ATP resynthesis and preserves muscle
glycogen.

REFERENCES: Jones AM, Carter H: The effect of endurance training on parameters of aerobic fitness.
Sports Med 2000:29:373-386.
Spina RJ, Chi MM, Hopkins MG, et al: Mitochondrial enzymes increase in muscle in response to 7-10 days
of cycle exercise. J Appl Physiol 1996;80:2250-2254.
Kiens B, Essen-Gustavsson B, Christensen NJ, et al: Skeletal muscle substrate utilization during
submaximal exercise in man: Effect of endurance training. J Physiol 1993;469:459-478.

Question 85
Kinematic testing of patellofemoral motion demonstrates that malalignment that produces increased Q angle
causes a shift of the patella laterally in the trochlear groove and is most pronounced during what phase of
the flexion arc?

1. 0 to 15 degrees
2. 20 to 30 degrees
3. 40 to 90 degrees
4. 100 to 120 degrees
5. 130 to 140 degrees

PREFERRED RESPONSE: 3

DISCUSSION: Dynamic patellofemoral joint contact measurements on cadaveric knees with simulated
increased Q angle demonstrated that forces shifted to the lateral facet. The lateral shift in the patella was most
pronounced from 40 to 90 degrees of flexion. At lower degrees of flexion, the lateral shift was significantly
less. At higher degrees of flexion, the continued shift of the patella was not as pronounced.

REFERENCES: Ramappa AJ, Apreleva M, Harrold FR, et al: The effects of medialization and
anteromedialization of the tibial tubercle on patellofemoral mechanics and kinematics. Am J Sports Med
2006;34:749-756.
Huberti HH, Hayes WC: Patellofemoral contact pressure: The influence of q-angle and tendofemoral
contact. J Bone Joint Surg Am 1984;66:715-724.

134
2010 Sports Medicine Examination Answer Book • 235

Question 86
An otherwise healthy 25-year-old man underwent a right anterior cruciate ligament reconstruction with a
bone-patellar tendon-bone allograft. Routine preimplantation cultures of the allograft taken by the surgeon
were positive for coagulase-negative Staphylococcus 5 days postoperatively. The patient has exhibited no
evidence of clinical infection and his postoperative course has been uncomplicated during this time. What is
the ideal management of this patient?

1. Observation
2. Oral antibiotics for 6 weeks
3. IV antibiotics for 6 weeks
4. Arthroscopic irrigation and debridement with graft retention
5. Arthroscopic irrigation and debridement with graft removal

PREFERRED RESPONSE: 1

DISCUSSION: The incidence of preimplantation positive cultures of musculoskeletal allografts used for
anterior cruciate ligament reconstruction has varied between 4.8% and 13.3%. Interestingly, in none of the
studies evaluating this issue did any of the patients implanted with a “contaminated” graft develop a clinical
infection. The results of the current literature suggest that the treatment of low- virulence organisms is
unnecessary if no evidence of clinical infection exists. Preimplantation cultures do not appear to correlate with
clinical infection. Therefore, the routine culture of allograft tissue is not recommended.

REFERENCES: Diaz-de-Rada P, Barriga A, Barroso JL, et al: Positive culture in allograft ACL-
reconstruction: What to do? Knee Surg Sports Traumatol Arthrosc 2003; 11:219-222.
Guelich DR, Lowe WR, Wilson B: The routine culture of allograft tissue in anterior cruciate ligament
reconstruction. Am J Sports Med 2007;35:1495-1499.
Centeno JM, Woolf S, Reid JB III, et al: Do anterior cruciate ligament allograft culture results correlate with
clinical infections? Arthroscopy 2007;23:1100-1103.

135
72 • American Academy of Orthopaedic Surgeons

Figure 87a Figure 87b

Question 87
What is the most likely diagnosis based on the MRI findings shown in Figures 87a and 87b?

1. Anterior cruciate ligament (ACL) tear


2. Posterior cruciate ligament (PCL) tear
3. Lateral collateral (LCL) ligament tear
4. Patellar dislocation
5. Patellar tendon rupture
PREFERRED RESPONSE: 4

DISCUSSION: The MRI scans reveal increased signal in the medial facet of the patella and the anterior aspect
of the lateral femoral condyle. This pattern is typically seen in patients with acute patellar dislocations. In
patients with ACL tears, the bone bruise of the lateral femoral condyle is usually seen in the central portion at
the sulcus terminalis and the posterior half of the lateral tibial plateau and is not usually seen in the patella.
This pattern of bone bruising is not seen with patellar tendon ruptures, LCL tears, and PCL tears.

REFERENCES: Elias DA, White LM, Fithian DA: Acute lateral patellar dislocation at MR imaging: Injury
patterns of medial patellar soft-tissue restraints and osteochondral injuries of the inferomedial patella.
Radiology 2002;225:736-743.
Virolainen H, Visuri T, Kuusela T: Acute dislocation of the patella: MR findings. Radiology
1993;189:243-246.

Question 88
What is the primary goal of the initial (acute) rehabilitation phase of an overhead athlete’s shoulder?

1. Improve flexibility
2. Strengthen muscles
3. Enhance power and endurance
4. Regain neuromuscular control
5. Perform functional drills

PREFERRED RESPONSE: 1

136
2010 Sports Medicine Examination Answer Book • 73

DISCUSSION: The goal in the initial phase of shoulder rehabilitation is to improve flexibility, reestablish
baseline dynamic stability, normalize muscle balance, and restore proprioception. In the advanced
strengthening and final phase, the goals are to initiate aggressive strengthening drills, enhance power and
endurance, perform functional drills, and to gradually initiate throwing activities.

REFERENCES: Wilk KE, Meister K, Andrews JR: Current concepts in the rehabilitation of the overhead
throwing athlete. Am J Sports Med 2002;30:136-151.
Wilk KE, Arrigo C: Current concepts in the rehabilitation of the athletic shoulder. J Orthop Sports Phys Ther
1993;18:365-378.

Question 89
A 12-year-old gymnast has had elbow pain for 4 weeks. She denies any specific trauma to the elbow.
Examination reveals lateral pain and no instability on testing. Range of motion is as follows: 15 degrees, loss
of elbow extension, normal flexion, and normal pronation and supination. Radiographs reveal a 3- x 7-mm
radiolucency of the capitellum. A ^-weighted MRI scan reveals a single solitary lesion, and T2- weighted
images show no signal around the lesion. There are no intra-articular loose bodies. Appropriate management
should include which of the following?

1. Arthroscopic debridement of the elbow


2. Open repair of the lesion
3. Open biopsy of the lesion
4. Continued participation in gymnastics until symptoms worsen
5. No participation in gymnastics until symptoms resolve and motion recovers

PREFERRED RESPONSE: 5

DISCUSSION: This is a typical presentation for an osteochondral lesion of the capitellum. This patient is
young and has, by definition, a stable lesion and has excellent potential to heal this lesion with nonsurgical
management. However, the patient should stop her activities (gymnastics) to prevent further damage and the
possible development of an unstable lesion that might then necessitate surgery. Surgical procedures are
generally not necessary for the treatment of these lesions.

REFERENCES: Byrd JW, Jones KS: Arthroscopic surgery for isolated capitellar osteochondritis dissecans in
adolescent baseball players: Minimum three-year follow-up. Am J Sports Med 2002;30:474-478.
Cain EL Jr, Dugas JR, Wolf RS, et al: Elbow injuries in throwing athletes: A current concepts review. Am J
Sports Med 2003;31:621-635.
Question 90
Which of the following statements best describes labral tears in the hip?

1. They are unrelated to degenerative joint disease.


2. They lead to increased movement of the femur relative to the acetabulum.
3. They usually result from lesions of the ligamentum teres.
4. They only occur with abnormal bone morphology.

137
74 • American Academy of Orthopaedic S urgeons

5. They commonly occur in the posteroinferior quadrant of the hip.

PREFERRED RESPONSE: 2

DISCUSSION: Labral and chondral lesions are observed within the anterosuperior quadrant of the
acetabulum. Tearing of the labrum markedly reduces resistance to joint motion, leading to instability.
The most common associated lesions are chondral injuries. They can occur with or without abnormal bone
morphology. The etiology for labral tears can be from traumatic and degenerative causes, structural
abnormalities from femoroacetabular impingement, developmental abnormalities, and hip instability.

REFERENCES: Beck M, Kalhor M, Leunig M, et al: Hip morphology influences the pattern of damage to the
acetabular cartilage: Femoroacetabular impingement as a cause of early osteoarthritis of the hip. J Bone Joint
Surg Br 2005;87:1012-1018.
Ito K, Leunig M, Ganz R: Histopathologic features of the acetabular labrum in femoroacetabular
impingement. Clin Orthop Relat Res 2004;429:262-271.
Crawford MJ, Dy CJ, Alexander JW, et al: The 2007 Frank Stinchfield Award. The biomechanics of the hip
labrum and the stability of the hip. Clin Orthop Relat Res 2007;465:16-22.

Question 91
Which of the following most accurately approximates the estimated risk of a musculoskeletal allograft
containing the human immunodeficiency virus (HIV) despite adequate screening?

1. 1 in 600
2. 1 in 6,000
3. 1 in 60,000
4. 1 in 1,600,000
5. 1 in 6,000,000

PREFERRED RESPONSE: 4

DISCUSSION: The calculated risk of a musculoskeletal allograft containing HIV despite adequate screening
has been estimated to be approximately 1 in 1.6 million. This estimate is based on the risk of HIV in the
population, projected population estimates, and current methods of donor screening.

138
2010 Sports Medicine Examination Answer Book • 75

REFERENCES: McAllister D, Joyce M, Mann B, et al: Allograft update: The current status of tissue regulation,
procurement, processing, and sterilization. Am J Sports Med 2007;2148-2158.
Buck B, Malinin T: Human bone and tissue allografts: Preparation and safety. Clin Orthop Relat Res 1994;303:8 -
17.
Buck B, Malinin T, Brown M: Bone transplantation and human immunodeficiency virus: An estimate of risk of
acquired immunodeficiency syndrome (AIDS). Clin Orthop Relat Res 1989;240:129-136.

% * f
A i;

Figure 92

Question 92
Which of the following physical examination findings is most likely present in the condition producing the MRI
findings shown in Figure 92?

1. Valgus laxity at 30 degrees of knee flexion


2. Varus laxity at 30 degrees of knee flexion
3. Posterior drawer
4. Pivot shift
5. Patellar apprehension

PREFERRED RESPONSE: 4

DISCUSSION: The T2-weighted sagittal MRI scan shows the classic “bone bruise” pattern seen with an anterior
cruciate ligament (ACL) tear. These lesions are thought to represent subcortical trabecular hemorrhages and are
manifested as an increase in signal intensity on T2-weighted images and diminished signal intensity on
Trweighted images. They are classically located in the mid-portion of the lateral femoral condyle and posterior
aspect of the lateral tibial plateau. This is due to the fact that an ACL tear typically is the result of a valgus-
extemal rotation of the femur on the fixed tibia. This places most of the weight-bearing stress on the lateral
femoral condyle, which rotates laterally and impacts the posterior lip of the lateral tibial plateau. This may result
in an impaction fracture if the force is great enough, but more frequently causes merely a microfracture of the
involved subcortical trabeculae.

139
76 • American Academy of Orthopaedic S urgeons

REFERENCES: Vellet AP, Marks PH, Fowler PJ, et al: Occult posttraumatic osteochondral lesions of the
knee: Prevalence, classification, and short-term sequelae evaluated with MR imaging. Radiology
1991;178:271-276.
Cone R: Imaging sports-related injuries of the knee, in DeLee J, Drez D, Miller M (eds): DeLee & Drez’s
Orthopaedic Sports Medicine: Principles and Practice, ed 2. Philadelphia, PA, WB Saunders, 2003, vol 2, pp
1595-1652.

Question 93
What is the theoretical advantage of an open subpectoral technique of tenodesis of the long head of the
biceps tendon compared to arthroscopic soft-tissue tenodesis techniques?

1. Improved cosmesis
2. Simpler to perform in the lateral decubitus position
3. Shorter surgical time
4. Removal of the biceps tendon from the bicipital groove
5. Superior outcomes when compared to soft-tissue tenodesis in level I studies

PREFERRED RESPONSE: 4

DISCUSSION: A subpectoral biceps tenodesis requires an additional incision at the insertion of the pectoralis
major tendon on the humerus. This can be difficult to do in the lateral decubitus position and certainly this
technique does not decrease surgical time when compared to arthroscopic soft-tissue techniques. There have
been no level I studies comparing the two techniques; however, the theoretical advantage of the open biceps
subpectoral tenodesis is that the biceps tendon is removed from the bicipital groove, which may eliminate a
source of pain in the biceps tendon.

REFERENCES: Mazzocca AD, Rios CG, Romeo AA, et al: Subpectoral biceps tenodesis with
interference screw fixation. Arthroscopy 2005;21:896.
Osbahr DC, Diamond AB, Speer KP: The cosmetic appearance of the biceps muscle after long-head
tenotomy versus tenodesis. Arthroscopy 2002;18:483-487.

Figure 94

Question 94
If the structure marked by the tip of the probe in Figure 94 is repaired to the bony glenoid with suture anchors

140
2010 Sports Medicine Examination Answer Book • 77

during an arthroscopic stabilization procedure, what is the most likely result?

1. Loss of external rotation with the glenohumeral joint abducted 90 degrees


2. Loss of external rotation with the arm at the side of the body
3. Loss of internal rotation with the glenohumeral joint abducted 90 degrees
4. Loss of internal rotation up the back
5. Loss of flexion

PREFERRED RESPONSE: 2

DISCUSSION: The probe is on the middle glenohumeral ligament (MGHL), which, in this case, is a cordlike
and robust structure, commonly known as a Buford complex. The space between the bony glenoid and the
MGHL (in this case, a cord-like Buford complex) is a normal variant and should not be repaired or tightened
to the bony glenoid with a soft-tissue anchor or other repair. If this structure is inadvertently repaired, the most
common scenario is loss of external rotation with the arm at the side, as the MGHL/ Buford complex becomes
tight with the arm in this position. The loss of external rotation is more pronounced with the arm at the side
than abducted at 90 degrees as the MGHL/Buford complex becomes tighter with the arm at the side than
abducted.

REFERENCES: Williams MM, Snyder SJ, Buford D Jr: The Buford complex: The “cord-like” middle
glenohumeral ligament and absent anterosuperior labrum complex: A normal anatomic capsulolabral variant.
Arthroscopy 1994;10:241-247.
Ide J, Maeda S, Takagi K: Normal variations of the glenohumeral ligament complex: An anatomic study for
arthroscopic Bankart repair. Arthroscopy 2004;20:164-168.
Gerber C, Werner CM, Macy JC, et al: Effect of selective capsulorrhaphy on the passive range of motion of
the glenohumeral joint. J Bone Joint Surg Am 2003;85:48-55.
Question 95
What allograft has the highest antigenicity when used for ligament reconstruction about the knee?

1. Tibialis anterior used for anterior cruciate ligament (ACL) reconstruction


2. Tibialis anterior used for posterolateral reconstruction
3. Bone-patellar tendon-bone used for ACL reconstruction
4. Semitendinosus used for posterior cruciate ligament reconstruction
5. Semitendinosus used for medial collateral ligament reconstruction

PREFERRED RESPONSE: 3

DISCUSSION: Although theoretically the intra-articular environment is slightly more immune privileged,
the role of immunogenicity is related more to bone than soft tissue. Therefore, the bone-patellar tendon-
bone used for ACL reconstruction would have the highest risk of immunogenicity if storage techniques and
harvest techniques were similar. This also is true for bone plugs associated with meniscal allografts.

REFERENCES: Rodeo SA, Seneviratne A, Suzuki K, et al: Histological analysis of human meniscal
allografts: Apreliminary report. J Bone Joint Surg Am 2000;82:1071-1082.
West RV, Hamer CD: Graft selection in anterior cruciate ligament reconstruction. J Am Acad Orthop Surg

141
78 • American Academy of Orthopaedic S urgeons

2005;13:197-207.

Question 96
Which of the following rehabilitation methods has proven as effective as surgical treatment for the treatment
of patellar tendinopathy (jumper’s knee)?

1. Electrotherapy
2. Concentric training
3. Eccentric training
4. Massage
5. Taping

PREFERRED RESPONSE: 3

DISCUSSION: Common treatments for patellar tendinopathy include rest, ice, electrotherapy, massage,
taping and injection. None has been demonstrated to be effective. Eccentric training has proven to be as
effective as surgical treatment. Achilles insertional tendinopathy has also proven to respond to eccentric
training.

REFERENCES: Bahr R, Fossan B, Loken S, et al: Surgical treatment compared with eccentric training for
patellar tendinopathy (Jumper’s Knee): A randomized, controlled trial. J Bone Joint Surg Am 2006;88:1689-
1698.
Coleman BD, Khan KM, Maffulli N, et al: Studies of surgical outcome after patellar tendinopathy: Clinical
significance of methodological deficiencies and guidelines for future studies. Victorian Institute of Sport
Tendon Study Group. Scand J Med Sci Sports 2000;10:2-11.

142
2010 Sports Medicine Examination Answer Book • 243

Question 97
Which of the following clinical findings is most commonly present in a chronic exertional compartment
syndrome of the anterior compartment of the leg?

1. Absence of the dorsalis pedis pulse with exercise


2. Pain in the anterior compartment 30 minutes post-exercise
3. Anterior leg pain with passive dorsiflexion of the toes 30 minutes post-exercise
4. Intracompartmental pressure of 30 mm Hg at 1 minute post-exercise
5. Sensory loss of the plantar aspect of the foot with exercise

PREFERRED RESPONSE: 4

DISCUSSION: Chronic exertional compartment syndrome of the leg is characterized by pain (often
burning in nature) of the involved compartment(s) (typically anterior) that worsens with activity and
completely subsides within 15 minutes of activity cessation. A high index of suspicion is warranted for this
condition. Intracompartmental pressure thresholds considered diagnostic are a 1-minute postexercise
pressure of 30 mm Hg and a 5-minute post-exercise pressure of 20 mm Hg. This condition is not associated
with the classic findings of an acute compartment syndrome. In this particular example, anterior leg pain
with passive dorsiflexion of the toes and sensory loss of the plantar aspect of the foot would not be
expected with an exertional compartment syndrome of the anterior compartment.

REFERENCES: Pedowitz RA, Hargens AR, Mubarek SJ, et al: Modified criteria for the objective diagnosis of
chronic compartment syndrome of the leg. Am J Sports Med 1990;18:35-40.
Rorabeck CH, Fowler PJ, Nott L: The results of fasciotomy in the management of chronic exertional
compartment syndrome. Am J Sports Med 1988;16:224-227.

Question 98
Which of the following is the most cost-effective method of screening for idiopathic hypertrophic
cardiomyopathy?

1. Obtaining a history of chest pain, syncope, or family history of early cardiac death dur ing the initial
physical examination
2. Screening echocardiogram
3. Screening EKGs
4. Auscultation of the heart
5. Exercise stress test

PREFERRED RESPONSE: 1

143
80 • American Academy of Orthopaedic S urgeons

DISCUSSION: Idiopathic hypertrophic cardiomyopathy is the leading cause of sudden cardiac death in
athletes. The diagnosis is made with an echocardiogram; however, echocardiograms have not been shown to
be a cost-effective screening tool for this condition. The most cost-effective method of screening for this
condition includes obtaining a history of chest pain or syncope in the athlete, or a family history of early
cardiac death. It has been estimated that it would cost over $500,000 to diagnose one case of hypertrophic
cardiomyopathy if screening echocardiograms were used. Auscultation of the heart and heart sounds can be
normal in an athlete at rest with this condition, and a stress test will not help diagnose idiopathic
hypertrophic cardiomyopathy.

REFERENCES: Bader RS, Goldberg L, Sahn DJ: Risk of sudden death in young athletes: Which screening
strategies are appropriate? Pediatr Clin North Am 2004;51:1421 -1441.
Maron BJ, Thompson PD, Ackerman MJ, et al: Recommendations and considerations related to prepar-
ticipation screening for cardiovascular abnormalities in competitive athletes: 2007 update: A scientific
statement from the American Heart Association Council on Nutrition, Physical Activity, and Metabolism:
Endorsed by the American College of Cardiology Foundation. Circulation 2007;115:1643-1655.

Question 99
Which of the following can be seen in the heart of a well-conditioned athlete?

1. Decreased stroke volume


2. Decreased cardiac output
3. Decreased resting heart rate
4. Decreased ventricular wall thickness
5. Decreased vagal tone
PREFERRED RESPONSE: 3

DISCUSSION: The well-conditioned heart of an athlete leads to increased ventricular wall thickness which in
turn increases the amount of blood ejected from the heart per given stroke (stroke volume). The increased
parasympathetic (vagal) tone also leads to a lower (decreased) resting heart rate. Cardiac output is equal to
stroke volume x heart rate and is increased during exercise in a well-conditioned athlete.

REFERENCES: Basilico FC: Cardiovascular disease in athletes. Am J Sports Med 1999;27:108-121. Huston
TP, Puffer JC, Rodney WM: The athletic heart syndrome. N Engl J Med 1985;313:24-32.
An 18-year-old boxer sustained a blow to his right eye in a boxing match. Examination on the sideline
reveals hyphema, reduced visual acuity and color vision, and a visual field cut. What is the next step in
management?

1. Eye patch and ophthalmology evaluation in 2 days


2. Fluorescein eye stain
3. Emergent CT
4. High-dose systemic steroids
5. Observation

PREFERRED RESPONSE: 3

DISCUSSION: With the examination demonstrating reduced visual acuity and visual field changes,

144
2010 Sports Medicine Examination Answer Book • 81

Question 100
emergent CT is needed to look for traumatic optic neuropathy from direct or indirect trauma. The most
common mechanism is blunt facial trauma (78%), but penetrating trauma is also common (22%). The most
common etiologies are sports and motor vehicle accidents. Fluorescein eye stain would only be useful for
corneal abrasion or corneal foreign body. CT scans are often helpful for an orbital fracture, optic nerve
sheath hemorrhage, optical canal fractures, skull fractures, foreign bodies, nonorbital facial fractures, or
associated brain injuries. The presence of a fracture of the optic canal on a CT scan was a poor prognostic
sign in a recent series by Goldenberg and associates. The treatment is controversial. Although treatment
options include high-dose corticosteroids, retrobulbar steroid injection, optic canal decompression, and optic
sheath fenestration, there is no consensus as to the optimum treatment. It has been reported that treatment
does not alter the prognosis in children and adolescents. Only 29% to 44% of children and adolescent
patients had significant improvement in visual acuity. Hyphema is a collection of free blood in the anterior
chamber of the eye. It is the most common intraocular eye injury associated with sports as reported by Denyi
and associates, and occurs in 24% of injured eye cases. At the time of injury, it occurs as a haze in the
anterior chamber. An eye patch and ophthalmology evaluation in 2 days is inappropriate because timely
evaluation in this scenario is important. High-dose steroids are often used for this injury but not before a full
evaluation including a CT scan. Observation is not appropriate because the injury needs an urgent evaluation.

REFERENCES: Goldenberg-Cohen N, Miller NR, Repka MX: Traumatic optic neuropathy in children
and adolescents. J AAPOS 2004;8:20-27.
Steinsapir K.D, Seiff SR, Goldberg RA: Traumatic optic neuropathy: Where do we stand? Ophthal Plast
Reconstr Surg 2002;18:232-234.
Denyi RG, Pashby RC, Pashby JJ: Treatment of sports eye injuries, in Fu FH, Stone DA (eds): Sports
Injuries, ed 2. Philadelphia, PA, Lippincott, 2001, p 950.

145
American Academy of Orthopaedic Surgeons Your Source for

Lifelong Orthopaedic Learning

146
2010 Pediatric Orthopaedic Examination Answer Book • 7

Figure la Figure lb
Question 1
A 5-year-old boy has had a limp for the past 4 weeks with intermittent pain at the foot. He remains normally
active and has no history of trauma. He has no fevers, rashes, or swelling. Examination reveals tenderness at the
mid-dorsum of the foot medially. Radiographs are seen in Figures la and lb. Treatment should include which of
the following?

1. MRI of the foot with gadolinium


2. Open biopsy of the lesion
3. Needle aspiration and culture, followed by antibiotic treatment
4. Observation or an orthotic arch support
5. Steroid injection of the lesion

PREFERRED RESPONSE: 4

DISCUSSION: Osteochondrosis of the tarsal navicular is most commonly identified between the ages of 2 and 9
years. The condition is benign and self limited in nature. In patients with severe pain, a period of casting may be
warranted, but otherwise management usually consists of observation or a supportive orthotic.

REFERENCES: DiGiovanni CW, Patel A, Calfee R, et al: Osteonecrosis in the foot. J Am Acad Orthop Surg
2007;15:208-217.
Williams GA, Cowell HR: Kohler’s disease of the tarsal navicular. Clin Orthop Relat Res 1981 ;158:53- 58.

AL-Madena Copy

147
8 • American Academy of Orthopaedic S urgeons

Figure 2

Question 2
A 3-year-old girl has had pain and swelling in her left thigh for the past 3 weeks. Her mother states she has had
a temperature as high as 100.4 degrees F (38 degrees C) and a weight loss of 5 pounds. A CBC shows a WBC
count of 11,000/mm3, an erythrocyte sedimentation rate of 13 mm/h, and a C-reactive protein of 0.3. A
radiograph is shown in Figure 2. What is the next step in management?

1. Biopsy and culture of the lesion


2. MRI of the left femur
3. IV antibiotics for 6 weeks
4. Incision and drainage of the left femur
5. Repeat radiograph in 3 months

PREFERRED RESPONSE: 2

DISCUSSION: The history and laboratory studies indicate that this is not an infection. A lesion in this location and
in this age group is likely a Ewing’s sarcoma. The presentation is usually a painful mass. About 20% of patients
have a fever. The radiograph shows a typical mottled, permeative lesion with periosteal reaction. An MRI scan
should be obtained to further evaluate the soft-tissue mass. Staging of the lesion should take place before biopsy,
which should be done by the surgeon who would be performing the next stage of surgical treatment, ideally an
orthopaedic oncologist.

REFERENCES: Gibbs CP Jr, Weber K, Scarborough MT: Malignant bone tumors. Instr Course Lect
2002;51:413-428.
Meyer JS, Nadel HR, Marina N, et al: Imaging guidelines for children with Ewing sarcoma and osteosarcoma: A
report from the Children’s Oncology Group Bone Tumor Committee. Pediatr Blood Cancer 2008;51:163-170.
T

AL-Madena Copy

148
2010 Pediatric Orthopaedic Examination Answer Book • 9

Figure 3

Question 3
A 9-year-old girl has had bilateral knee and leg pain for the past 2 years. The family has noted increasing
deformity in both lower extremities. She is less than the fifth percentile for height. Examination reveals bilateral
femoral bowing, mild medial-lateral laxity of the knees, and the deformities shown in the radiograph seen in
Figure 3. What is the most likely diagnosis?

1. Renal osteodystrophy
2. Diastrophic dysplasia
3. Metaphyseal dysplasia
4. Osteogenesis imperfecta
5. Fibrous dysplasia
PREFERRED RESPONSE: 1

DISCUSSION: The widening, bowing, and cupping of the physes indicate some form of metabolic bone disease;
therefore, the most likely diagnosis is renal osteodystrophy. The age of onset makes X-linked hypophosphatemic
rickets less likely. The ground glass lesions and widening of the medullary canal characteristic of fibrous
dysplasia are not present. There are no fractures creating the deformities indicating osteogenesis imperfecta.
There is an asymmetry of the deformities that makes diastrophic dysplasia less likely.

REFERENCES: Goldberg MJ, Yassir W, Sadeghi-Nejad A: Clinical analysis of short stature. J Pediatr Orthop
2002;22:690-696.
Parmar VS, Stanitski DF, Stanitski CL: Interpretation of radiographs in a pediatric limb deformity practice: Do
radiologists contribute? J Pediatr Orthop 1999;19:732-734.
Question 4
Patients with slipped capital femoral epiphysis are more likely to experience a delay in definitive diagnosis if they
initially present to a physician reporting which of the following problems?

L Limp
2. Hip pain
3. Knee pain
4. Proximal thigh pain
5. Buttock pain

PREFERRED RESPONSE: 3

DISCUSSION: A delay in diagnosis of slipped capital femoral epiphysis (SCFE) can lead to significant worsening
of the deformity or even progression from a stable to an unstable SCFE. Those patients that report knee pain as their
primary complaint are most likely to experience significant delay. Other variables associated with this delay include
Medicaid insurance and stable SCFE.

REFERENCES: Kocher MS, Bishop JA, Weed B, et al: Delay in diagnosis of slipped capital femoral epiphysis.

AL-Madena Copy

149
10 • American Academy of Orthopaedic S urgeons

Pediatrics 2004;113:e322-e325.
Rahme D, Comley A, Foster B, et al: Consequences of diagnostic delays in slipped capital femoral epiphysis. J
Pediatr Orthop B 2006;15:93-97.

Question 5
Physiologic bowing of the lower extremities should spontaneously correct by what age?

1. 3 months
2. 6 months
3. 12 months
4. 36 months
5. 72 months

PREFERRED RESPONSE: 4

DISCUSSION: Physiologic bowing is common and benign. Bowing is typically symmetric, involves both the
femur and tibia, and is usually most prominent in toddlers. It usually resolves by 2 years of age but there is
great variability. By age 36 months, almost all children will correct spontaneously. In children with
physiologic bowing, the screening examination is typically normal and a family history is absent; therefore,
radiographs are not necessary. If the deformity has not resolved by age 2 years, an AP radiograph of the lower
limbs should be obtained. This provides documentation of the severity of the bowing, permits measurement of
the metaphyseal-diaphyseal angle and/or Langenskiold grade, and allows evaluation for conditions such as
rickets or bony dysplasia. No treatment is indicated for physiologic bowing.

REFERENCES: Abel MF (ed): Orthopaedic Knowledge Update: Pediatrics 3. Rosemont, IL, American Academy
of Orthopaedic Surgeons, 2006, p 7.
Salenius P, Vankka E: The development of the tibiofemoral angle in children. J Bone Joint Surg Am
JV : AL-Madena Copy
Question 6
A 2-year-old child refused to walk 3 days prior to being seen because of pain in the left hip. The pain has gradually
subsided and the child is now walking. He is afebrile and has full motion of the hips. Laboratory studies show a
normal CBC with differential and C-reactive protein. An ultrasound shows a joint effusion in the right hip. What is
the most likely diagnosis?

1. Juvenile inflammatory arthritis


2. Septic arthritis
3. Osteomyelitis of the femur
4. Leukemia
5. Toxic synovitis
PREFERRED RESPONSE: 5

DISCUSSION: The most likely diagnosis is toxic synovitis, and the normal C-reactive protein supports that
diagnosis. Juvenile inflammatory arthritis is extremely rare to present with hip involvement. The child most likely
does not have a bacterial infection because he has improved rapidly without treatment. A normal CBC with
differential precludes the diagnosis of leukemia.

REFERENCES: Herring JA (ed): Tachdjian’s Pediatric Orthopaedics, ed 4. Philadelphia, PA, WB Saunders, 2008,
pp 2068-2070.
Del Beccaro MA, Champoux AN, Bockers T, et al: Septic arthritis versus transient synovitis of the hip:
The value of screening laboratory tests. Ann Emerg Med 1992;21:1418-1422.
Kocher MS, Mandiga R, Zurakowski D, et al: Validation of a clinical prediction rule for the differentiation between

150
2010 Pediatric Orthopaedic Examination Answer Book • 11

septic arthritis and transient synovitis of the hip in children. J Bone Joint Surg Am 2004:86:1629-1635.

AL-Madena Copy

151
12 • American Academy of Orthopaedic S urgeons

Figure 7a Figure 7b

Question 7
A 14-year-old boy is lifting weights and feels a sudden pain in his back, associated with sciatica bilaterally. The
sciatica persists for several weeks. The radiograph shown in Figure 7a is negative, and the CT scan shown in Figure
7b is available for evaluation. An MRI scan is read as a disk bulge. Management should consist of

1. resection of the fragment through a microdiskectomy approach.


2. epidural steroid injections until symptoms improve.
3. laminectomy with surgical excision of the limbus fragment.
4. activity restrictions until the symptoms improve.
5. chiropractic manipulation.

PREFERRED RESPONSE: 3

DISCUSSION: A limbus or apophyseal fracture caused by heavy lifting or twisting is commonly seen in older
children and adolescents. Patients describe feeling a popping sensation and report radicular symptoms.
Radiographs usually are not sufficient to diagnose the injury. MRI or CT should be used to determine the exact
location of the fracture. Nonsurgical management is rarely successful. A wide laminectomy with surgical
excision of the limbus fragment is recommended if neurologic symptoms are present.

REFERENCE: Fischgrund JS (ed): Orthopedic Knowledge Update 9. Rosemont, IL, American Academy of
Orthopaedic Surgeons, 2008, p 694.

AL-Madena Copy

152
2010 Pediatric Orthopaedic Examination Answer Book • 13

Figure 8

Question 8
A 16-year-old football player reports the acute onset of pain in his left foot. An AP radiograph is shown in Figure
8. What treatment is most likely to result in successful healing for this injury?

1. Restricted activity
2. Short leg walking cast
3. Short leg cast with no weight bearing
4. Percutaneous fixation with a screw
5. Open reduction and internal fixation with bone grafting

PREFERRED RESPONSE: 4

DISCUSSION: Fractures in this region of the fifth metatarsal have been shown to be prone to delayed union and
nonunion and therefore are most reliably managed with internal fixation. Bone grafting is generally not required.

REFERENCES: Herrera-Soto JA, Scherb M, Duffy MF, et al: Fractures of the fifth metatarsal in children and
adolescents. J Pediatr Orthop 2007;27:427-431.
Fetzer GB, Wright RW: Metatarsal shaft fractures and fractures of the proximal fifth metatarsal. Clin Sports Med
2006;25:139-150.

Question 9
A 13-year-old girl with adolescent idiopathic scoliosis is otherwise healthy with a normal neurologic
examination and she began her menstrual cycle 3 months ago. Standing radiographs show a high left thoracic
curve from T1-T6 that measures 29 degrees, a right thoracic curve from T7-L1 that measures 65 degrees, and
a left lumbar curve from L1-L5 that measures 31 degrees, correcting to 12, 37, and 10 degrees, respectively,
on bending films. Her Risser sign is 1. What is the most appropriate management?

1. Bracing
2. Posterior spinal fusion of only the right thoracic curve
3. Posterior spinal fusion from T2-L4
4. Vertebral body stapling to halt progression of the curve
5. Anterior and posterior spinal fusion
PREFERRED RESPONSE: 2
AL-Madena Copy
DISCUSSION: The patient has typical adolescent idiopathic scoliosis with a right thoracic curve. This represents
a Lenke-IB curve pattern; therefore, only treatment of the thoracic curve is required. The proximal thoracic and
thoracolumbar curves are very flexible. The patient is Risser 1 and has just started her menstrual cycles; therefore,
she is at significant risk for further curve progression. Bracing is not appropriate for a curve of this magnitude and
will not halt the progression of this curve, nor will vertebral body stapling stop this curve. Vertebral body stapling
is sometimes useful in small thoracic curves of less than 35 degrees and skeletally immature patients. Anterior and

153
14 • American Academy of Orthopaedic S urgeons

posterior spinal fusion is not required because the patient has no other risk factors, such as neurofibromatosis nor
is she at risk for crankshaft. Anterior fusion is an option, but it is not listed.

REFERENCES: Lenke LG, Betz RR, Harmes J, et al: Adolescent idiopathic scoliosis: Anew classification to
determine extent of spinal arthrodesis. J Bone Joint Surg Am 2001;83:1169-1181.
Lenke LG, Betz RR, Haher TR, et al: Multisurgeon assessment of surgical decision making in adolescent
idiopathic scoliosis: Curve classification, operative approach, and fusion levels. Spine 2001;26:2347- 2353.

Question 10
The risk of progression with congenital kyphosis is greatest with which of the following?

1. Anterior unsegmented bar


2. Block vertebra
3. Posterior hemivertebra
4. Anterolateral bar and contralateral quadrant vertebrae
5. Butterfly vertebra

PREFERRED RESPONSE: 4

DISCUSSION: The risk of neurologic compromise associated with congenital kyphosis is normally secondary
to risk of progression. The classic study of the natural history of congenital spinal deformity by McMaster and
Singh confirms that an anterolateral bar with contralateral quadrant vertebrae has the greatest risk.

REFERENCES: McMaster MJ, Singh H: Natural history of congenital kyphosis and kyphoscoliosis: A study of
one hundred and twelve patients. J Bone Joint Surg Am 1999;81:1367-1383.
Herring JA (ed): Tachdjian’s Pediatric Orthopaedics, ed 4. Philadelphia, PA, WB Saunders, 2008, p 351.

AL-Madena Copy

154
2010 Pediatric Orthopaedic Examination Answer Book *15

Question 11
Achondroplasia is caused by an abnormality of which of the following?

1. Parathyroid-related protein (PTHrP)


2. Bone morphogenic protein 2 i
3. Transforming growth factor-B (TGF-B)
4. Fibroblast growth factor receptor 3 (FGFR3)
5. Insulin-like growth factor binding proteins

PREFERRED RESPONSE: 4

DISCUSSION: Achondroplasia results from mutation of the fibroblast growth factor receptor 3. Bone
morphogenic proteins are regulators of growth differentiation and morphogenic embryology. Anomalies of this
protein are seen in increasing defects in limbs distally. Parathyroid-related protein is seen in Jensen-type
metaphyseal chondrodysplasia. Transforming growth factors and the morphogenic proteins affect the production
of matrix.

REFERENCES: Leet Al, Chomey GS: The physis, in Cramer KE, Scherl SA, Einhom TA (eds): Orthopaedic
Surgery Essentials: Pediatrics. Philadelphia, PA, Lippincott Williams & Wilkins, 2004, pp 327-332.
van der Eerden BC, Karperian M, Wit JM: Systemic and local regulators of the growth plate. Endocr Rev
2003;24:782-801.

Figure 12
Question 12
A 15-year-old boy is seen for an injury to his left shoulder. Examination reveals that he has pain and deformity of
the proximal humerus. The skin in the area is intact and there is no neurovascular compromise involving the
extremity. An AP radiograph is seen in Figure 12. What is the most appropriate treatment?

1. Shoulder immobilizer
2. Hanging arm cast
3. Closed reduction and a shoulder spica cast
4. Closed reduction and percutaneous pin fixation
5. Hemiarthroplasty
DISCUSSION: In adolescents with proximal humeral physeal fractures and minimal remodeling potential, closed
reduction and pin fixation has been shown to be safe and results in excellent long-term shoulder function.
Satisfactory reduction is not likely to result from the use of a hanging arm cast or be maintained by a shoulder
immobilizer or a spica cast following closed reduction. Hemiarthroplasty is not indicated for this injury.

REFERENCES: Dobbs MB, Luhmann SL, Gordon JE, et al: Severely displaced proximal humeral epiphyseal
1
AL-Madena Co^PONSE: 4

155
16 • American Academy of Orthopaedic S urgeons

fractures. J Pediatr Orthop 2003;23:208-215.


Bahrs C, Zipplies S, Ochs BG, et al: Proximal humeral fractures in children and adolescents. J Pediatr Orthop
2009;29:238-242.

mm
Question 13
Figure 13a 1 ’
Figure 13b

A 12-year-old child falls from his bicycle and injures his right knee. Evaluation in the emergency department
reveals knee effusion and pain with extremes of range of motion. Radiographs are shown in Figures 13a and 13b.
Attempts at closed reduction are made and he is placed in a long leg cast with the knee flexed at 10 to 20 degrees.
At follow-up, repeat radiographs continue to show anterior displacement of the fracture. What structure is most
likely entrapped under the fragment?

1. Anterior fat pad


2. Anterior cruciate ligament
3. Posterior cruciate ligament
4. Anterior horn of the medial meniscus
5. Anterior horn of the lateral meniscus

PREFERRED RESPONSE: 4
DISCUSSION: Avulsion fractures of the tibial spine are a relatively rare injury in children. Historically, the most
common cause of this fracture was falls from bicycles, but with the increased participation in competitive sports,
the etiology is changing. Most fractures occur in children ages 8 to 14 years, and they typically present with a
painful hemarthrosis and refusal to bear weight. The Meyers and McKeever classification is based on degree of
displacement, where type I is minimally displaced, type II is anteriorly displaced with an intact posterior hinge,
and type III is completely displaced. The Ilia and Illb modifications have been added to account for fragment
comminution and rotation, respectively. Long leg casting is advocated for type I fractures, though there is debate
whether the knee should be maintained in full extension or in 10 to 20 degrees of flexion. Management of type II
and III fractures is much more controversial. Type II fractures can be treated closed if adequate reduction can be
achieved, but if not, surgical management is indicated. Surgery is also indicated for type III fractures, and results
of open versus arthroscopic procedures are similar long term. Kocher and associates examined 80 consecutive
skeletally immature patients with type II or III tibial eminence fractures that were treated surgically. They found
that the anterior hom of the medial meniscus was entrapped beneath the displaced fracture fragment in 36 of 80
cases, whereas the lateral meniscus was only entrapped in 1 of 80 cases. This is not to be confused with the data

AL-Madena Copy

156
2010 Pediatric Orthopaedic Examination Answer Book • 17

from Lowe and associates in JBJS 2002 where they found the lateral meniscus to be involved in blocking
reduction. This was not thought due to entrapment of the lateral meniscus. Rather, with the anterior cruciate
ligament and lateral meniscus still being attached to the avulsed fracture fragment, they felt the two structures were
pulling in opposite directions and therefore blocking reduction of the fragment.

REFERENCES: Falstie-Jensen S, Sondergard-Petersen PE: Incarceration of the meniscus in fractures of the


intercondylar eminence of the tibia in children. Injury 1984;15:236-238.
Kocher MS, Micheli LJ, Gerbino P, et al: Tibial eminence fractures in children: Prevalence of meniscal
entrapment. Am J Sports Med 2003;31:404-407.
Accousti WK, Willis RB: Tibial eminence fractures. Orthop Clin North Am 2003;34:365-375.
Lowe J, Chaimsky G, Freedman A, et al: The anatomy of tibial eminence fractures: arthroscopic observations
following failed closed reduction. J Bone Joint Surg Am 2002;84:1933-1938.

Figure 14a Figure 14b Figure 14c


Question 14
A 14-year-old boy underwent in situ screw fixation for a left slipped capital femoral epiphysis 8 months ago. He
noted 3 months of intermittent right hip pain but is presently asymptomatic. The last episode of pain was 2 days
prior to this office visit. He reports that he has pain approximately once a week over the past 3 months.
Examination of the right hip is normal, and includes pain-free internal rotation. Radiographs and an MRI scan are
shown in Figures 14a through 14c. Treatment should consist of which of the following?

1. In situ screw fixation of the right hip


2. Physical therapy
3. Limitation of activities and return to the clinic if pain persists
4. Biopsy of the femoral neck lesion
5. Irrigation and debridement of the right hip

PREFERRED RESPONSE: 1

DISCUSSION: The patient history is concerning for a pre-slip slipped capital femoral epiphysis (SCFE) of the
right hip. In one study, nearly 40% of patients with SCFE had bilateral involvement, and of that 40%, half
presented initially with a unilateral SCFE but had a subsequent SCFE on the contralateral limb. Radiographs are
normal, but the MRI scan shows increased signal about the proximal femoral physis. Treatment should include
prophylactic screw fixation of the right hip.

REFERENCES: Aronsson DD, Loder RT, Breur GJ, et al: Slipped capital femoral epiphysis: Current
concepts. J Am Acad Orthop Surg 2006;14:666-679.
Loder RT, Aronson DD, Greenfield ML: The epidemiology of bilateral slipped capital femoral epiphysis: A
study of children in Michigan. J Bone Joint Surg Am 1993;75:1141-1147.

AL-Madena Copy

157
18 • American Academy of Orthopaedic S urgeons

Loder RT: Controversies in slipped capital femoral epiphysis. Orthop Clin North Am 2006;37:211-221, vii.

Figure 15a Figure 15b

Question 15
When first seen in the emergency department, the patient with the injury seen in Figures 15a and 15b was not
able to extend the wrist or the thumb. What is the best initial management?

1. Closed reduction and casting, with expected nerve injury recovery with time
2. Closed reduction and percutaneous pinning, with expected nerve injury recovery with time
3. Immediate open reduction with internal fixation and exploration of the radial nerve
4. Immediate open reduction with internal fixation and exploration of the median nerve
5. Immediate open reduction with internal fixation and exploration of the ulnar nerve

PREFERRED RESPONSE: 2

DISCUSSION: The injury is a type 3 supracondylar humerus firaacture with a radial nerve injury. Most nerve
injuries associated with fractures recover spontaneously within 6 to 12 weeks. Complete recovery is expected
within 3 to 6 months. Closed reduction and percutaneous pinning is the recommended treatment for
supracondylar fractures of the elbow. Cast treatment for displaced fractures carries a higher risk of associated
compartment syndrome. There is no indication for exploring the radial nerve acutely. Open reduction is
necessary only if the closed reduction fails.

REFERENCES: Abel MF (ed): Orthopaedic Knowledge Update: Pediatrics 3. Rosemont, IL, American
Academy of Orthopaedic Surgeons, 2006, pp 406-409.
Campbell CC, Water PM, Emans JB, et al: Neurovascular injury and displacement in type III
supracondylar humerus fractures. J Pediatr Orthop 1995;15:47-52.

AL-Madena Copy

158
2010 Pediatric Orthopaedic Examination Answer Book • 19

Figure 16

Question 16
In a fracture such as the one shown in Figure 16 (Salter-Harris type I fracture of the distal femur), which of the
following best describes the location of the fracture?

1. The fracture occurs through the zone of hypertrophy of the physis.


2. The fracture occurs through the zone of proliferation of the physis.
3. The fracture is generally confined to the germinal zone, which explains the high rate of growth arrest in
these fractures.
4. The fracture generally propagates through multiple layers of the physis.
5. The fracture is generally confined to the zone of endochondral ossification.

PREFERRED RESPONSE: 4

DISCUSSION: The growth plate in the distal femur has an undulating topography, with prominences called
mammillary bodies that interdigitate with other portions of the physis to provide stability at the distal femur. A
typical distal femoral physeal fracture propagates through multiple layers of the growth plate as opposed to most
Salter-Harris type I physeal fractures.

REFERENCES: Smith DG, Geist RW, Cooperman DR: Microscopic examination of a naturally occurring epiphyseal
plate fracture. J Pediatr Orthop 1985;5:306-308.
Jaramillo D, Kammen BF, Shapiro F: Cartilaginous path of physeal fracture separations: Evaluation with MR
imaging: An experimental study with histologic correlation in rabbits. Radiology 2000;215:504-511.
Question 17
A 14-year-old boy has had a 3-month history of low back pain with no known trauma. The pain is worse with
activity and relieved by rest, although he does report difficulty with prolonged sitting in school.
The patient was on the football team but stopped participating because of the back pain during football
practice. He reports no history of radicular pain and denies any numbness, tingling, or weakness in the legs.

AL-Madena Copy

159
20 • American Academy of Orthopaedic S urgeons

Neurologic examination is normal. Back examination reveals slight tenderness over the lower back area but no
swelling or skin defects. Strength testing is 5 over 5 in the lower extremities and the straight leg raise test is
negative. Back range of motion is nearly full, but back extension is painful. The hamstrings are slightly tight.
Initial radiographs, including AP, lateral and oblique views, are negative. What is the best test to determine the
patient’s diagnosis?

1. Flexion and extension lateral radiographs


2. MRI
3. Myelogram
4. Diskogram
5. Bone scan with SPECT

PREFERRED RESPONSE: 5

DISCUSSION: A bone scan with SPECT is very sensitive and specific for spondylolysis not seen on initial
radiographs. MRI can sometimes visualize spondylolysis, but it is not as sensitive nor as specific as a bone scan
with SPECT. Flexion and extension views have no role in the evaluation of the patient who presents with classic
spondylolysis-type symptoms. The most sensitive physical examination finding is pain with back extension.
Oblique radiographs can be obtained, but they are not as sensitive or specific as a bone scan with SPECT. The
patient does not have any signs of a disk problem; therefore, an evaluation of the disk is not helpful.

REFERENCES: Hu SS, Tribus CB, Diab M, et al: Spondylolisthesis and spondylolysis. J Bone Joint Surg Am
2008;90:656-671.
Lawrence JP, Greene HS, Grauer JN: Back pain in athletes. J Am Acad Orthop Surg 2006;14:726-735.

Figure 18a Figure 18b

Question 18
A 14-year-old boy is involved in a motor vehicle accident and sustains the injury shown in Figures 18a and 18b.
What is the most likely diagnosis?

1. Hawkins type 1 talar neck fracture


2. Hawkins type II talar neck fracture
3. Hawkins type III talar neck fracture
4. Hawkins type IV talar neck fracture
5. Talar body fracture

PREFERRED RESPONSE: 2

DISCUSSION: Talar neck fractures are uncommon. In children younger than age 6 years, displacement is rare and

AL-Madena Copy

160
2010 Pediatric Orthopaedic Examination Answer Book • 21

closed treatment is usually successful in achieving union and avoiding osteonecrosis. In adolescence, however,
talar neck fractures should be treated as they are in adults. This fracture is displaced, and there is dislocation of the
subtalar joint. The tibiotalar and talonavicular joints remain reduced. In the classification originally created by
Hawkins and modified by Canale and Kelly, this would be a Hawkins type II, carrying a 20% to 50% risk of
osteonecrosis. The rate of osteonecrosis increases with the Hawkins grade. The presence of talar neck comminution
and open talar neck fractures are also risk factors for osteonecrosis after talar neck fracture.

REFERENCES: Vallier HA, Nork SE, Barei DP, et al: Talar neck fractures: Results and outcomes. J Bone Joint
Surg Am 2004;86:1616-1624.
Jensen I, Wester JU, Rasmussen F, et al: Prognosis of fracture of the talus in children: 21 year follow up of 14
cases. Acta Orthop Scand 1994;65:398-400.
Bucholz RW, Heckman JD, Court-Brown C (eds): Rockwood and Green’s Fractures in Adults, ed 6. Philadelphia,
PA, Lippincott Williams and Wilkins, 2006, pp 2249-2291.
Beaty JH, Kasser JR (eds): Rockwood and Green’s Fractures in Children, ed 6. Philadelphia, PA, Lippincott
Williams and Wilkins, 2006, pp 1129-1180.
Question 19
The clinical factors shown to most significantly predict the long-term outcome of Perthes disease of the hip include
which of the following?

1. Limb-length discrepancy, range of motion of the hip


2. Age at presentation, range of motion of the hip
3. Age at presentation, limb-length discrepancy
4. Range of motion, pain/limp for more than 6 months
5. Limb-length discrepancy, pain/limp for more than 6 months

PREFERRED RESPONSE: 2

DISCUSSION: Age at presentation and range of motion of the hip are the two most significant predictors of long-
term outcome. Younger patients and patients who maintain range of motion of the hip are more likely to have a
good outcome. In Herring’s study, children with a chronologic age of younger than 8 years or a bone age of less
than 6 years had significantly more favorable outcomes compared with older children. Limited hip range of motion
may be due to muscle spasm early on, or synovitis; but in late disease, it may reflect incongruity of the joint.
Classifications based on femoral head shape have also been correlated to prognosis. Significant shortening of the
affected hip is not common.

REFERENCES: Herring JA, Kim HT, Browne R: Legg-Calve-Perthes disease. Part II: Prospective multicenter
study of the effect of treatment on outcome. J Bone Joint Surg Am 2004;86:2121-2134. Herring JA, Kim HT,
Browne R: Legg-Calve-Perthes disease. Part I: Classification of radiographs with use of the modified lateral pillar
and Stulberg classifications. J Bone Joint Surg Am 2004;86:2103-2120. Skaggs DL, Tolo VT: Legg-Calve-Perthes
disease. J Am Acad Orthop Surg 1996;4:9-16.

Question 20
Pediatric flexor tendon injuries of the upper extremity differ from adult flexor tendon injuries in which of the
following ways?

1. Delayed presentation is not common.


2. A staged repair is never necessary.
AL-Madena Copy

161
22 • American Academy of Orthopaedic S urgeons

3. Six to eight weeks of postoperative immobilization is recommended.


4. Cooperation with occupational therapy can be difficult.
5. The use of Botulinum is contraindicated.
PREFERRED RESPONSE: 4

DISCUSSION: Pediatric flexor tendon injuries have several remarkable distinctions from those in adults.
Delayed presentation is more common in children, at times requiring staged flexor tendon reconstruction. Three
to four weeks of postoperative immobilization following acute repair is recommended in children as opposed to
early motion protocols used in adults. Temporary paralytic agents (botulinum toxin type A) have also been shown
to facilitate the rehabilitation phase of flexor tendon care in very young children.
REFERENCE: Fischgrund JS (ed): Orthopedic Knowledge Update 9. Rosemont, IL, American Academy of
Orthopaedic Surgeons, 2008, p 675.

Question 21
A 14-year-old boy is seen for back pain. Radiographic evaluation reveals a grade III isthmic spondylolisthesis.
What measurement is most useful in predicting the likelihood of progression?

1. Pelvic incidence
2. Slip angle
3. Sacral inclination
4. Lumbosacral joint angle
5. Sagittal rotation

PREFERRED RESPONSE: 2

DISCUSSION: Slip angle has been shown to be highly predictive of the risk for increased slippage in patients
with spondylolisthesis. None of the other radiographic parameters listed has been shown to be predictive of the
risk for increased slippage.

REFERENCES: Huang RP, Bohlman HH, Thompson GH, et al: Predictive value of pelvic incidence in progression
of spondylolisthesis. Spine 2003;28:2381-2385.
Mac-Thiong JM, Wang Z, de Guise JA, et al: Postural model of sagittal spino-pelvic alignment and its relevance for
lumbosacral developmental spondylolisthesis. Spine 2008;33:2316-2325.

AL-Madena Copy

162
2010 Pediatric Orthopaedic Examination Answer Book • 23

Figure 22a Figure 22b


Question 22
An 8-year-old girl has asymmetry on a forward bend test of the spine. She is asymptomatic
and has a normal clinical neurologic examination. Radiographs are shown in Figures 22a and 22b. What should be
the next step in her work-up?

1. MRI of the cervical thoracic lumbar spine


2. Supine side bending radiographs of the spine
3. Return to the clinic in 12 months with repeat radiographs
4. Anterior and posterior spinal fusion with instrumentation
5. Echocardiogram and renal ultrasound
PREFERRED RESPONSE: 1

DISCUSSION: There are several reasons to obtain an MRI of the entire spinal cord of this patient to evaluate for
abnormalities. These include her young age and the presence of a left-sided curve. For juvenile scoliosis patients
with more than a 20-degree Cobb angle, there is an approximately 20% prevalence of a neurologic abnormality.
Therefore, recommendations for work-up include an MRI scan of the entire spine.

REFERENCES: Gillingham BL, Fan RA, Akbamia BA: Early onset idiopathic scoliosis. J Am Acad Orthop Surg
2006;14:101-112.
Gupta P, Lenke LG, Bridwell KH: Incidence of neural axis abnormalities in infantile and juvenile patients with
spinal deformity: Is a magnetic resonance image screening necessary? Spine 1998;23:206-210.

AL-Madena Copy

163
24 • American Academy of Orthopaedic S urgeons

Figure 23a Figure 23b

Question 23
A 10-year-old girl returns for follow-up of a right Salter II distal radius fracture she sustained a year ago. She
reports pain and increasing deformity of her wrist. A radiograph and clinical photograph are shown in Figures 23a
and 23b. What is the next step in management?

1. CT scan to evaluate the extent of the growth arrest


2. Osteotomy of the radius and epiphysiodesis of the ulna
3. Physical therapy and further follow-up
4. Osteotomy of the radius and ulna
5. Bilateral epiphysiodesis of the radius and ulna

PREFERRED RESPONSE: 1

DISCUSSION: The radiograph and clinical photograph show a growth arrest of the distal radius on the right. There
is shortening and narrowing of the physis of the radius, and there is radial deviation of the hand. Greater than 2 cm
of growth still remains in the distal radius of a 10-year-old girl. Epiphysiodesis of both bones bilaterally would
leave the same deformity. The first step in treatment is to evaluate the extent of the growth arrest to see if the arrest
is resectable. Lengthening of the radius and epiphysiodesis of the ulna could restore the proper length and
alignment and would be the treatment of choice if the arrest was not resectable. Osteotomy of the radius and ulna
would not address the growth disturbance.

REFERENCES: Pritchett JW: Growth and development of the distal radius and ulna. J Pediatr Orthop
1996;16:575-577.
Waters PM, Bae DS, Montgomery KD: Surgical management of posttraumatic distal radial growth arrest in
adolescents. J Pediatr Orthop 2002;22:717-724.

Figure 24

Question 24
A 4-year-old girl has knee pain after a fall. Examination reveals tenderness about the proximal tibia with modest
deformity. She has no neurovascular deficits. A radiograph is seen in Figure 24. What should her parents be told?

AL-Madena Copy

164
2010 Pediatric Orthopaedic Examination Answer Book • 25

1. The injury will do well with treatment in a knee immobilizer.


2. The injury will do well with immobilization in a long leg cast.
3. Long leg cast immobilization and reduction is recommended but she may develop a deformity that
always requires surgical correction.
4. Long leg cast immobilization and reduction is recommended but she may develop a deformity;
however, if the deformity develops there is a significant chance it will resolve spontaneously.
5. Surgery is indicated to prevent deformity that may follow with this injury.
PREFERRED RESPONSE: 4

DISCUSSION: The patient has a so-called Cozen fracture, and she is at significant risk for a posttraumatic genu
valgum deformity. However, long-term studies have shown that when such a deformity occurs, it frequently
resolves spontaneously and therefore surgical intervention to try and prevent the deformity is not advised.

REFERENCES: Jordan SE, Alonso JE, Cook FF: The etiology of valgus angulation after metaphyseal
fractures of the tibia in children. J Pediatr Orthop 1987;7:450-457.
Tuten HR, Keeler KA, Gabos PG, et al: Posttraumatic tibia valga in children: A long-term follow-up note. J Bone
Joint Surg Am 1999;81:799-810.
Question 25
You are asked to consult on a 4-day-old neonate admitted because of failure to thrive. She has swelling of her left
shoulder. Examination reveals limited motion of her hips and left shoulder. Radiographs of the shoulder and pelvis
are negative. Laboratory studies show a WBC count of 24,000/mm3, an erythrocyte sedimentation rate of 50/h, and
C-reactive protein is 16.4. What is the next most appropriate step in management?

1. Ultrasound of the hip and shoulder


2. Bone scan
3. MRI of the shoulder
4. Pavlik harness
5. Excision and drainage

PREFERRED RESPONSE: 1

DISCUSSION: Ultrasound of both the hip and the shoulder can show the presence of septic arthritis and
osteomyelitis. Multiple sites of infection are common in neonates. A bone scan can be used to identify other
areas of involvement.

REFERENCES: Wong M, Isaacs D, Howman-Giles R, et al: Clinical and diagnostic features of osteomyelitis
occurring in the first three months of life. Pediatr Infect Dis J 1995;14:1047-1053.
Abel MF (ed): Orthopaedic Knowledge Update: Pediatrics 3. Rosemont, IL, American Academy of Orthopaedic
Surgeons, 2006, pp 57-73.

Question 26
Which of the following statements best describes the prognosis following a fracture of the distal femoral physis?

1. There is a high risk of nonunion.


2. There is a high risk of premature growth arrest that frequently causes deformity.
3. There is a high risk of premature growth arrest but it rarely causes deformity.

AL-Madena Copy

165
26 • American Academy of Orthopaedic S urgeons

4. There is a low risk of premature growth arrest but when it occurs it usually causes deformity.
5. There is a low risk of premature growth arrest and when it occurs it rarely causes deformity.

PREFERRED RESPONSE: 2

DISCUSSION: Displaced physeal fractures of the distal femur are at high risk for causing premature growth arrest
of the involved physis and subsequent deformity. Nonunion of these fractures is extremely rare.

REFERENCES: Arkader A, Warner WC Jr, Horn BD, et al: Predicting the outcome of physeal fractures of the
distal femur. J Pediatr Orthop 2007;27:703-708.
Thomson JD, Strieker SJ, Williams MM: Fractures of the distal femoral epiphyseal plate. J Pediatr Orthop
1995;15:474-478.

AL-Madena Copy

166
2010 Pediatric Orthopaedic Examination Answer Book • 27

Figure 27a Figure 27b

Question 27
Four days ago, a 13-year-old boy stubbed his toe on a chair while running barefoot through his home.
He received no treatment at the time. He is now seen at the orthopaedic clinic with the radiograph and clinical
photograph shown in Figures 27a and 27b. What is the next step in management?

1. Buddy taping to the adjacent toe and use of a hard-soled shoe for 2 weeks
2. Buddy taping to the adjacent toe and use of a cast extending to the tips of the toes for 3 weeks
3. Open reduction and internal fixation of the fracture, with irrigation of the wound and postoperative
antibiotics
4. Antibiotics and closed treatment of the fracture
5. Closed pinning of the phalanx fracture

PREFERRED RESPONSE: 3

DISCUSSION: The boy has a Seymour’s fracture of the toe. The germinal matrix of the nail bed is trapped in
the fracture site; thus this should be considered an open fracture. Ideally, it should be treated with open
reduction and internal fixation and use of antibiotics at the time of injury. Because this is a delayed
presentation, it is even more important to do a formal open reduction and a good irrigation and debridement,
followed by the use of postoperative antibiotics. Because the fracture has been displaced for several days,
overall management will be easier if the fracture reduction is maintained with pin fixation.

REFERENCE: Fischgrund JS (ed): Orthopedic Knowledge Update 9. Rosemont, IL. American Academy of
Orthopaedic Surgeons, 2008, p 737.

Question 28
A healthy 2-year-old boy falls from a swing and sustains a displaced midshaft femoral fracture with 1 cm of
shortening. What is the most appropriate treatment?

1. Pavlik harness
2. Skeletal traction for 3 weeks followed by a spica cast
3. Skin traction for 3 weeks followed by a spica cast
4. Closed reduction and spica casting
5. Closed reduction and an intramedullary pin

PREFERRED RESPONSE: 4
AL-Madena Copy

167
28 • American Academy of Orthopaedic S urgeons

DISCUSSION: For children between the ages of 1 and 6 years, closed reduction and early spica casting is
recommended. In some instances, associated injuries or body habitus may preclude cast treatment. Pavlik harness
treatment of femoral fractures is for infants younger than 1 year of age. Rarely is there an indication for traction.
Internal fixation is reserved in general for children older than age 6 years or with confounding factors.

REFERENCES: Abel MF (ed): Orthopaedic Knowledge Update: Pediatrics 3. Rosemont, IL, American Academy
of Orthopaedic Surgeons, 2006, pp 271-280.
Flynn JM, Schwend RM: Management of pediatric femoral shaft fractures. J Am Acad Orthop Surg 2004;12:347-
359.

Figure 29a Figure 29b

Question 29
A 10-year-old boy hit a tree with his sled and is seen in the emergency department with extreme left hip pain and
inability to ambulate. He has no history of pain in the left groin, thigh, or knee. Radiographs are seen in Figures
29a and 29b. What is the most common complication resulting from this injury?

1. Femoral artery intimal tear


2. Femoral nerve injury
3. Nonunion
4. Malunion
5. Osteonecrosis of the femoral head

PREFERRED RESPONSE: 5

DISCUSSION: The child has a type I hip fracture without associated dislocation. This is an acute hip fracture
through the proximal femoral physis, and can occur with or without associated dislocation. He had no prodrome of
hip or thigh pain and no femoral neck changes to indicate that this is an unstable slipped capital femoral epiphysis.
Osteonecrosis in these transepiphyseal hip fractures is the most common and most devastating complication. The
rate of osteonecrosis is most dependent on the initial displacement of the fracture. These fractures should be
treated emergently, and decompression of the hip joint is recommended by many authors.
REFERENCES: Moon ES, Mehlman CT: Risk factors for avascular necrosis after femoral neck fractures in
children: 25 Cincinnati cases and meta analysis of 360 cases. J Orthop Trauma 2006;20:323-329. Canale ST:
Fractures of the hip in children and adolescents. Orthop Clin North Am 1990;21:341-352.

AL-Madena Copy

168
2010 Pediatric Orthopaedic Examination Answer Book • 29

Question 30
A 10-day-old girl has decreased active motion of the left upper extremity. The mother reports a difficult vaginal
delivery with presumed shoulder dystocia. Examination shows full passive range of motion of the shoulder,
elbow, and wrist but only active flexion of the fingers and wrist. Factors predictive of a good outcome include
which of the following?

1. Breech delivery
2. Absence of an ipsilateral clavicle fracture
3. Homer’s sign and an APGAR score of 10 at 1 minute
4. Return of active biceps before 3 months and preservation of full passive shoulder range of motion
5. Absent Moro and Babinski reflexes
PREFERRED RESPONSE: 4

DISCUSSION: Return of active biceps before 3 months and preservation of full passive shoulder range of
motion are predictors of a good outcome. Breech delivery is usually associated with preganglionic injury.
Preganglionic injury can result in a Homer’s sign, which includes ptosis, myosis, and anhydrosis. Preganglionic
injuries are unlikely to recover. The Moro reflex is elicited by dropping a baby’s head a short distance and
observing active elbow extension and fanning of the fingers, followed by elbow flexion and crying. Absence of
the Moro reflex suggests a poor prognosis.

REFERENCES: Smith NC, Rowan P, Benson LJ, et al: Neonatal brachial plexus palsy: Outcome of absent
biceps function at three months of age. J Bone Joint Surg Am 2004;86:2163-2170.
Waters PM: Obstetric brachial plexus injuries: Evaluation and management. J Am Acad Orthop Surg
1997;5:205-214.

Figure 31
Question 31
A 9-month-old nonambulatory girl is seen in the emergency department with a fracture of her right forearm. The
mother says she fell from the changing table yesterday and continues to cry and not use her right arm. Radiographs
are shown in Figure 31. Treatment should consist of which of the following?

1. Closed reduction and a long arm cast


2. Closed reduction, a long arm cast, and a skeletal survey
3. Closed reduction, a long arm cast, a skeletal survey, and a referral to child protective services
4. Closed reduction and a long arm cast, a bone scan, and referral to child protective services
5. Closed reduction and a long arm cast, MRI of the brain, and a referral to child protective services
PREFERRED RESPONSE: 3

AL-Madena Copy

169
30 • American Academy of Orthopaedic S urgeons

DISCUSSION: The occurrence of a forearm fracture in a 9-month-old child has a greater than 50% chance that the
injury is due to child abuse. It is mandatory to report this to child protective services unless there is some
compelling reason that it is definitely not child abuse. In addition, a skeletal survey should be requested to look for
other injuries. A bone scan would show other injuries, but a skeletal survey is a more efficient way to evaluate for
other fractures. A MRI of the brain is not indicated unless fimdoscopic examination reveals an abnormality.

REFERENCES: Kocher MS, Kasser JR: Orthopaedic aspects of child abuse. J Am Acad Orthop Surg 2000 ;8:10-
20.
Chang DC, Knight V, Ziegfeld S, et al: The tip of the iceberg for child abuse: The critical roles of the pediatric
trauma service and its registry. J Trauma 2004;57:1189-1198.
Question 32
A child with an idiopathic clubfoot is successfully treated by the Ponseti method. The risk of recurrence of the
deformity is most dependent on which of the following factors?

1. Maternal age
2. Positive family history
3. Family’s compliance with bracing
4. The child’s age at walking
5. The child’s body mass index

PREFERRED RESPONSE: 3

DISCUSSION: The recurrence rate of clubfoot deformity after successful correction by the Ponseti method has
been shown to inversely correlate with reported brace compliance. Maternal age, walking age, and body mass
index have not been correlated to recurrence. A positive family history increases the risk of a child being bom
with a clubfoot but does not influence the recurrence rate.

REFERENCES: Dobbs MB, Rudzki JR, Purcell DB, et al: Factors predictive of outcome after use of the Ponseti
method for the treatment of idiopathic clubfeet. J Bone Joint Surg Am 2004;86:22-27.
Noonan KJ, Richards BS: Nonsurgical management of idiopathic clubfoot. J Am Acad Orthop Surg
2003;11:392-402.

AL-Madena Copy

170
2010 Pediatric Orthopaedic Examination Answer Book • 31

Figure 33

Question 33
An 18-month-old child was involved in a motor vehicle accident and sustained an isolated injury to the left upper
extremity. A radiograph is shown in Figure 33. What is the most appropriate management for this injury?

1. Hanging arm cast


2. Closed reduction with flexible intramedullary nail fixation
3. Coaptation splinting and bandaging the arm to the thorax
4. Closed reduction and external fixation
5. Locking plate fixation

PREFERRED RESPONSE: 3

DISCUSSION: Humeral shaft fractures in infants and young children heal rapidly and have excellent remodeling
potential. Appropriate treatment in this age group is immobilization with a coaptation splint and bandaging the
arm to the thorax for comfort. Internal fixation is appropriate in multiple trauma, and
external fixation may be useful when soft-tissue injury is extensive.
AL-Madena Copy
REFERENCES: Caviglia H, Garrido CP, Palazzi FF, et al: Pediatric fractures of the humerus. Clin Orthop Relat
Res 2005;432:49-56.
Husain SN, King EC, Young JL, et al: Remodeling of birth fractures of the humeral diaphysis. J Pediatr Orthop
2008;28:10-13.

Figure 34
Question 34
A 7-year-old girl with a known diagnosis of neurofibromatosis has neck pain and deformity. She has been
wearing a soft cervical collar for the past 2 months with mild relief of her symptoms. An MRI scan shows several
small neurofibromas on the left side of the cervical spine near the foramina at C6 and 7. A lateral cervical spine
radiograph is shown in Figure 34. What is the most appropriate management?

1. Anterior and posterior spinal fusion


2. Anterior spinal fusion
3. In situ posterior fusion
4. Halo traction correction and posterior fusion
5. Continued soft cervical collar treatment

PREFERRED RESPONSE: 1
AL-Madena Copy

171
32 • American Academy of Orthopaedic S urgeons

DISCUSSION: With a diagnosis of neurofibromatosis and severe kyphosis, anterior and posterior treatment is
needed to achieve correction and fusion. In situ fusion has a high failure rate with the kyphotic deformity and
even with traction, correction of the kyphosis is not expected. Anterior treatment alone may achieve correction,
but in neurofibromatosis only circumferential treatment has been shown to provide long-term stability.

REFERENCES: Crawford AH, Schorry EK: Neurofibromatosis update. J Pediatr Orthop 2006;26:4I3- 423.
MehJman CT, Al-Sayyad MJ, Crawford AH: Effectiveness of spinal release and halo-femoral traction in the
management of severe spinal deformity. J Pediatr Orthop 2004;24:667-673.

Figure 35

Question 35
A 10-month-old infant has no flexion at the elbows, mild flexion contractures at the wrist, a rigid clubfoot
deformity on the left foot, and a rigid rocker bottom deformity on the right foot. Examination of the patient’s hips
reveals limited abduction with 80 degrees of hip flexion/extension and full range of motion of the knees. A
radiograph of the pelvis is seen in Figure 35. What is the most appropriate treatment for the patient’s hip problem?

1. Preliminary skin traction followed by closed reduction under general anesthesia


2. Immediate closed reduction under anesthesia
3. Preliminary skeletal traction followed by closed reduction under general anesthesia
4. Bilateral open reduction performed through a medial approach
5. Bilateral open reduction performed through an anterior approach

PREFERRED RESPONSE: 4

DISCUSSION: The patient has arthrogryposis. Szoke and associates performed open reduction through a medial
approach on 40 hip dislocations in 26 patients with this condition and reported good results in 80% and fair results
in 12%. Due to the stiffness associated with this disorder, closed reduction with or without skin or skeletal traction
is not feasible. Open reduction through an anterior approach is reserved for older children.

REFERENCES: Szoke G, Staheli LT, Jaffe K., et al: Medial-approach open reduction of hip dislocations in
amyoplasia-type arthrogryposis. J Pediat Orthop 1996;16:127-130.
Staheli LT, Chew DE, Elliott JS, et al: Management of hip dislocations in children with arthrogryposis. J Pediatr
Orthop 1987;7:681-685.

AL-Madena Copy

172
2010 Pediatric Orthopaedic Examination Answer Book • 33

Figure 36a Figure 36b

Question 36
A 10-year-old boy tripped as he was running down a hill, felt a painful pop in his right knee, and was unable to bear
weight on the involved lower extremity. Examination reveals a tense effusion and an extensor lag of the right knee.
Figures 36a and 36b show AP and lateral radiographs. Management should consist of

1. long leg casting in 30 degrees of flexion for 6 weeks.


2. a long leg cast in full extension for 6 weeks.
3. knee arthroscopy to rule out internal derangement.
4. physical therapy for range of motion and quadriceps strengthening.
5. open reduction and internal fixation.

PREFERRED RESPONSE: 5

DISCUSSION: The examination and radiographs are consistent with a sleeve fracture of the patella, which is an
avulsion fracture of the distal pole of the patella with a disruption of the extensor mechanism. Treatment is open
reduction and internal fixation of the patella, and repair of the extensor mechanism.
The distal fragment can be much larger than it appears on the radiographs because it consists largely of cartilage.

REFERENCES: Wu CD, Huang SC, Liu TK: Sleeve fracture of the patella in children: A report of five cases. Am J
Sports Med 1991;19:525-528.
Grogan DP, Carey TP, Leffers D, et al: Avulsion fractures of the patella. J Pediatr Orthop 1990; 10:721 - 730.
Question 37
When addressing a proximal intertrochanteric or subtrochanteric fracture in a juvenile with open growth plates, the
arterial supply from what artery at the neck must be preserved?

1. Lateral femoral circumflex


2. Medial femoral circumflex
3. Superior gluteal
4. Inferior gluteal
5. Obturator

PREFERRED RESPONSE: 2

DISCUSSION: The medial femoral circumflex artery supplies blood to the femoral head. Its position along the
posterior-superior femoral neck places this structure at risk with intramedullary nailing of the femur. Therefore,
lateral entry through the greater trochanter is preferred when intramedullary fixation is performed.

AL-Madena Copy

173
34 • American Academy of Orthopaedic S urgeons

REFERENCES: Gordon JE, Swenning TA, Burd TA, et al: Proximal femoral radiographic changes after lateral
transtrochanteric intramedullary nail placement in children. J Bone Joint Surg Am 2003;85:1295- 1301.
Green NE, Swiontkowski MF: Skeletal Trauma in Children, ed 3. Philadelphia, PA, WB Saunders, 2003, pp 419-
424.

Question 38
What is the primary cause of the decreasing incidence of hemophilic arthropathy in the last 10 to 20 years?

1. Aggressive physical therapy of involved joints


2. Selective joint injections with steroids
3. Availability of and use of home factor treatment
4. Surgical debridement of involved joints
5. Use of splinting to prevent joint contractures

PREFERRED RESPONSE: 3

DISCUSSION: Home factor treatment has decreased the incidence of hemophilic arthropathy. Since 1992,
recombinant factor VIII was approved in the United States and can be safely used as prophylaxis or episodic
treatment of hemarthrosis. Essentially, recombinant factor VIII eliminated the risk of HIV and hepatitis virus
transmission associated with plasma-derived coagulation factors.

REFERENCES: Luck JV Jr, Silva M, Rodriguez-Merchan EC, et al: Hemophilic arthropathy. J Am Acad Orthop
Surg 2004;12:234-245.
Manco-Johnson MJ, Abshire TC, Shapiro AD. et al: Prophylaxis versus episodic treatment to prevent joint disease
in boys with severe hemophilia. N Engl J Med 2007;357:535-544.

AL-Madena Copy

174
2010 Pediatric Orthopaedic Examination Answer Book • 35

Question 39
The use of bisphosphonates in children with osteogenesis imperfecta is becoming more widely accepted as
treatment to improve quality of life and to decrease the risks of fracture. What is the mechanism by which
bisphosphonates work?

1. Inhibits osteoclasts
2. Stimulates osteoblasts
3. Increases gastrointestinal absorption of calcium
4. Decreases renal excretion of calcium

5. Acts as a transcription factor to increase production of type I collagen

PREFERRED RESPONSE: 1

DISCUSSION: The mechanism by which bisphosphonates act is by inhibiting osteoclasts. One mechanism of
bisphosphonates is to cause osteoclast apoptosis. Another mechanism of bisphosphonates is to disrupt the
cytoskeleton of osteoclasts, resulting in loss of the ruffled border. The uncoupling of bone resorption and bone
formation with decreased bone resorption results in increased bone mineralization. This translates into fewer
fractures in patients with osteogenesis imperfecta and improved quality of life.

REFERENCES: Bumei G, Vlad C, Georgescu I, et al: Osteogenesis imperfecta: Diagnosis and treatment.
J Am Acad Orthop Surg 2008;16:356-366.
Lin JT, Lane JM: Bisphosphonates. J Am Acad Orthop Surg 2003; 11:1-4.
Seikaly MG, Kopanati S, Salhab N, et al: Impact of alendronate on quality of life in children with
osteogenesis imperfecta. J Pediatr Orthop 2005;25:786-791.

Figure 40

Question 40
Evaluation of a nonambulatory 11-year-old girl with spinal muscular atrophy reveals mild scoliosis and full
painless range of motion in her hips. An AP radiograph of her pelvis is shown in Figure 40. What is the most
appropriate management for the hips?

1. Observation
2. Closed reduction and spica cast application
3. Abduction bracing
4. Open reduction and capsulorrhaphy of the hip
5. Total hip arthroplasty

PREFERRED RESPONSE: 1

DISCUSSION: Hip instability in nonambulatory children with spinal muscular atrophy has been shown to be an
infrequent cause of pain or disability; therefore, aggressive treatment generally is not indicated. Observation is
the most appropriate management.

175
36 • American Academy of Orthopaedic S urgeons

REFERENCES: Sporer SM, Smith BG: Hip dislocation in patients with spinal muscular atrophy. J Pediatr
Orthop 2003;123:10-14.
Thompson CE, Larsen LJ: Recurrent hip dislocation in intermediate spinal atrophy. J Pediatr Orthop
1990;10:638-641.

Question 41
The addition of which of the following food supplements may lead to a decrease in neural tube defects?

1. Vitamin D-1,25
2. Vitamin B-12
3. Niacin
4. Folic Acid
5. Thiamine

PREFERRED RESPONSE: 4

DISCUSSION: The use of folic acid in developed countries has lead to a decrease in neural tube defects.
The incidence of neural tube defects is increased in third world countries.
AL-Madena Copy
REFERENCES: Abel MF (ed): Orthopaedic Knowledge Update: Pediatrics 3. Rosemont, IL, American
Academy of Orthopaedic Surgeons, 2006, pp 111-122.
Lemke L, Dias L: Spina bifida, in Cramer KE, Scherl SA, Einhom TA (eds): Orthopaedic Surgery Essentials:
Pediatrics. Philadelphia, PA, Lippincott Williams & Wilkins, 2004, pp 203-210.

Figure 42

Question 42
A 5-year-old boy had a 1-week history of left hip pain and a limp that resolved 5 weeks prior to his office visit.
Examination demonstrates a pain-free and symmetric range of motion. A radiograph is seen in Figure 42. What
is the next step in management?

1. Physical therapy for range of motion and strengthening of the hips


2. Hip abduction brace wear
3. Left Salter pelvic osteotomy
4. Limitations of activities and observation
5. Radiographs of the knees and spine

PREFERRED RESPONSE: 5

DISCUSSION: Whereas bilateral Perthes of the hips occurs in 11% of cases, in patients with symmetric
changes/stages, other diagnoses must be considered such as Meyers dysplasia. Multiple epiphyseal dysplasia is
mostAL-Madena
readily diagnosed
Copy by evaluation of other radiographs, in particular of the knee and, if confirmatory, of the

176
2010 Pediatric Orthopaedic Examination Answer Book • 37

spine to assess for spondyloepiphyseal dysplasia.

REFERENCES: Herring JA (ed): Tachdjian’s Pediatric Orthopaedics, ed 4. Philadelphia, PA, WB Saunders,


2008, pp 806-810.
Hesse B, Kohler G: Does it always have to be Perthes’ disease? What is epiphyseal dysplasia? Clin Orthop Relat
Res 2003;414:219-227.
Question 43
Which of the following is a characteristic of odontoid fractures in children?

1. Usually occur in the body of C2


2. Are reduced by gentle cervical flexion
3. Frequently progress to nonunion
4. Almost always occur at the basilar synchondrosis
5. Are commonly associated with neurologic injury

PREFERRED RESPONSE: 4

DISCUSSION: Fracture of the odontoid process in children is usually caused by a fall, motor vehicle accident, or
minor trauma, and almost always occurs through the synchondrosis at the base of the dens. Neurologic deficits are
rare in isolated odontoid fractures in children. Closed reduction by neck extension and immobilization using a
cast, a brace, or halo traction for 6 to 8 weeks is usually sufficient to allow the fracture to heal.

REFERENCE: Fischgrund JS (ed): Orthopedic Knowledge Update 9. Rosemont, IL, American Academy of
Orthopaedic Surgeons, 2008, p 692.

Question 44
A 6-year-old boy is being treated for acute hematogeneous osteomyelitis of the distal femur with intravenous
antibiotics. The best method to determine the success or failure of initial treatment is by serial evaluations of
which of the following studies?

1. Radiographs
2. MRI
3. Erythrocyte sedimentation rate (ESR)
4. CBC with differential
5. C-reactive protein (CRP)

177
38 • American Academy of Orthopaedic S urgeons

PREFERRED RESPONSE: 5

DISCUSSION: Successful antibiotic treatment of acute osteomyelitis should lead to a rapid decline in the CRP.
The CRP is the most sensitive study to follow the treatment of osteomyelitis. The CRP should decline after 48 to
72 hours of appropriate treatment. CBC and ESR are helpful in initial evaluation and diagnosis, but remain
abnormal in the early phase of treatment regardless of response. Imaging studies are useful for surgical planning or
secondarily if the CRP remains elevated.

REFERENCES: Unkila-Kallio L. Kallio MJ, Eskola J, et al: Serum C-reactive protein, erythrocyte sedimentation
rate, and white blood cell count in acute hematogenous osteomyelitis of children. Pediatrics 1994;93:59-62.
Herring JA(ed): Tachdjian’s Pediatric Orthopaedics, ed 4. Philadelphia. PA. WB Saunders, 2008, pp 2090-2100.
AL-Madena Copy
Question 45
What is the most important predictor of functional outcome in patients with myelomeningocele?

1. Functional motor level


2. Sensory level
3. Dysplasia of the hip
4. Foot deformity
5. Hydrocephalus

PREFERRED RESPONSE: 1

DISCUSSION: The functional motor level of the patient is of prime importance in determining prognosis and
outcome. Patients with thoracic and upper lumbar motor levels will need wheelchairs or hip-knee- ankle-foot
orthoses to ambulate at all. Patients with midlumbar motor levels can be household or limited community
walkers, whereas children with low lumbar or sacral motor levels are likely to be able to walk in the community.

REFERENCES: Abel MF (ed): Orthopaedic Knowledge Update: Pediatrics 3. Rosemont, IL, American Academy
of Orthopaedic Surgeons, 2006, pp 117-120.
Swank M, Dias L: Myelomeningocele: A review of the orthopaedic aspects of 206 patients treated from birth
with no selection criteria. Dev Med Child Neurol 1992;34:1047-1052.

Figure 46a Figure 46b

Question 46
A 12-year-old boy reports a 6-week history of left hip pain. He denies any history of trauma or fever.
Examination reveals diminished internal rotation of both hips and discomfort with this manuever.
Radiographs are shown in Figures 46a and 46b. WTiat is the most appropriate management?

1. Surgical in situ pinning of the left hip


2. Surgical dislocation with reduction of the left slipped capital femoral epiphysis
AL-Madena Copy

178
2010 Pediatric Orthopaedic Examination Answer Book • 39

3. In situ pinning of bilateral hips


4. Bed rest

5. Application of a hip spica cast PREFERRED RESPONSE: 3

179
2010 Pediatric Orthopaedic Examination Answer Book • 40

DISCUSSION: The patient has left hip pain and clinical and radiographic evidence of a left slipped
capital femoral epiphysis. He also has open triradiate cartilage and a grade 1 slip on the right side that,
at the present time, is silent. The best treatment is pinning of bilateral slipped capital femoral epiphysis.
Reduction is not indicated because of the mild nature of both slips. Although prophylactic pinning of
the uninvolved contralateral hip is controversial, this patient shows a clinically silent grade 1 slip on the
right side.

REFERENCES: Puylaert D, Dimeglio A, Bentahar T: Staging puberty in slipped capital femoral


epiphysis: Importance of the triradiate cartilage. J Pediatr Orthop 2004;24:144-147.
Dewnany G, Radford P: Prophylactic contralateral fixation in slipped upper femoral epiphysis: Is it
safe? J Pediatr Orthop B 2005;14:429-433.

Question 47
Nutritional rickets in the US occurs more frequently in infants older than 6 months of age who do
not receive vitamin D supplementation and are

1. Caucasian and formula fed.


2. Caucasian and breast fed.
3. African American and formula fed.
4. African American and breast fed.
5. Asian and formula fed.

PREFERRED RESPONSE: 4

DISCUSSION: Numerous reports suggest an increased frequency of nutritional rickets in the US


in children with dark skin pigmentation who are breast fed past 6 months of age without vitamin
D supplementation. Nutritional rickets is rare in light-skinned children or those who are formula
fed.

REFERENCES: Herring JA (ed): Tachdjian’s Pediatric Orthopaedics, ed 4. Philadelphia, PA,


WB Saunders, 2008, p 1918.
Weisberg P, Scanlon KS, Li R, et al: Nutritional rickets among children in the United States: Review of
cases reported between 1986 and 2003. Am J Clin Nutr 2004;80:1697S-1705S.
Jacobsen ST, Hull CK, Crawford AH: Nutritional rickets. J Pediatr Orthop 1986;6:713-716.
Question 48
A 10-year-old girl has had nontraumatic swelling of the left knee for the past month. No other joints are
swollen and there is no history of fever, although the patient’s mother does recall the child having a
localized, but expanding “target-like” rash a few months ago when the family was vacationing in
Connecticut. Examination of the knee reveals moderate swelling with no tenderness and near full range
of motion. The child lacks perhaps the final 15 degrees of extension and the final 20 degrees of flexion.
Laboratory studies show a normal CBC count but the erythrocyte sedimentation rate is 35 mm/h (0-20
normal). Antinuclear antibody test and rheumatoid factor tests are negative. What is the most likely
diagnosis?

AL-Madena Copy

180
41 • American Academy of Orthopaedic Surgeons

1. Acute rheumatic fever


2. Septic arthritis
3. Transient synovitis
4. Lyme arthritis
5. Gout

PREFERRED RESPONSE: 4

DISCUSSION: Lyme arthritis is associated with a tick bite and is endemic to certain areas of
Connecticut. The earliest presentation of the disease is manifested by erythema migrans which is the
classic expanding rash that occurs at the site of the tick bite and can develop within 1 week to 1 month
after exposure. Joint involvement with manifestation of Lyme arthritis can occur months to years after
the initial infection. Most patients have single joint involvement with the knee being the most affected
site. Lyme arthritis is a low-grade inflammatory synovitis that can present with large and relatively
painless joint effusion. The most effective treatment is with a single 4-week course of oral amoxicillin or
doxycycline.

REFERENCES: Feder HM Jr: Lyme disease in children. Infect Dis Clin North Am 2008;22:315-
326. Gerber MA, Zemel LS, Shapiro ED: Lyme arthritis in children: Clinical epidemiology and
long-term outcomes. Pediatrics 1998;102:905-908.

Question 49
An 8-year-old girl was treated for a Salter-Harris type I fracture of the right distal femur 2 years ago. She
has symmetric knee flexion, extension, and frontal alignment to her contralateral knee. She has a 1-cm
limb-length discrepancy of the femur. She has always been in the 50th percentile for height and her
skeletal age matches her chronologic age. She has a complete physeal closure of the right distal femur.
What is the expected limb-length discrepancy at maturity?

1. 3 cm
2. 6 cm
3. 10 cm
4. 14 cm
5. 18 cm
PREFERRED RESPONSE: 2
DISCUSSION: The child has a near complete central physeal arrest of the distal femur. She will
develop worsening limb-length discrepancy. She is growing at the average rate for the population. The
distal femoral physis grows roughly at a rate of 9 mm/year. Girls finish their growth roughly at 14 years.
Thus, at maturity, the uninjured side will be 6.4 cm longer than the injured side. Since she has not
developed an angular deformity at this point and her arrest is central, she is unlikely to develop angular
deformity in any plane.

REFERENCES: Little DG, Nigo L, Aiona MD: Deficiencies of current methods for the timing of
epiphysiodesis. J Pediatr Orthop 1996;16:173-179.
Moseley CF: Assessment and prediction in leg-length discrepancy. Instr Course Lect 1989;38:325-330.

AL-Madena Copy

181
42 • American Academy of Orthopaedic Surgeons

Figure 50
Question 50
Figure 50 shows the radiographs of a 3-year-old child who has elbow pain. What is the most appropriate
treatment?

1. Collar and cuff for comfort


2. Long arm cast
3. Closed reduction and percutaneous pinning
4. Closed reduction and long arm cast
5. Open reduction and internal fixation

PREFERRED RESPONSE: 3

DISCUSSION: The radiographs show an extension type II supracondylar fracture with hyperextension
of the distal fragment. There is medial impaction of the fracture as well. If the fracture heals in this
aligmnent, the result will be cubitus varus with a loss of flexion of the elbow. Management should
consist of closed reduction that potentially converts the fracture to an unstable fracture. Percutaneous
pinning is recommended as cast treatment alone could lead to loss of reduction. Treatment in a cast with
hvperflexion may lead to compartment syndrome.
AL-Madena Copy
REFERENCES: Mehserle WL, Meehan PL: Treatment of displaced supracondylar fractures of the
humerus (type III) with closed reduction and percutaneous cross-pin fixation. J Pediatr Orthop
1991;11:705-711.
Herring JA: Upper extremity injuries: Supracondylar fractures of the humerus, in Herring JA (ed):
Tachdjian’s Pediatric Orthopaedics, ed 4. Philadelphia, PA, WB Saunders, 2008, pp 2470-2471.

Question 51
An 11-year-old girl is struck in the leg by a loaded sled while sledding and is seen in the emergency
department; she is reporting severe knee pain. Radiographs are read as normal. Examination reveals that
she is exquisitely tender over the proximal tibial physis. The neurovascular examination is normal. What
is the next step in management?

1. Splinting, admission, and frequent neurovascular checks


2. Cylinder cast and discharge
AL-Madena
3. Copy knee arthroscopy
Emergent

182
2010 Pediatric Orthopaedic Examination Answer Book • 43

4. Four-compartment calf fasciotomy


5. Non-weight-bearing, a knee immobilizer, and follow- up in 1 week

PREFERRED RESPONSE: 1
DISCUSSION: The anatomic lesion in this patient is not exactly defined, but she has most likely
sustained an injury about the knee. A Salter-Harris type I proximal tibial physeal fracture is likely. The
normal radiograph reading can be misleading because these injuries may displace and spontaneously
reduce. The child is at risk of compartment syndrome although she is currently not displaying signs of it.
Thus, even though this injury may seem trivial by radiographic findings, it should be treated like a knee
dislocation with a risk of late developing compartment syndrome. MRI or CT may be necessary to
define the injury. She does not require emergent treatment, but merits close observation for possible
compartment syndrome. Any of the possible injuries about the knee can be unstable and require internal
fixation after reduction.

REFERENCES: McGuigan JA, O’Reilly MJ, Nixon JR: Popliteal arterial thrombosis resulting from
disruption of the upper tibial epiphysis. Injury 1984;16:49-50.
Burkhart SS, Peterson HA: Fractures of the proximal tibial epiphysis. J Bone Joint Surg Am
1979;61:996- 1002.
Question 52
An otherwise healthy 4-week-old girl is noted on examination of the left hip to have a positive Ortolani
and Barlow test. She is placed in a Pavlik harness and returns for interval adjustments. At 3 weeks she
returns for a harness check and an ultrasound reveals a persistent hip dislocation. What is the next most
appropriate step in management?

1. Adjustment of the harness to maintain 80 degrees of abduction


2. Removal of the harness to avoid creating further deformity of the acetabulum
3. Removal of the harness and acceptance of the hip position without further treatment
4. Surgical open reduction of the hip within 2 weeks
5. Continued use of the harness and recheck in 2 to 3 weeks
PREFERRED RESPONSE: 2

DISCUSSION: The patient has failed to respond to Pavlik harness treatment. If use of the Pavlik harness
fails to maintain reduction at 2 weeks, use of the harness should be discontinued to avoid creating further
deformity of the acetabulum. Alternative treatments considered later include bracing, closed reduction,
and spica casting, or open reduction and spica casting. With a Pavlik harness, continued abduction and
hip flexion of the displaced hip may lead to posterolateral acetabular dysplasia.

AL-Madena Copy

183
44 • American Academy of Orthopaedic Surgeons

REFERENCES: Guille JT, Pizzutillo PD, MacEwen GD: Development dysplasia of the hip from birth
to six months. J Am Acad Orthop Surg 2000;8:232-242.
Hedequist D, Kasser J, Emans J: Use of an abduction brace for developmental dysplasia of the hip after
failure of Pavlik harness use. J Pediatr Orthop 2003;23:175-177.

AL-Madena Copy

184
2010 Pediatric Orthopaedic Examination
Answer Book • 45

Figure 53a Figure 53b Figure 53c Figure 53d

Question 53
Figures 53a through 53d show the clinical photographs and radiographs of the lower extremity of a
newborn male. Examination reveals this to be an isolated finding. The child otherwise has a normal
neurologic examination. The hips are stable and there are no spinal defects. What is the most appropriate
treatment at this time?

1. Symes amputation once ambulatory


2. Observation as the deformity will slowly resolve and the child will be left with a limb- length
discrepancy
3. Immediate osteotomy for correction of the deformity
4. Casting for correction of the deformity
5. Genetic testing for neurofibromatosis

PREFERRED RESPONSE: 2

DISCUSSION: The radiographs and clinical photographs reveal a child with posteromedial bowing of
the tibia. This is a congenital anomaly that is associated with a calcaneal valgus foot. It is a relatively
benign condition. The severity of the bow diminishes with time; however, the child will be left with a
limb-length discrepancy, usually in the range of 4 cm. The residual limb- length discrepancy presents the
greatest challenge for orthopaedic management. This, however, can usually be handled with limb-
lengthening techniques. Casting can be used for severe cases with unresolving significant contracture;
however, gradual spontaneous correction is usually the norm. This condition is quite different from
anterior lateral bowing that can be associated with neurofibromatosis and pathologic fracture or
pseudoarthrosis of the tibia.

REFERENCES: De Maio F, Corsi A, Roggini M, et al: Congenital unilateral posteromedial bowing of


the tibia and fibula: Insights regarding pathogenesis from prenatal pathology. A case report. J Bone Joint
Surg Am 2005;87:1601-1605.
Schoenecker PL, Rich MM: The lower extremity in pediatric orthopaedics, in Morrissy RT, Weinstein
SL (eds): Lovell and Winter’s Pediatric Orthopaedics, ed 6. Philadelphia, PA, Lippincott, Williams and
Wilkins, 2006, pp 1198-1200.

AL-Madena Copy

185
46 • American Academy of Orthopaedic Surgeons

Figure 54a Figure 54b


Question 54
A newborn male child has a left foot deformity as shown in Figures 54a and 54b. The family history and
birth history are unremarkable. The child is healthy and thriving, and examination of the spine, hips, and
neurologic system reveals normal findings. What is the best treatment for the foot deformity?

1. Stretching by the parents


2. Ankle-foot orthosis (AFO) and night splints
3. Anterior tibial tendon transfer
4. Casting with the Ponseti method
5. Short leg cast application

PREFERRED RESPONSE: 4

DISCUSSION: The foot shows all the classic signs of a clubfoot with hindfoot equinus, heel varus,
supination, and forefoot adduction. The Ponseti method is now well recognized as the best treatment for
idiopathic clubfoot. It calls for manipulation of the clubfoot on a weekly basis with the application of
long leg cast to slowly achieve correction. A percutaneous heel cord tenotomy is often required,
followed by an additional 3-week period of casting and eventual use of a foot abduction orthosis. AFO
night splints will not achieve any correction. Anterior tibial tendon transfer is sometimes performed for a
clubfoot with recurrence or if there is supination in the swing phase of gait. Short leg casts are not
sufficient to achieve full correction of a clubfoot.

REFERENCES: Herzenberg JE, Radler C, Bor N: Ponseti versus traditional methods of casting for
idiopathic clubfoot. J Pediatr Orthop 2002;22:517-521.
Morcuende JA, Dolan LA, Dietz FR, et al: Radical reduction in the rate of extensive corrective surgery
for clubfoot using the Ponseti method. Pediatrics 2004; 113:376-380.
Question 55
In Ewing’s sarcoma, neoplastic properties are thought to be related to a

1. environmental toxins.
2. a prior history of osteomyelitis.
3. a prior history of viral illness.
4. a prior history of trauma.
5. translocation of chromosomes.

PREFERRED RESPONSE: 5

DISCUSSION: In 95% of patients with Ewing’s sarcoma, there is a translocation, t(l 1:22). This results
in EWS/FLI-1 transcription factor that results in tumor cell proliferation. Other mechanisms causing

AL-Madena Copy

186
2010 Pediatric Orthopaedic Examination Answer Book • 47

tumor cell proliferation include inactivation of tumor suppressor genes, or activation of proto-oncogenes.

REFERENCES: Arndt CA, Crist WM: Common musculoskeletal tumors of childhood and adolescence.
N Engl J Med 1999;341:342-352.
Pierz KA, Womer RB, Dormans JP: Pediatric bone tumors: Osteosarcoma Ewing’s sarcoma, and
chondrosarcoma associated with multiple hereditary osteochondromatosis. J Pediatr Orthop 2001
;21:412- 418.

Question 56
Which of the following statements best describes what treatment is required for children with adolescent
tibia vara?

1. No treatment is necessary because spontaneous resolution is common.


2. Orthotic treatment is highly effective.
3. In addition to proximal tibial osteotomy, valgus deformity of the distal femur quently
develops to compensate for the proximal tibia vara and must be corrected with femoral
osteotomy.
4. In addition to proximal tibial osteotomy, varus deformity of the distal femur is frequent and must
be corrected by distal femoral osteotomy.
5. Surgical elevation of the medial tibial plateau is an integral part of all surgical interventions
for this condition.

PREFERRED RESPONSE: 4

DISCUSSION: Spontaneous resolution of adolescent tibia vara is uncommon. Orthotic treatment has not
been shown to be effective. Surgical elevation of the medial tibial plateau is a procedure that is
occasionally necessary in individuals with early onset Blount’s disease but is not indicated for
individuals with late onset Blount’s disease. Distal femoral varus deformity is commonly present and
must be addressed.

AL-Madena Copy

187
2010 Pediatric Orthopaedic Examination Answer Book • 48
REFERENCES: Gordon JE, King DJ, Luhmann SJ, et al: Femoral deformity in tibia vara. J Bone Joint
Surg Am 2006;88:380-386.
Gordon JE, Heidenreich FP, Carpenter CJ, et al: Comprehensive treatment of late-onset tibia vara. J
Bone Joint Surg Am 2005;87:1561-1570.

Question 57
The parents of a 14-year-old female soccer player are concerned about any future injury. They have been
advised that she has the potential to play for the US Olympic team. They are especially concerned about
the anterior cruciate ligament (ACL). What should you advise them?

1. ACL injuries are more common in men younger than 30 years of age.
2. ACL injuries are more common in women younger than 30 years of age.
3. ACL injuries are usually the result of contact sports.
4. The incidence of ACL injuries can be decreased by a neuromuscular training program.
5. ACL injuries are rarely associated with meniscal injury.

PREFERRED RESPONSE: 2

DISCUSSION: ACL injuries are five to eight times more common in young women. The highest
incidence is associated with basketball and soccer. These sports require rapid directional and rotational
changes. Use of neuromuscular training programs has not been associated with a decrease in ACL
injuries. It is recommended that there be more frequent rests. ACL injuries are commonly associated
with meniscal injury.

REFERENCES: Shea KG: ACL Injury: Epidemiology and Prevention Presented at Sports Related
Injuries in the Skeletally Mature Athlete. POSNA: One Day Course, 2008.
Millett PJ, Willis AA, Warren RF: Associated injuries in pediatric and adolescent anterior cruciate
ligament tears: Does a delay in treatment increase the risk of meniscal tears? Arthroscopy 2002; 18:955-
959.

Question 58
An 8-year-old boy weighing 70 lb sustains a displaced diaphyseal femur fracture and is treated with two
flexible retrograde intramedullary rods. What is the most common complication following treatment
with this technique?

1. Limb-length discrepancy
2. Mechanical irritation around the knee
3. Quadriceps weakness
4. Malunion
5. Patellofemoral pain
DISCUSSION: Flexible retrograde intramedullary nailing is now the preferred treatment for most length-
stable diaphyseal femur fractures in school-aged children. The most commonly described complication is
irritation about the knee at the rod insertion sites that resolves with rod removal. Limb-length discrepancy
and weakness have also been described at lower rates. Malunion or rod bending is usually related to
AL-Madena Cop,^SIONSE: 2

188
2010 Pediatric Orthopaedic Examination Answer Book • 49

placement of the rods in an unstable fracture pattern or in a larger patient.

REFERENCES: Flynn JM, Hresko T, Reynolds RA, et al: Titanium elastic nails for pediatric femur
fractures: A multicenter study of early results with analysis of complications. J Pediatr Orthop 2001
;21:4- 8 .
Flynn JM, Schwend RM: Management of pediatric femoral shaft fractures. J Am Acad Orthop Surg
2004;12:347-359.

1
1Ivi
rfl
Figure 59a
I
Figure 59b
\
"

Question 59
A 12-year-old girl with foot pain who has been diagnosed with hereditary motor sensory neuropathy is
seen for the foot deformity shown in Figure 59a. A “block test” is performed and shown in Figure 59b.
What is the most appropriate management for this patient?

1. Observation
2. Corrective shoes
3. Plantar release with first metatarsal osteotomy and possible tendon transfers
4. Calcaneal osteotomy
5. Triple arthrodesis

PREFERRED RESPONSE: 3

DISCUSSION: The hindfoot varus in this individual with a cavovarus deformity is nonstructural as
shown by the “block test”. Therefore, surgical procedures directed at correcting the hindfoot deformity
are not necessary. Observation is not in order and shoe modifications have not been shown to be effective
in managing this problem. The patient is symptomatic; therefore, the treatment of choice is plantar
release with first metatarsal osteotomy and possible tendon transfers.
REFERENCES: Paulos L, Coleman SS, Samuelson KM: Pes cavovarus: Review of a surgical approach
using selective soft-tissue procedures. J Bone Joint Surg Am 1980;62:942-953.
McCluskey WP, Lovell WW, Cummings RJ: The cavovarus foot deformity: Etiology and management.
Clin Orthop Relat Res 1989;247:27-37.
Ward CM, Dolan LA, Bennett DL, et al: Long-term results of reconstruction for treatment of a flexible
cavovarus foot in Charcot-Marie-Tooth disease. J Bone Joint Surg Am 2008;90:2631-2642.

AL-Madena Copy

189
50 • American Academy of Orthopaedic Surgeons

Figure 60a Figure 60b Figure 60c


Question 60
A 9-month-old boy fell down three steps onto his elbow. Immediate swelling was noted and he was taken
to the emergency department. Radiographs are shown in Figures 60a through 60c. Treatment should
consist of

1. a sling and early range of motion.


2. a splint in situ, with removal of the splint and range-of-motion exercises in 1 week.
3. closed reduction in the emergency department, followed by splinting and range-of-motion
exercises in 1 week.
4. closed reduction and percutaneous pin fixation in the operating room, with immobilization for
3 to 4 weeks.
5. open reduction and plate fixation in the operating room with early range of motion.

PREFERRED RESPONSE: 4

DISCUSSION: The radiographs show a transphyseal fracture separation of the distal humerus. Secondary
ossification centers are not yet apparent. The radius and ulna maintain a normal relationship to each
other, but not with the humerus. Commonly, as in these radiographs, displacement is medial. Elbow
dislocation is more often lateral displacement. These physeal injuries are often stable; however, anatomic
reduction and percutaneous pinning is recommended to prevent late deformity. Child abuse can be
associated with these fractures.

REFERENCES: Flynn JM, Sarwark JF, Waters PM, et al: The surgical management of pediatric fractures
of the upper extremity. Instr Course Lect 2003;52:635-645.
Oh CW, Park BC, Ihn JC, et al: Fracture separation of the distal humeral epiphysis in children younger
than three years old. J Pediatr Orthop 2000;20:173-176.

Figure 61
Question 61

AL-Madena Copy

190
2010 Pediatric Orthopaedic Examination Answer Book • 51

A 220-lb high school basketball player injured his knee while landing after a rebound. Figure 61 shows a
lateral view of the knee. This fracture is associated with which of the following complications?

1. Limb-length discrepency
2. Varus deformity of the proximal tibia
3. Compartment syndrome
4. Genu procurvatum
5. Ligamentous instability of the knee

PREFERRED RESPONSE: 3

DISCUSSION: There is a high incidence of compartment syndrome seen in type III tibial tubercle
fractures. Fasciotomy should be considered at the time of initial repair. Type III tibial tubercle fractures
extending through the joint are often associated with meniscal injuries, which must be repaired. Delayed
complications included recurvatum and refracture. Its association with Osgood-Schlatter’s disease has not
been proven. This is a fracture that occurs in later adolescence, so significant limb-length discrepencies
are unusual after this fracture.

REFERENCES: Ogden JA, Tross RB, Murphy MJ: Fracture of the tibial tuberosity in adolescents. J
Bone Joint Surg Am 1980;62:205-215.
Sponseller PE, Beaty JH: Fractures and dislocations about the knee, in Rockwood CA, Wilkins KE,
Beaty JH (eds): Fractures in Children, ed 4. Philadelphia, PA, Lippincott Raven, 1996, pp 1273-1281.
Question 62
In the swing phase of gait, the leg is carried forward by the momentum generated by which of the
following?

1. Ankle plantar flexors and hip flexors at terminal stance


2. Knee extensors and ankle dorsiflexors at terminal stance
3. Knee flexors and ankle dorsiflexors in early swing
4. Hip flexors and ankle dorsiflexors in early swing
5. Hip flexors and knee extensors in early swing

PREFERRED RESPONSE: 1

DISCUSSION: More muscle activity occurs during stance phase than during swing phase. During
stance phase, the muscles of the leg and foot work to stabilize the plantigrade foot. In swing phase,
momentum generated by the gastrocsoleus and hip flexors at terminal stance carries the leg forward.
Knee flexion in early swing and then extension at terminal swing occur passively. The main concentric
contraction that occurs during swing phase is that of the anterior tibialis, which dorsiflexes the foot for
easier clearance during swing and prepositions the foot for initial contact.

REFERENCES: Herring JA: Tachdjian’s Pediatric Orthopaedics, ed 4. Philadelphia, PA, WB


Saunders, 2008, pp 79-85.
Ounpuu S, Gage JR, Davis RB: Three-dimensional lower extremity joint kinetics in normal pediatric
gait. J Pediatr Orthop 1991;11:341-349.
Kadaba MP, Ramakrishnan HK, Wootten ME: Measurement of lower extremity kinematics during
level walking. J Orthop Res 1990;8:383-392.

AL-Madena Copy

191
52 • American Academy of Orthopaedic Surgeons

Question 63
Posttraumatic physeal arrest is most common at which of the following locations?

1. Proximal tibia
2. Proximal humerus
3. Distal radius
4. Distal humerus
5. Distal tibia
PREFERRED RESPONSE: 5

DISCUSSION: Posttraumatic physeal arrest occurs most commonly in the distal medial tibia. Using
MRI, Echlund and associates confirmed this finding. Arrest of the distal radius and proximal humerus
are rare after trauma. Traumatic injuries of the distal femoral and distal ulnar physis have a high
incidence of growth arrest as well.

REFERENCES: Ecklund K, Jaramillo D: Patterns of premature physeal arrest: MR imaging of 111


children. AJR Am J Roentgenol 2002; 178:967-972.
YAtotoartanadQpyiQter GN: Physeal bridge resection. J Am Acad Orthop Surg 2005; 13:4

Figure 64a Figure 64b

Question 64
The newborn foot deformity seen in Figures 64a and 64b should initially treated with

1. observation with possible stretching.


2. serial casting.
3. medial surgical release.
4. posterior medial surgical release.
5. dynamic ankle-foot orthosis.

PREFERRED RESPONSE: 1

DISCUSSION: Mild to moderate metatarsus adductus is best treated with observation and possible
passive stretching exercises because most of these feet will self correct. Numerous types of shoes,
braces, and splints have been devised but the efficacy of these have not been determined. Serial casting
is reserved for severe metatarsus adductus in the infant, although a medial surgical release may be
indicated if the deformity is symptomatic and persists beyond age 4 years.

REFERENCES: Abel MF (ed): Orthopaedic Knowledge Update: Pediatrics 3. Rosemont, IL, American
Academy AL-Madena Copy
of Orthopaedic Surgeons, 2006, pp 240-241.
Farsetti P, Weinstein SL, Ponseti IV: The Long-term functional and radiographic outcomes of untreated

192
2010 Pediatric Orthopaedic Examination Answer Book • 53

and non-operatively treated metatarsus adductus. J Bone Joint Surg Am 1994;76:257-265.


Question 65
A 4-year-old girl has been limping for the past 2 months. There is no history of trauma, previous injury,
fever, or other systemic complaints. Examination reveals a moderate right knee effusio n with a 10-degree
knee flexion contracture. What is the next most appropriate step in evaluation?

1. Arthroscopy
2. Antinuclear antibody
3. MRI
4. Bone scan
5. HLA-B27

PREFERRED RESPONSE: 2

DISCUSSION: The patient presents with juvenile idiopathic arthritis manifestations. The American
College of Rheumatology defines this as one or more joints involved with swelling of 6 weeks or longer.
A positive antinuclear antibody test would be diagnostic. Consideration should be made to have the
patient see an ophthalmologist for evaluation of possible uveitis. Although the patient could have Lyme
disease, that choice is not an option. The presence of an elevated antinuclear antibody by itself should not
necessarily be used for diagnosing arthritis; however, the test does have clinical utility as a screening test.
The frequency of a positive antinuclear antibody test is greatest in younger girls with oligoarticular
disease and carries an increased risk for anterior uveitis. Arthroscopy might be indicated if this patient
was presenting with a discoid meniscus, but there is no history of clicking, which is often one of the
classic signs of discoid meniscus. MRI would not be used to diagnose juvenile idiopathic arthritis, but
MRI would be useful to help diagnose discoid meniscus. A bone scan would show increased uptake in the
patient’s knee but again, this would not help diagnose her condition. HLA-B27 has no role in diagnosing
juvenile idiopathic arthritis, especially in females.

REFERENCES: Iesaka K, Kubiak EN, Bong LR, et al: Orthopaedic surgical management of hip and knee
involvement in patients with juvenile rheumatoid arthritis. Am J Orthop 2006;35:67-73.
Wright DA: Juvenile idiopathic arthritis, in Morrissey RT, Weinstein SL (eds): Lovell and Winter’s
Pediatric Orthopaedics, ed 6. Philadelphia PA, Lippincott Williams and Wilkins, 2006, pp 405-438.
Question 66
An 18-month-old girl is brought in by her parents because of concerns about intoeing, bowlegs, and
tripping and falling. Prenatal and birth history are otherwise unremarkable. The child’s growth and

AL-Madena Copy

193
54 • American Academy of Orthopaedic Surgeons

development appear to be normal and she has a normal neurologic exam, a straight spine with no
defects, and the hips are stable. Examination reveals hip internal rotation of 40 degrees and hip external
rotation of 60 degrees. The thigh-foot angle is internal 30 degrees. Feet are straight and supple. Gait is
characterized by intoeing with occasional tripping and falling. Based on these findings, what is the most
appropriate action?

1. No treatment because internal tibial torsion slowly resolves on its own


2. Immediate treatment with a Denis-Browne bar
3. Distal tibial osteotomies
4. Proximal femoral derotational osteotomies
5. Treatment with twister cables

PREFERRED RESPONSE: 1

DISCUSSION: The child has classic internal tibial torsion that is very commonly seen in younger
children who are just beginning to walk. The normal outcome is for slow resolution of this problem and
it seldom requires any treatment. Treatment with a Denis-Browne bar or with twister cables has not been
proven to be effective. Surgical treatment at this point is premature and clearly not indicated.

REFERENCES: Lincoln TL, Suen PW: Common rotational variations in children. J Am Acad Orthop
Surg 2003;11:312-320.
Staheli LT, Corbett M, Wyss C, et al: Lower-extremity rotational problems in children: Normal values
to guide management. J Bone Joint Surg Am 1985;67:39-47.

Figure 67

Question 67
A 5-year-old boy reports intermittent left elbow pain. History reveals that he injured his elbow 4 months
ago, but had no treatment. He is now using his arm normally but reports pain almost daily. Examination
reveals tenderness over the lateral epicondyle and a prominence is evident. Range of motion is from -5

AL-Madena Copy

194
2010 Pediatric Orthopaedic Examination Answer Book • 55

degrees to 120 degrees. Radiographs are shown in Figure 67. Management should include

1. open reduction and internal fixation.


2. cast immobilization.
3. percutaneous pin fixation.
4. observation, with follow-up in 3 months.
5. an MRI scan of the elbow.

PREFERRED RESPONSE: 1

DISCUSSION: The patient has a nonunion of the lateral condyle of the left humerus. Observation or cast
treatment at this stage is not likely to lead to healing of the fracture. MRI will not add any additional
information. Open reduction, with minimal posterior soft-tissue stripping, is recommended to establish
union of the fracture. Local or other bone graft may also be required. There are no studies that indicate
that the displaced fracture will heal with late percutaneous fixation.

REFERENCES: Wattenbarger JM, Gerardi J, Johnson CE: Late open reduction internal fixation of
lateral condyle fractures. J Pediatr Orthop 2002;223:94-398.
Flynn JC: Nonunion of slightly displaced fractures of the lateral humeral condyle in children: An update.
J Pediatr Orthop 1989;9:691-696.

Figure 68

Question 68
A 6-year-old girl sustains an ankle injury after falling on roller blades. An AP radiograph is shown in
Figure 68. Treatment should consist of which of the following?

1. Closed manipulation and a long leg cast


2. Closed manipulation and a short leg cast
3. Long leg cast without manipulation
4. Open reduction and internal fixation with a screw crossing the growth plate

AL-Madena Copy

195
56 • American Academy of Orthopaedic Surgeons

5. Open reduction and internal fixation with fixation parallel to the physis

PREFERRED RESPONSE: 5

DISCUSSION: The child has a Salter-Harris type IV injury involving both the growth plate and the
articular surface of the ankle. This injury pattern has a high risk of physeal arrest; open reduction and
internal fixation is indicated to realign the physis and joint surface. The best method of fixation to
avoid growth arrest is one that does not cross the physis. This is usually achieved by an epiphyseal
screw or pins parallel to the physis. If the metaphyseal fragment were large enough, a transverse
metaphyseal screw could be used. The incidence of growth arrest following physeal ankle injuries is
high and longterm follow- up is indicated.

REFERENCES: Cass JR, Peterson HA: Salter-Harris type-IV injuries of the distal tibial epiphyseal
growth plate, with emphasis on those involving the medial malleolus. J Bone Joint Surg Am
1983;65:1059-1070. Barmada A, Gaynor T, Mubarak SJ: Premature physeal closure following distal tibia
physeal fractures: A new radiographic predictor. J Pediatr Orthop 2003;23:733-739.
Question 69
What is the most likely reason open fractures tend to heal more slowly than closed fractures?

1. Loss of osteoinductive potential from the hematoma that is lost around the fracture
2. Introduction of foreign material
3. Subclinical infection
4. Loss of blood supply at the fracture site
5. Loss of soft-tissue coverage at the fracture site

PREFERRED RESPONSE: 1

DISCUSSION: In open fractures, the hematoma that forms beneath the periosteum and around the ends
of the fracture site is lost from the open wound. In addition, the irrigation process washes out the
hematoma that contains growth factors and cytokines from the platelets. While loss of blood supply at
the fracture site and soft-tissue coverage are important factors, the most important is loss of the factors
that initiate the inflammatory phase of fracture healing. Infection may also delay healing, but is less
common in this population.

REFERENCES: Buckwalter JA, Einhom TA, Simon SR (eds): Orthopaedic Basic Science: Biology and
Biomechanics of the Musculoskeletal System, ed 2. Rosemont, IL, American Academy of Orthopaedic
Surgeons, 2000, pp 377-381.
Green NE, Swiontkowski MF (eds): Skeletal Trauma in Children, ed 3. Philadelphia, PA, WB Saunders,
2003, pp 1-14.

AL-Madena Copy

196
2010 Pediatric Orthopaedic Examination Answer Book • 57

Question 70
A patient with Pott’s disease, tuberculosis of the spine, is more likely to have which of the following
early findings?

1. Acute onset back pain and neurologic dysfunction


2. Preservation of the disk space between two affected adjacent end plates
3. Involvement of the cervical spine and torticollis
4. Elevated WBC count and markedly elevated erythrocyte sedimentation rate
5. Lordotic deformity in late stages of the disease
PREFERRED RESPONSE: 2

DISCUSSION: Tuberculosis of the spine typically has an indolent presentation. Unlike pyogenic
infections of the spine, the disk space is usually preserved. Most commonly, the thoracic and lumbar
spine are affected. Laboratory studies may be nonspecific. Delayed presentation usually results in
neurologic compromise and a kyphotic deformity. Treatment includes a multidrug regimen. Surgery is
indicated for deformity correction or failure of medical treatment.
REFERENCES: Rajasekaran S: Buckling collapse of the spine in childhood spinal tuberculosis. Clin
Orthop Relat Res 2007;460:86-92.
TAL-MaeKXCpy I Hu SS: Spinal infections. J Am Acad Orthop Surg 2002; 10:188-197fcUa-MJ ^

Figure 71a Figure 71b Figure 71c Figure 71 d


Question 71
A 10-year-old child was referred for spinal curvature and a 2-year history of back pain. She has pain
during the day and pain at night that wakes her from sleep and is temporarily relieved with
nonsteroidal anti-inflammatory drugs. Examination shows very tight hamstrings and an irritative
spinal curvature. Figures 71a through 71 d show radiographs, a bone scan, and a CT scan. What is the
most appropriate treatment?

1. Bracing with a thoracolumbosacral orthosis (TLSO)


2. Observation with repeat radiographs of the scoliosis in 3 months and nonsteroidal
antiinflammatory drugs for the pain
3. MRI of the neuro-axis
4. Surgical removal
5. Radiofrequency ablation

PREFERRED RESPONSE: 4

DISCUSSION: The history, examination findings, and studies are consistent with an osteoid osteoma.
The CT scan shows a classic “target” lesion, and the bone scan has intense uptake at the site of the
osteoid osteoma. The child has had a 2-year history of pain that even wakes her from sleep, so
observation
AL-Madena and
Copyanti-inflammatory drugs is not a preferred treatment. Bracing will not help with the

197
58 • American Academy of Orthopaedic Surgeons

discomfort because the pain is not mechanical in nature. MRI would not be needed in addition to the
studies already completed. The osteoid osteoma is close to the spinal cord so radiofrequency ablation is
not preferred. Surgical removal and biopsy is the treatment of choice.

REFERENCES: Frassica FJ, Waltrip RL, Sponseller PD, et al: Clinicopathologic features and treatment
of osteoid osteoma and osteoblastoma in children and adolescents. Orthop Clin North Am 1996;27:559-
574. Cantwell CP, Obyme J, Eustace S: Current trends in treatment of osteoid osteoma with an emphasis
on radiofirequency ablation. Eur Radiol 2004;14:607-617.

Figure 72

Question 72
A 10‘/2-year-old boy sustained the injury shown in Figure 72 when he fell out of a tree. This is a closed,
neurologically intact injury and the patient has no head injury or loss of consciousness. He weighs 115
pounds and is otherwise healthy. What is the optimal treatment option for this injury?

1. Immediate spica casting


2. Flexible intramedullary nail placement
3. Traction and casting
4. External fixation
5. Solid intramedullary nail fixation via the greater trochanter

PREFERRED RESPONSE: 5

DISCUSSION: Although flexible intramedullary nails are a good treatment alternative for femoral shaft
fractures in older children, patients weighing more than 100 pounds have a higher incidence of
complications that include bending of the nails. Therefore, transtrochanteric solid intramedullary nail
fixation is most likely the best option for this patient. Using a greater trochanteric entry point avoids the
piriformis fossa and the possibility of osteonecrosis. External fixation is not a good alternative for this
patient because of the transverse nature of the fracture. External fixation of this fracture pattern has been
associated with a high refracture rate. Traction and casting can be performed but results in a lengthy
hospital stay and a very large cast in an overweight 10-year-old child.

REFERENCES: Flynn JM, Schwend RM: Management of pediatric femoral shaft fractures. J Am Acad
Orthop Surg 2004;12:347-359.
Gordon JE, Swenning TA, Burd TA, et al: Proximal femoral radiographic changes after lateral
transtrochanteric intramedullary nail placement in children. J Bone Joint Surg Am 2003;85:1295-1301.

AL-Madena Copy

198
199
62 • American Academy of Orthopaedic Surgeons

Figure 71a Figure 71b Figure 71c Figure 71d

Question 71
A 10-year-old child was referred for spinal curvature and a 2-year history of back pain. She has pain
during the day and pain at night that wakes her from sleep and is temporarily relieved with
nonsteroidal anti-inflammatory drugs. Examination shows very tight hamstrings and an irritative
spinal curvature. Figures 71a through 7Id show radiographs, a bone scan, and a CT scan. What is the
most appropriate treatment?

1. Bracing with a thoracolumbosacral orthosis (TLSO)


2. Observation with repeat radiographs of the scoliosis in 3 months and nonsteroidal
antiinflammatory drugs for the pain
3. MRI of the neuro-axis
4. Surgical removal
5. Radiofrequency ablation

PREFERRED RESPONSE: 4

DISCUSSION: The history, examination findings, and studies are consistent with an osteoid osteoma.
The CT scan shows a classic “target” lesion, and the bone scan has intense uptake at the site of the
osteoid osteoma. The child has had a 2-year history of pain that even wakes her from sleep, so
observation and anti-inflammatory drugs is not a preferred treatment. Bracing will not help with the
discomfort because the pain is not mechanical in nature. MRI would not be needed in addition to the
studies already completed. The osteoid osteoma is close to the spinal cord so radiofrequency ablation is
not preferred. Surgical removal and biopsy is the treatment of choice.

REFERENCES: Frassica FJ, Waltrip RL, Sponseller PD, et al: Clinicopathologic features and treatment
of osteoid osteoma and osteoblastoma in children and adolescents. Orthop Clin North Am 1996;27:559-
574. Cantwell CP, Obyme J, Eustace S: Current trends in treatment of osteoid osteoma with an emphasis
on radiofrequency ablation. Eur Radiol 2004;14:607-617.

AL-Madena Copy

200
2010 Pediatric Orthopaedic Examination Answer Book • 63

Figure 72

Question 72
A lO'/z-year-old boy sustained the injury shown in Figure 72 when he fell out of a tree. This is a closed,
neurologically intact injury and the patient has no head injury or loss of consciousness. He weighs 115
pounds and is otherwise healthy. What is the optimal treatment option for this injury?

1. Immediate spica casting


2. Flexible intramedullary nail placement
3. Traction and casting
4. External fixation
5. Solid intramedullary nail fixation via the greater trochanter

PREFERRED RESPONSE: 5

DISCUSSION: Although flexible intramedullary nails are a good treatment alternative for femoral shaft
fractures in older children, patients weighing more than 100 pounds have a higher incidence of
complications that include bending of the nails. Therefore, transtrochanteric solid intramedullary nail
fixation is most likely the best option for this patient. Using a greater trochanteric entry point avoids the
piriformis fossa and the possibility of osteonecrosis. External fixation is not a good alternative for this
patient because of the transverse nature of the fracture. External fixation of this fracture pattern has been
associated with a high refracture rate. Traction and casting can be performed but results in a lengthy
hospital stay and a very large cast in an overweight 10-year-old child.

REFERENCES: Flynn JM, Schwend RM: Management of pediatric femoral shaft fractures. J Am Acad
Orthop Surg 2004;12:347-359.
Gordon JE, Swenning TA, Burd TA, et al: Proximal femoral radiographic changes after lateral
transtrochanteric intramedullary nail placement in children. J Bone Joint Surg Am 2003;85:1295-1301.

Figure 73a Figure 73b

AL-Madena Copy

201
64 • American Academy of Orthopaedic Surgeons

Question 73
A 15-year-old right-handed pitcher reports shoulder pain after throwing. His symptoms have been
present for 3 months and have been getting progressively worse. Clinical examination shows no atrophy
of the shoulder muscles, but he has pain with resisted motion of the shoulder, especially internal rotation.
Radiographs are shown in Figures 73a and 73b. What is the next step in the evaluation and treatment of
his shoulder pain?

1. MRI/arthrogram of the right shoulder


2. CT of the right proximal humerus
3. Bone biopsy of the right proximal humerus
4. Cessation of throwing for 6 to 8 weeks, followed by a progressive throwing program
5. Arthroscopic evaluation of the right shoulder

PREFERRED RESPONSE: 4

DISCUSSION: The patient has proximal humeral epiphyseolysis, otherwise known as “Little League
shoulder.” This is an overuse injury of the shoulder in the skeletally immature overhead throwing athlete.
Most frequently seen in pitchers, it usually develops after an increase in the amount or intensity of
throwing activity. Initial treatment involves cessation of throwing activities so the proximal humeral
growth plate injury can heal, followed by a gradual return to throwing.

REFERENCES: Chen FS, Diaz VA, Loebenberg M, et al: Shoulder and elbow injuries in the skeletally
immature athlete. J Am Acad Orthop Surg 2005;13:172-185.
Keeley DW, Hackett T, Keims M, et al: A biomechanical analysis of youth pitching mechanics. J Pediatr
Orthop 2008;28:452-459.
Sabick MB, Kim YK, Torry MR, et al: Biomechanics of the shoulder in youth baseball pitchers:
Implications for the development of proximal humeral epiphysiolysis and humeral retrotorsion. Am J
Sports Med 2005;33:1716-1722.
Question 74
A 12-year-old child with Duchenne’s muscular dystrophy has a 40-degree scoliotic deformity. Prior
to surgery, the orthopaedic surgeon should

1. wait for further progression.


2. request a hematology consult.
3. request a neurology consult.
4. request a cardiology consult.
5. implement a 6-month trial of bracing.

PREFERRED RESPONSE: 4

DISCUSSION: In Duchenne’s muscular dystrophy, spinal deformities are common. Spinal deformity
usually develops as a child begins sitting in the preteen years. Unlike adolescent idiopathic scoliosis,
scoliosis in Duchenne’s muscular dystrophy is treated early; spinal fusion for a 40-degree deformity
is not unusual. Although hematology and neurology consults usually are not necessary prior to
surgery, every child should have a comprehensive cardiac evaluation, including an EKG and an
echocardiogram because cardiomyopathy is part of the pathologic spectrum of Duchenne’s muscular
dystrophy requiring preoperative assessment and intervention.

AL-Madena Copy

202
2010 Pediatric Orthopaedic Examination Answer Book • 65

REFERENCE: Fischgrund JS (ed): Orthopedic Knowledge Update 9. Rosemont, IL, American Academy
of Orthopaedic Surgeons, 2008, p 790.

Question 75
An 8-month-old child is seen in the emergency department with seizures and a fractured femur. The
mother states that the child fell off the bed at the babysitter’s house. There are bilateral bruises on the
anterior and posterior chest walls. Retinal hemorrhages are present. The temperature is 98.9 degrees
F (37.2 degrees C). What is the most likely diagnosis?

1. Febrile seizure
2. Fractured skull
3. Subdural hematoma
4. Shaken baby syndrome
5. Contracoup brain injury

PREFERRED RESPONSE: 4

DISCUSSION: Shaken baby syndrome is associated with chest ecchymosis and head trauma. Retinal
hemorrhages are often found, but are not pathognomonic. Contracoup injury was originally implicated,
but more recent evidence shows that the head is actually struck against a hard object that causes a
subdural hematoma.
REFERENCES: LeFanu J, Edwards-Brown R: Patterns of presentation of shaken baby syndrome:
Subdural and retinal haemorrhages are not necessarily signs of abuse. BMJ 2004;328:767.
Richards PG, Bertocci GE, Bonshek RE, et al: Shaken baby syndrome. Arch Dis Child 2006;91:205-206.

Question 76
A 12-year-old boy with a family history of neurofibromatosis has anterolateral bowing of the left tibia.
He has no pain and is ambulatory. Radiographs show a narrowed medullary canal but intact cortices.
Treatment should consist of which of the following?

1. Ankle-foot orthosis with anterior shell


2. Vascularized fibular graft
3. Intramedullary nailing of the left tibia
4. Amputation
5. Physical therapy

PREFERRED RESPONSE: 1

DISCUSSION: Anterolateral bowing of the tibia is associated with confirmed neurofibromatosis in


approximately 50% of patients. Although the risk of fracture with the development of pseudarthrosis
exists, the initial treatment consists of bracing through maturity.

REFERENCES: Vander Have KL, Hensinger RN, Caird M, et al: Congenital pseudarthrosis of the tibia.

AL-Madena Copy

203
66 • American Academy of Orthopaedic Surgeons
J Am Acad Orthop Surg 2008;16:228-236.
Vitale MG, Guha A, Skaggs DL: Orthopaedic manifestations of neurofibromatosis in children: An
update. Clin Orthop Relat Res 2002;401:107-118.

Question 77
A 4-year-old boy has had an isolated painful limp for the past month. He is diagnosed with Legg-Calve-
Perthes disease (LCPD) that involves nearly all of his capital femoral epiphysis. Which of the following
best describes his prognosis?

1. A spherical, painless hip at maturity


2. An incongruous hip joint at maturity
3. Likely spontaneous hip fusion
4. A 30% to 40% chance of a poor outcome
5. Rapid recovery with minimal sequelae
PREFERRED RESPONSE: 4

DISCUSSION: This young child with total head involvement LCPD is at some risk of a poor outcome
due to the extent of his disease. Most children of this age will recover well with a good outcome. He is
more likely to end up with a spherical femoral head than an older child with the same extent of
involvement.
AL-Madena Copy
REFERENCES: Schoenecker PL, Stone JW, Capelli AM: Legg-Perthes disease in children under 6
years old. Orthop Rev 1993;22:201-208.
Rosenfeld SB, Herring JA, Chao JC: Legg-Calve-Perthes disease: A review of cases with onset before
six years of age. J Bone Joint Surg Am 2007;89:2712-2722.

Figure 78

Question 78
Figure 78 shows the radiograph of a 4-year-old girl who has progressive bow legs. Management should
include which of the following?

1. Bracing
2. Observation
3. Tibial osteotomy
AL-Madena Copy
4. Physeal bridge resection

204
2010 Pediatric Orthopaedic Examination Answer Book • 67

5. Lateral physeal hemiepiphysiodesis plate

PREFERRED RESPONSE: 3

DISCUSSION: A diagnosis of Blount’s disease is indicated by the abnormal shape of the medial
metaphysis of the tibia, the progressive nature of the deformity, and the focal nature of the angulation. A
4-year-old child with Blount’s disease should undergo surgical correction consisting of a tibial
osteotomy before there is a permanent growth arrest that would require physeal bridge resection and/or
repeated osteotomies. With a Langenskiold type IV lesion, bracing or hemiepiphyseal plate fixation is
not expected to correct the deformity.

REFERENCES: Schoenecker PL, Meade WC, Pierron RL, et al: Blount’s disease: A retrospective
review and recommendations for treatment. J Pediatr Orthop 1985;2:181-186.
Langenskiold A: Tibia Vara: A critical review. Clin Orthop Relat Res 1989;246:195-207.
Bowen RE, Dorey FJ, Moseley CF: Relative tibial and femoral varus as a predictor of varus deformities
of the lower limbs in young children. J Pediatr Orthop 2002;22:105-111.

Figure 79

Question 79
A 12-year-old girl has the painful foot deformity seen in Figure 79. You advise her that she has juvenile
bunions. How do they differ from adult bunions?

1. Metatarsus primus varus


2. Large exostosis
3. Rigidity of the metatarsal phalangeal joint
4. Greater hallux valgus angle than in adult bunions
5. Prominent bursal thickening over the medial eminence

PREFERRED RESPONSE: 1

DISCUSSION: The hallmark of the juvenile bunion is metatarsus primus varus. Increased flexibility of
the first metatarsal phalangeal joint leads to increased deformity. The hallux valgus angle is less than the
adult bunion. Bursal thickenings and prominence of the medial eminence are less in a juvenile bunion.

REFERENCES: Coughlin MJ: Juvenile bunions, in Mann RA, Coughlin MJ (eds): Surgery of the Foot
and Ankle, ed 6. Philadelphia, PA, Mosley, 1993, pp 297-339.
Coughlin MJ, Mann RA: The pathophysiology of juvenile bunion. Instr Course Lect 1987;36:123-136.

AL-Madena Copy

205
68 • American Academy of Orthopaedic Surgeons

Figure 80a Figure 80b Figure 80c


Question 80
A 12-year-old girl is seen after tripping and twisting her ankle earlier in the morning. She had immediate
pain and swelling and was unable to bear weight. Radiographs are shown in Figures 80a through 80c.
Appropriate treatment should consist of which of the following?

1. Short leg cast with no weight bearing for the first 3 weeks
2. Short leg cast with immediate weight bearing
3. Long leg cast with no weight bearing for the first 3 weeks
4. Open reduction and internal fixation, avoiding the physeal growth plate and joint
5. CAM walker with immediate weight bearing

PREFERRED RESPONSE: 4

DISCUSSION: Salter Harris III and IV fractures of the medial malleolus often contain a large
cartilaginous portion, larger than the apparent ossified fragment seen on radiographs. Consequently,
articular incongruity is common. As such, open reduction and internal fixation is often required. Growth
disturbance and angular deformity are also common complications of Salter Harris III and IV fractures of
the distal tibia.

REFERENCES: Flynn JM, Skaggs DL, Sponseller PD, et al: The surgical management of pediatric
fractures of the lower extremity. Instr Course Lect 2003;52:647-659.
Kay RM, Matthys GA: Pediatric ankle fractures: Evaluation and treatment. J Am Acad Orthop Surg
2001;9:268-278.
Question 81
A 6-year-old child is seen in the emergency department after falling from the monkey bars. Examination
reveals tenderness of the right humerus and an inability to dorsiflex the wrist. No other injuries are
identified. Radiographs show a minimally displaced and angulated (10 degrees of varus angulation)
fracture of the distal one third of the humeral shaft. Initial management should consist of which of the
following?

1. Immediate exploration of the radial nerve and cast application


2. Immediate exploration of the radial nerve, with percutaneous Kirschner wire fixation

AL-Madena Copy

206
2010 Pediatric Orthopaedic Examination Answer Book • 69

3. Immediate exploration of the radial nerve with open reduction and plate fixation
4. Monitoring of radial nerve function and application of a sling and swathe
5. Monitoring of radial nerve function and external fixation

PREFERRED RESPONSE: 4

DISCUSSION: Humeral shaft fractures in children rarely require open reduction. Shoulder and elbow
function does not appear to be affected by up to 40 degrees of angulation in this patient population.
Because of the high rate of remodeling in pediatric patients, the standard treatment is immobilization in
a sling and swathe, a hanging arm cast, or a compressive dressing. Surgical fixation of humeral shaft
fractures is usually only necessary in open injuries, multitrauma, or severely displaced fractures. Most
radial nerve injuries associated with humerus fractures are secondary to contusion. Almost all associated
radial nerve injuries in pediatric patients can be treated with observation.

REFERENCES: Abel MF (ed): Orthopaedic Knowledge Update: Pediatrics 3. Rosemont, IL, American
Academy of Orthopaedic Surgeons, 2006, p 304.
Shrader MW: Proximal humerus and humeral shaft fractures in children. Hand Clin 2007;23:431-435.
Caviglia H, Garrido CP, Palazzi FF, et al: Pediatric fractures of the humerus. Clin Orthop Relat Res
2005;432:49-56.

Question 82
In infantile idiopathic scoliosis, which of the following factors suggests progression?

1. Age at presentation
2. Rib overlap of the apical vertebra
3. Rib vertebral angle difference of greater then 10 degrees
4. Male gender
5. Family history

PREFERRED RESPONSE: 2
DISCUSSION: Infantile idiopathic scoliosis occurs more commonly in boys, with a 3 to 1 male to female
ratio. Neural axis abnormalities, hip dysplasia, and congenital heart disease are all associated with the
condition; spontaneous correction frequently occurs. Curve progression can be predicted by the rib vertebral
angle difference or the phase of the rib head. Rib overlap of the apical vertebral body or a rib vertebral angle
difference of greater than 20 degrees indicates that the curve is likely to progress. Gender, family history,
and age at presentation have not been found to be risk factors for progression.

REFERENCES: Mehta MH: The rib-vertebra angle in the early diagnosis between resolving and
progressive infantile scoliosis. J Bone Joint Surg Br 1972;54:230-243.
Fischgrund JS (ed): Orthopedic Knowledge Update 9. Rosemont, IL, American Academy of Orthopaedic
Surgeons, 2008, p 697.

AL-Madena Copy

207
70 • American Academy of Orthopaedic Surgeons

Question 83
A 3-year-old child sustains a T2/T3 fracture-dislocation with complete paraplegia secondary to a car
accident in which the child was an unrestrained passenger. What is the likelihood that this child will
develop subsequent spinal deformity in the future?

1. 0% if bracing is used
2. 25%
3. 50%
4. 75%
5. 90% or greater

PREFERRED RESPONSE: 5

DISCUSSION: More than than 90% of preadolescent children who sustain a significant spinal cord injury
subsequently develop scoliosis. Conversely, progressive paralytic spinal deformity is uncommon in the
postadolescent patient. Bracing has not been shown to be effective in the prevention of scoliosis in the
preadolescent patient with spinal cord injury.

REFERENCES: Mayfield JK, Erkkila JC, Winter RB: Spine deformity subsequent to acquired childhood
spinal cord injury. J Bone Joint Surg Am 1981;63:1401-1411.
Dearolf WW III, Betz RR. Vogl LC, et al: Scoliosis in pediatric spinal cord-injured patients. J Pediatr
Orthop 1990;10:214-218.
Mehta S, Betz RR. Mulcahey MJ, et al: Effect of bracing on paralytic scoliosis secondary to spinal cord
injury. J Spinal Cord Med 2004;27:S88-S92.
Question 84
What is the most common causative bacteria in septic arthritis in children?

1. Staphylococcus aureus
2. Brucella melitensis
3. Haemophilus influenzae
4. Kingella kingae
5. Streptococcus pneumonia

PREFERRED RESPONSE: 1

DISCUSSION: The spectrum of causative bacteria and frequency of occurrence of specific pathogens in
septic arthritis are similar to those seen in osteomyelitis, with Staphylococcus aureus being the most
common. Other common causative organisms include Kingella Kingae, Streptococcus pneumonia,
Klebsiella species, Salmonella, Brucella melitensis, and Haemophilus influenzae.

REFERENCES: Herring JA: Tachdjian’s Pediatric Orthopaedics, ed 4. Philadelphia, PA, WB


Saunders, 2008, p 2109.
Jackson MA, Nelson JD: Etiology and medical management of acute suppurative bone and joint
infections in pediatric patients. J Pediatr Orthop 1982;2:313-323.

AL-Madena Copy

208
2010 Pediatric Orthopaedic Examination Answer Book • 71

Question 85
A 10-year-old girl fell from her bike and now reports pain and swelling in the left knee and pain with
weight bearing. Examination reveals a left knee effusion and pain with range of motion. A radiograph
is shown in Figure 85. Treatment should consist of

1. a long leg cast in extension.


2. a long leg cast in 10 degrees of flexion.
3. closed reduction and long leg casting in 10 degrees of flexion.
4. aspiration of the hemarthrosis for comfort and a knee immobilizer for 6 weeks.
5. open or arthroscopic reduction and internal fixation

PREFERRED RESPONSE: 5
DISCUSSION: The child has a type III tibial spine avulsion fracture. When the avulsed fragment is
completely displaced, the preferred treatment is open or arthroscopic reduction of the fragment and
internal fixation with sutures or screws. Type I fractures are nondisplaced and can be treated with a
long leg cast; type II fractures are hinged and can be treated in a long leg cast if closed reduction is
successful. Many patients have some objective anterior cruciate ligament laxity after a tibial spine
avulsion fracture; however, with adequate treatment most patients do not have symptomatic laxity.

REFERENCES: Mah JY, Adili A, Otsuka NY, et al: Follow-up study of arthroscopic reduction and
fixation of type III tibial-eminence fractures. J Pediatr Orthop 1998; 18:475-477.
McLennen JG: Lessons learned after second-look arthroscopy in type III fractures of the tibial spine. J
Pediatr Orthop 1995;15:59-62.
Meyers MH, McKeever FM: Fracture of the intercondylar eminence of the tibia. J Bone Joint Surg Am
1970;52:1677-1684.

AL-Madena Copy

209
72 • American Academy of Orthopaedic Surgeons

Question 86
A non-communicative 16-year-old girl with spastic quadriplegic cerebral palsy and a 75-degree
thoracolumbar scoliosis undergoes a successful posterior spinal fusion with instrumentation. What is the
most predictable outcome of the surgical procedure?

1. Improved cognitive function


2. Improved caregiver satisfaction
3. Improved nutrition
4. Decreased pain
5. Improved mobility

PREFERRED RESPONSE: 2

DISCUSSION: Surgical treatment of spinal deformity in a totally involved child with cerebral palsy has
been shown on outcomes instruments to significantly improve the caregiver’s perception of the child’s
comfort. The other parameters mentioned are difficult to measure and unpredictable.

REFERENCES: Tsirikos Al, Lipton G, Chang WN, et al: Surgical correction of scoliosis in pediatric
patients with cerebral palsy using the unit rod instrumentation. Spine 2008;33:1133-1140.
Cassidy C, Craig CL, Perry A, et al: A reassessment of spinal stabilization in severe cerebral palsy. J
Pediatr Orthop 1994;14:731-739.

Figure 87
Question 87
A 12-year-old boy has had left thigh pain for the past 4 months. Examination shows lack of internal
rotation and abduction, and external rotation with hip flexion. A radiograph is shown in Figure 87.
What is the most appropriate treatment?

1. Physical therapy
2. In situ pinning
3. Reduction and percutaneous pinning
4. Surgical dislocation of the hip with reduction under direct vision
5. Spica casting
DISCUSSION: The patient has a stable slipped capital femoral epiphysis (SCFE). Preferred treatment of
stable SCFE is in situ pinning. In situ fixation of stable SCFE has an extremely low rate of osteonecrosis.
Gentle postural reduction with hip capsulotomy or surgical dislocation of the hip with reduction has been
PREFERRED RESPONSE: 2
advocated for unstable SCFE.

REFERENCES: Aronson DD, Peterson DA, Miller DV: Slipped capital femoral epiphysis: The case for
AL-Madena Copy

210
2010 Pediatric Orthopaedic Examination Answer Book • 73

internal fixation in situ. Clin Orthop Relat Res 1992;281:115-122.


Loder RT, Richards BS, Shapiro PS, et al: Acute slipped capital femoral epiphysis: The importance of
physeal stability. J Bone Joint Surg Am 1993;75:1134-1140.

Figure 88a Figure 88b


Question 88
A 4-year-old girl falls off a swing and injures her right elbow. The radiographs are shown in Figures 88a
and 88b. What is the most likely diagnosis?

1. Displaced right olecranon fracture


2. Right elbow dislocation with spontaneous reduction and entrapped medial epicondyle
fragment
3. Effusion of the right elbow without an identifiable radiographic fracture
4. Nondisplaced right lateral condyle fracture of the distal humerus
5. Right Gartland type I supracondylar humerus fracture

PREFERRED RESPONSE: 1

DISCUSSION: To accurately diagnose skeletal injuries around the elbow in children, the practitioner
must be knowledgeable about the progressive ossification centers. In this case, at age 4 years, a female
will have a partially ossified capitellum, radial head, and medial epicondyle. The trochlea, olecranon, and
lateral epicondyle ossification centers should remain fully cartilaginous at this stage of development. The
irregularity in the area of the olecranon on the radiograph represents a displaced fracture requiring
accurate reduction and fixation to ensure restoration of articular congruity and full elbow function. The
presence of a posterior fat pad sign on the radiograph is indicative of a traumatic effusion, but there is an
identifiable fracture in this patient. Olecranon fractures are commonly seen in children with osteogenesis
imperfecta.
REFERENCES: Cheng JC, Wing-Man K, Shen WY, et al: A new look at the sequential development of
elbow-ossification centers in children. J Pediatr Orthop 1998;18:161-167.
Parent S, Wedemeyer M, Mahar AT, et al: Displaced olecranon fractures in children: A biomechanical
analysis of fixation methods. J Pediatr Orthop 2008;28:147-151.
Beaty JH, Kasser JR (eds): Rockwood and Green’s Fractures in Children, ed 6. Philadelphia, PA,
Lippincott Williams and Wilkins, 2006, pp 591-660.

AL-Madena Copy

211
74 • American Academy of Orthopaedic Surgeons

Question 89
A 5-year-old boy is seen in the emergency department with a 2-day history of refusing to walk.
Examination shows that he has a temperature of 102.2 degrees F (39 degrees C) and limited range ot
motion of the right hip. The AP pelvic radiograph is normal. The WBC count is normal but the C-
reactive protein and erythrocyte sedimentation rate (ESR) are elevated. What is the next step in
management?

1. IV antibiotics
2. Oral antibiotics
3. Ibuprofen
4. Observation and repeat evaluation in 2 weeks
5. Aspiration of the right hip

PREFERRED RESPONSE: 5

DISCUSSION: The history, physical examination, and laboratory studies suggest a septic hip. Recent
studies indicate that a child with elevated ESR, a WBC count of greater than 12,000/mm3, a temperature
of greater than 38.5 degrees, and unwillingness to walk is very likely to have septic arthritis of the hip
versus toxic synovitis. The best way to confirm the diagnosis is by hip aspiration. No medications should
be started until a diagnosis is made. Toxic synovitis is common, but significantly less likely if three of
the above criteria are present. This condition usually responds well to ibuprofen, but requires close
observation. Septic hips are considered urgent conditions and therefore a repeat evaluation in 2 weeks is
inappropriate.

REFERENCES: Herring JA: Tachdjian’s Pediatric Orthopaedics, ed 4. Philadelphia, PA, WB Saunders,


2008, pp 2109-2113.
Abel MF (ed): Orthopaedic Knowlede Update: Pediatrics 3. Rosemont, IL, American Academy of
Orthopaedic Surgeons, 2006, pp 62-65.
Kocher MS, Mandiga R, Murphy JM, et al: A clinical practice guideline for treatment of septic arthritis
in children: Efficacy in improving process of care and effect on outcome of septic arthritis of the hip. J
Bone Joint Surg Am 2003;85:994-999.
Kocher MS, Mandiga R, Zurakowski D, et al: Validation of a clinical prediction rule for the
differentiation between septic arthritis and transient synovitis of the hip in children. J Bone Joint Surg
Am 2004;86:1629-1635.
Question 90
Abnormal autosomal-dominant genes typically result in what type of defect?

1. Structural
2. Enzymatic
3. Biochemical
4. Translocations
5. Deletions
PREFERRED RESPONSE: 1

DISCUSSION: Autosomal-dominant gene defects usually cause structural deformities. An example is


achondroplasia where the dominant gene codes for fibroblast growth factor receptor 3, a structural

AL-Madena Copy

212
2010 Pediatric Orthopaedic Examination Answer Book • 75

protein that results in the quantitative decrease in cartilage formation. The gene defects are inherited
from one of the parents or are sporadic mutations. Autosomal-recessive genes code for enzymatic and
biochemical defects. Translocations and deletions relate to chromosomal abnormalities.
REFERENCE: Einhom TA, O’Keefe RJ, Buckwalter JA (eds): Orthopaedic Basic Science: Foundations
of Clinical Practice, ed 3. Rosemont, IL, American Academy of Orthopaedic Surgeons, 2007, pp 86-99.

Question 91
A 10-year-old boy is struck by a car and sustains open left tibia and fibula fractures with bone
protruding through a 7-cm laceration, multiple deep and superficial abrasions over the anterior leg, and
road gravel is present in the wounds. His foot is warm and well-perfused with normal sensation and he
has no pain with passive range of motion of the toes. Optimal treatment should consist of
1. irrigation and debridement of the fractures and application of an external fixator.
2. irrigation and debridement of the fractures and a reamed intramedullary nail.
3. irrigation and debridement of the fracture and percutaneous Kirschner wire fixation.
4. submuscular plating.
5. reduction and a short leg cast.
PREFERRED RESPONSE: 1

DISCUSSION: The patient has a grade 2 open fracture and therefore needs wound debridement as a first
step, followed by fracture stabilization preferably with an external fixator. A reamed intramedullary nail
is not indicated in a 10-year-old child with open growth plates. Submuscular plating is not needed in an
open fracture and there is no mention of fracture debridement. Percutaneous Kirschner wires will not
provide adequate fracture stabilization, nor will a short leg cast. Flexible nailing should be considered as
another form of fixation.
REFERENCES: Buckley SL, Smith G, Sponseller PD, et al: Open fractures of the tibia in children. J
Bone Joint Surg Am 1990;72:1462-1469.
Song KM, Sangeorzan B, Benirschke S, et al: Open fractures of the tibia in children. J Pediatr Orthop
AL-Madena Copy
Question 92
Brace treatment is recommended for adolescent idiopathic scoliosis when which of the following findings
is present?

1. Any patient with a curve of greater than 25 degrees


2. Boys with a curve of greater than 20 degrees
3. Premenarchal girls with a curve of greater than 30 degrees
4. More than 5 degrees of progression in a growing child with a 20-degree curve
5. A girl who is Risser 4 with a 30-degree curve

PREFERRED RESPONSE: 3

DISCUSSION: Brace treatment is recommended for a patient with substantial growth potential (Risser
2), any curve of greater than 30 degrees, or for a patient with a curve of greater than 20 to 25 degrees
with more than 5 degrees of documented progression. Skeletally immature patients who have a curve
of greater than 25 degrees are not considered for bracing. If a boy has a curve of 20 degrees,
progression would be necessary to recommend bracing. A premenarchal girl with the curve of greater
AL-Madena Copy

213
76 • American Academy of Orthopaedic Surgeons

than 45 degrees is best treated with early surgical intervention, as progression is extremely likely. A
girl who is Risser 4 with a 30-degree curve on presentation should be observed.

REFERENCE: Fischgrund JS (ed): Orthopedic Knowledge Update 9. Rosemont, IL, American Academy
of Orthopaedic Surgeons, 2008, p 698.

Figure 93a Figure 93b

Question 93
A 75-lb, 10-year-old boy fell and is now unable to walk and has left thigh pain and deformity. He has no
other injuries and is otherwise healthy. Radiographs are shown in Figures 93a and 93b. What is the most
appropriate management?

1. Retrograde elastic nail fixation


2. Traction now and a hip spica cast in 7 to 10 days
3. Antegrade intramedullary rod fixation
4. External fixation
5. Open reduction and plate fixation

PREFERRED RESPONSE: 1

DISCUSSION: The fracture is best treated with elastic nails. Traction and a spica cast could be used but
requires more time in the hospital and the cast increases family inconvenience for 4 to 6 weeks. There
are lateral entry nails that could be used, but to date there is still not a long enough follow-up to
determine the risk of osteonecrosis. External fixation may be an alternative for a patient with multiple
injuries or extensive soft-tissue damage. Open plating could also be used but in the midshaft location,
flexible nails are the most appropriate choice and offer the fewest number of potential complications.

REFERENCES: Abel MF (ed): Orthopaedic Knowledge Update: Pediatrics 3. Rosemont, IL, American
Academy of Orthopaedic Surgeons, 2006, pp 271-279.
Buechsenschuetz KE, Mehlman CT, Shaw KJ, et al: Femoral shaft fractures in children: Traction and
casting versus elastic stable intramedullary nailing. J Trauma 2002;53:914-921.
•b

AL-Madena Copy

214
2010 Pediatric Orthopaedic Examination Answer Book • 77

M
Figure 94
Question 94
A 7-year-old boy is seen for follow-up for a scoliotic deformity. His parents are concerned because
his deformity seems to have increased. He has no pain and is neurologically intact. A radiograph is
shown in Figure 94, and measurement of his curve reveals that it has increased 10 degrees. What is
the most appropriate recommendation for this patient at this time?
1. Observation
2. Bracing
3. A “growing rod”
4. Distraction instrumentation and posterior arthrodesis
5. Hemivertebra excision and limited fusion

PREFERRED RESPONSE: 5

DISCUSSION: Nakamura and associates have reported good results in patients with resection for
hemivertebra-related congenital scoliosis who have a progression of their deformity. Because of the
progression, observation is not appropriate for this patient’s deformity. Bracing has not been shown
to alter the progression of congenital scoliosis. The “growing rod” technique is also not effective in
preventing progression related to hemivertebra. Distraction instrumentation carries an increased risk
of neurologic complications in children with congenital spine deformities. Progression after posterior
arthrodesis alone can occur through the so-called “crankshaft phenomenon.”
REFERENCES: Nakamura H, Matsuda H, Konishi S, et al: Single-stage excision of hemivertebrae via
the posterior approach alone for congenital spine deformity: Follow-up period longer than ten years.
Spine 2002;27:110-115.
Ruf M, Harms J: Posterior hemivertebra resection with transpedicular instrumentation: Early correction
in children aged 1 to 6 years. Spine 2003;15:2132-2138.

AL-Madena Copy

215
78 • American Academy of Orthopaedic Surgeons

Figure 95a Figure 95b Figure 95c Figure

95d

Question 95
A 12-year-old boy is seen 1 week after injuring his knee while playing soccer. He notes pain and
swelling. Examination reveals an effusion, laxity with Lachman testing, and he walks with a limp.
Radiographs and an MRI scan are shown in Figures 95a through 95d. Treatment should consist of which
of the following?

1. Physical therapy
2. Anterior cruciate ligament reconstruction
3. Anterior cruciate ligament reconstruction when the child reaches skeletal maturity
4. Anatomic reduction and internal fixation of the fracture fragments
5. Casting in extension

PREFERRED RESPONSE: 4

DISCUSSION: The radiographs and MRI scan show a displaced tibial eminence fracture. Meyer and
McKeever classified these injuries, with type 1 being a nondisplaced tibial eminence fracture; type 2
being a displaced tibial eminence fracture with a posterior hinge, and type 3 being a displaced tibial
eminence fracture. Tibial eminence fractures in children are equivalent to anterior cruciate ligament tears
in adults. Treatment should be anatomic reduction, which often requires an arthroscopic or open
procedure, followed by fixation.

REFERENCES: Green NE, Swiontkowski MF: Skeletal Trauma in Children, ed 3. Philadelphia, PA, WB
Saunders, 2003, pp xvi, 452-455, 638.
Zionts LE: Fractures around the knee in children. J Am Acad Orthop Surg 2002;10:345-355.

AL-Madena Copy

216
2010 Pediatric Orthopaedic Examination Answer Book • 79

Figure 96

Question 96
A teenager had pain in the left buttock while running the hurdles. He was treated with 4 weeks of rest
and crutch walking, and then started physical therapy for stretching and muscle strengthening. Nine
months later he now reports pain with sitting and has not been able to resume running or sports activity.
Figure 96 shows a radiograph of the pelvis. Treatment should consist of which of the following?

1. Continued physical therapy


2. Spica casting for 6 weeks
3. Biopsy of the lesion
4. Steroid injection into the area of discomfort
5. Excision of the fragment

PREFERRED RESPONSE: 5

DISCUSSION: The patient has an established nonunion of the ischial tuberosity. Avulsion fractures of
the pelvis are generally treated with rest and symptomatic treatment. Avulsion fractures of the ischial
tuberosity are the most prone to nonunion. Most patients have few symptoms but some have trouble
sitting and returning to sports. Excision of the avulsed fragment or open reduction and internal fixation
are indicated for painful nonunions of the ischial tuberosity.

REFERENCES: Fembach SK., Wilkinson RH: Avulsion injuries of the pelvis and proximal femur. AJR
Am J Roentgenol 1981;137:581-584.
Watts HG: Fractures of the pelvis in children. Orthop Clin North Am 1976;7:615-624.
Question 97
Congenital anomalies of the vertebral column are associated frequently with other organ system
problems. In addition to radiographs of the spine, what other screening tests should be ordered?

1. Spinal MRI, coagulation panel


2. Liver enzymes, coagulation panel
3. Renal ultrasound, upper and lower GI
4. Cardiac evaluation/echocardiogram, upper and lower GI

AL-Madena Copy

217
80 • American Academy of Orthopaedic Surgeons

5. Renal ultrasound, cardiac evaluation/echocardiogram, spinal MRI

PREFERRED RESPONSE: 5

DISCUSSION: Approximately 60% of patients with congenital anomalies of the spine have other
associated findings. The spine develops around the same time as the cardiovascular system, the
genitourinary system, and the musculoskeletal system. Around 20% of patients with congenital
scoliosis have an associated urologic abnormality. Approximately 25% of patients with congenital
scoliosis have an associated cardiac defect. Spinal cord abnormalities in one study occurred in
approximately 37% of patients with congenital scoliosis.

REFERENCES: Basu PS, Elsebaie H, Noordeen MH: Congenital spinal deformity: A comprehensive
assessment at presentation. Spine 2002;27:2255-2259.
Ferguson RL: Medical and congenital comorbidities associated with spinal deformities in the immature
spine. J Bone Joint Surg Am 2007;89:34-41.
McMaster MJ, Ohtsuka K: The natural history of congenital scoliosis: A study of two hundred and
fifty- one patients. J Bone Joint Surg Am 1982;64:1128-1147.

AL-Madena Copy

218
2010 Pediatric Orthopaedic Examination Answer Book • 81

Figure 98a Figure 98b

Question 98
A 5-month-old child was referred for evaluation of right lower extremity bowing. The child’s
developmental history is normal and there is no pain or history of trauma. Figures 98a and 98b show AP
and lateral radiographs of the lower extremities. What is the natural progression of this condition?

1. Spontaneous resolution of the bowing by age 2 years without residual deformity


2. Spontaneous improvement or resolution of the bowing with a resulting limb-length
inequality at maturity
3. No change in the condition over time
4. Spontaneous fracture in the area of the bowing
5. Spread of the abnormal bone until the entire tibia is involved

PREFERRED RESPONSE: 4

DISCUSSION: The patient has anterior lateral bowing of the tibia with intramedullary sclerosis at the
site of the deformity. This is the prefracture stage of congenital pseudoarthrosis of the tibia and the child
is at risk for spontaneous fracture and nonunion. A clamshell orthosis may prevent or delay fracture.
Treatment of established nonunion ranges from bone grafting and intramedullary nailing to Uizarov
treatment, vascularized fibula bone grafting, or amputation.

REFERENCES: Herring JA: Disorders of the leg, bowing of the tibia, in Herring JA (ed): Tachdjian’s
Pediatric Orthopaedics, ed 4. Philadelphia, PA, WB Saunders, 2008, pp 1007-1023.
Ohnishi I, Sato W, Matsuyama J, et al: Treatment of congenital pseudarthrosis of the tibia: A
multicenter study in Japan. J Pediatr Orthop 2005;25:219-224.
Question 99

219
82 • American Academy of Orthopaedic Surgeons

A 12-year-old girl is seen for left ankle pain. Radiographs reveal osteochondritis dissecans
(OCD) involving the talus. What should the parents be told regarding management?

1. No treatment is required because spontaneous healing is common.


2. Nonsurgical management typically relieves pain and results in radiographic healing in
less than 12 weeks.
3. Nonsurgical management frequently relieves pain but often may not result in radiographic
healing even 6 months after treatment.
4. Hyperbaric oxygen treatment is helpful.
5. Ankle fusion is frequently necessary.

PREFERRED RESPONSE: 3

DISCUSSION: Nonsurgical management of OCD of the talus in skeletally immature individuals


frequently results in a fairly rapid decrease in symptoms, but radiographic abnormalities can frequently
be found even 6 months after treatment. Spontaneous resolution of this condition is rare. Hyperbaric
oxygen treatment has not been shown to be beneficial for this condition. Progression of the condition to
the point of requiring ankle fusion is rare.

REFERENCES: Perumal V, Wall E, Babekir N: Juvenile osteochondritis dissecans of the talus. J Pediat
Orthop 2007;27:821-825.
Letts M, Davidson D, Ahmer A: Osteochondritis dissecans of the talus in children. J Pediatr Orthop
2003;23:617-625.

Question 100
A 4-year-old child has droopy shoulders. Examination shows that the child has a large head, short
stature, and a narrow chest. Radiographs of the spine and chest show absent clavicles, delayed
ossification of the pubis and ischium, and mild coxa vara. What is the inheritance pattern for this
condition?

1. Autosomal dominant
2. Autosomal recessive
3. Sex-linked recessive
4. Sex-linked dominant
5. No inheritance pattern

PREFERRED RESPONSE: 1
DISCUSSION: The child has the clinical and radiographic features of cleidocranial dysostosis. This is a
disorder of bones formed by intramembranous ossification. It is inherited as an autosomal-dominant
condition. About two thirds of cases are familial.

REFERENCES: Dietz FR, Mathews KD: Update on the genetic bases of disorders with orthopaedic
manifestations. J Joint Bone Surg Am 1996;78:1583-1598.
Lee B, Thirunavukkarasu K, Zhou L, et al: Missense mutations abolishing DNA binding of osteoblast-
specific transcription factor OSF2/CBFA1 in cleidocranial dysplasia. Nat Genet 1997;16:307-310.
AL-Madena Copy cU.u^Ml <UjJlU

AL-Madena Copy

220

You might also like